Vous êtes sur la page 1sur 207

• 2 Item: 1 of 44 F' Mark

. 3 Previous Next Lab Values. Notes Calculator.

• 4
• 5
• 6 A 16-year-old girl comes to the physician for a routine examination. She is wearing a sweater despite the
. 7 warm weather and says that she often feels cold. She also complains of difficulty with weight loss_ The
• 8 patient wants to lose about 4.5 kg (10 Ib) because she believes she is "enormous." For the past year, she
• 9 has been on a vegetarian diet because meat makes her feel bloated. Menarche was at age 12 and her last
• 10 menstrual period was 2 months ago. The patient is an honors student and involved in ballet and
• 11 cross-country running. Her mother has Graves' disease. Temperature is 36_2 C (97.2 F), blood pressure is
• 12 88/58 mm Hg, pulse is 42/min, and respirations are 16/min. Her body mass index is 15 kg/m2. The patient is
• 13 alert and cooperative but appears tired. Her skin is dry with lanugo. Laboratory results are as follows:
• 14
Serum chemistry
• 15
Sodium 136 mEdiL
• 16
Potassium 2.9 mEq/L
• 17
Chloride 102 mEq/L
• 18
Bicarbonate 24 mEq/L
• 19
Blood urea nitrogen 18 mg/dL
• 20
Creatinine mg/dL
• 21
Calcium 8 mg/dL
• 22
Glucose 78 mg/dL
• 23
Magnesium 2.3 mg/dL
• 24
Phosphorus 2 mg/dL
• 25
• 26
Thyroid function tests
• 27
Thyroid-stimulating hormone 4.5 pU/mL
• 28
Free T4 ng/dL
• 29
Triiodothyronine (T3) 65 ngfdL
• 30
• 31
Urine human chorionic gonadotropin Negative
• 32
• 33
. 34
What is the most appropriate next step in management of this patient's symptoms?
• 35
• 36 A. Administer bupropion
• 37
• B. Administer fluoxetine
• 38
• 39 • C. Administer levothyroxine
• 40 (--) Ariminktpr nlAn7Anine se
• 41
• 42 0
Feedback Suspend End Block
• 2 Item: 1 of 44 .11V-Mark
• 3 Previous Next Lab Values. Notes Calculator.
• 4 menstrual period was 2 months ago. The patient is an honors student and involved in ballet and
• 5 cross-country running. Her mother has Graves' disease. Temperature is 36_2 C (972 F), blood pressure is
• 6 88/58 mm Hg, pulse is 42/min, and respirations are 16/min. Her body mass index is 15 kg/m2. The patient is
• 7 alert and cooperative but appears tired. Her skin is dry with lanugo. Laboratory results are as follows:
• 8
Serum chemistry
• 9
Sodium 136 mEq/L
• 10
Potassium 2.9 mEq/L
• 11
Chloride 102 mEq/L
• 12
Bicarbonate 24 mEq/L
• 13
Blood urea nitrogen 18 mg/dL
• 14
Creatinine 0.6 mgldL
• 15
Calcium 8 mg/dL
• 16
Glucose 78 mg/dL
• 17
Magnesium 2.3 mgfdL
• 18
Phosphorus 2 mg/dL
• 19
• 20
Thyroid function tests
• 21
Thyroid-stimulating hormone 4.5 pU/mL
• 22
Free T4 GA ng/dL
• 23
Triiodothyronine (T3) 65 ng/dL
• 24
• 25
Urine human chorionic gonadotropin Negative
• 26
• 27
What is the most appropriate next step in management of this patient's symptoms?
• 28
• 29
• 30 O A. Administer bupropion
• 31
0 B. Administer fluoxetine
• 32
• 33
O C. Administer levothyroxine
• 34 • D. Administer olanzapine
• 35
• E Hospitalization
• 36
• 37
O F. Order serum 25-hydroxy-vitamin
• 38 O G. Order serum cobalamin
• 39
O H. Refer for cognitive behavioral therapy, with follow-up in 2 weeks
• 40
• 41
• 42 0
Feedback Suspend End Block
2 Item: 2 of 44 F' Mark
3 Previous Next Lab Values. Notes Calculator.
• 4
5
6 A 4-year-old boy is brought to the physician for evaluation of speech that is "difficult to understand." His
▪ 7 parents state that "he doesn't seem to listen" and describe him as a slow learner who only uses single words
▪ 8 and frequently mumbles. The boy prefers to play by himself and is obsessed with building towers with
▪ 9 blocks_ He becomes extremely distressed if prevented from performing this activity. He was born at term
• 10 from an uncomplicated pregnancy and delivery, and his physical growth has been normal. His older brother
• 11 was aggressive as a young child and diagnosed with attention deficit hyperactivity disorder. In the office
• 12 waiting room, the boy plays quietly with blocks and seems oblivious to the other children. When his mother
• 13 calls him to come to her, he turns toward her once and then resumes playing. His physical examination and
• 14 hearing are normal. Which of the following is the most appropriate statement to tell his parents?
• 15
• 16
• 17 • A. "He most likely has a developmental language disorder and would benefit from speech therapy."
• 18 • B. "His behavior is most likely due to the inattentive subtype of attention deficit hyperactivity disorder."
• 19
• 20 • C. "I am concerned that his difficulties may indicate intellectual disability.'
• 21 • D. "I understand your concerns; a brain MRI will help establish a definitive diagnosis."
• 22
• 23 E "I understand your concerns; a comprehensive evaluation for autism spectrum disorder will be
• 24 ".•-1 helpful."
• 25
• 26
• 27
• 28
• 29
• 30
• 31
• 32
• 33
• 34
• 35
• 36
• 37
• 38
• 39
• 40
• 41
• 42 0
Feedback Suspend End Block
1
Item: 3 of 44 F' Mark
Previous Next Lab Values. Notes Calculator.

▪ 5
▪ 6 A 12-year-old boy is accused of setting his neighbor's house on fire. His parents say, 'We can't believe he
▪ 7 would do anything like this; he has always been hyperactive but is basically a good kid_ The past year he has
▪ 8 been a little more irritable and argumentative with us, but that's just typical of preteens." They add that he has
▪ 9 wanted to be a firefighter since a young age and spends hours on the internet watching videos about fires and
• 10 firefighting_ The boy's grades are average, and his teachers describe him as a loner who has difficulty making
• 11 friends at school_ Two years ago he was caught setting the interior of his father's car on fire_ The boy has
• 12 also been linked to several suspicious fires in the neighborhood, although no criminal charges have been
• 13 brought against him. Based on this information, which of the following is the most likely diagnosis?
• 14
• 15
• 16 • A. Antisocial personality disorder
• 17 • B. Attention deficit hyperactivity disorder
• 18
• 19
• C. Conduct disorder
• 20 • D. Normal developmental experimentation
• 21
• 22
• E Oppositional defiant disorder
• 23 C. F. Pyromania
• 24
• 25
• 26
• 27
• 28
• 29
• 30
• 31
• 32
• 33
• 34
• 35
• 36
• 37
• 38
• 39
• 40
• 41
• 42 411 0
Feedback Suspend End Block
1
2 Item: 4 of 44 F' Mark
3 Previous Next Lab Values. Notes Calculator.
4 tenderness to deep palpation; there is no rebound tenderness or guarding. Bowel sounds are normal.
5 Bilateral lower extremities have nonblanching palpable purplish lesions below the knee and mild swelling of the
6 feet. The remainder of the examination is normal_
7
▪ 8 Laboratory results are as follows:
▪ 9 Complete blood count
• 10 Hemoglobin 13.0 WI:IL
• 11 Hematocrit 40%
• 12 Platelets 140,000/p L
• 13 Leukocytes 7,500/pL
• 14
• 15 Urinalysis
• 16 Specific gravity 1.025
• 17 pH TO
• 18 Protein +1
• 19 Blood Moderate
• 20 Glucose Negative
• 21 Ketones Negative
• 22 Leukocyte esterase Negative
• 23 Nitrites Negative
• 24 Bacteria None
• 25 White blood cells 1-2/hpf
• 26 Red blood cells 20-30/hpf
• 27 Casts None
• 28 Crystals None
• 29
• 30 Which of the following is the most likely cause of this child's skin manifestations?
• 31
• 32
• 33
• A. Bacterial infection
• 34 • B. Bone marrow infiltration
• 35
• C. Clotting factor deficiency
• 36
• 37
• ID. Nutritional deficiency
• 38 • E Vasculitis
• 39
• Viral infection
• 40
• 41
• 42 0
Feedback Suspend End Block
1
2 Item: 5 of 44 F' Mark
3 Previous Next Lab Values. Notes Calculator.
4

A I D-year-old boy is brought to the emergency department due to abdominal pain and bloody diarrhea. The
mother says that he was "fine" a few days ago, and then suddenly became ill. Physical examination shows a
▪ 7
pale and jaundiced child_ There is diffuse abdominal tenderness and 2+ pedal edema_ Laboratory studies
▪ 8
show anemia, thrombocytopenia and renal insufficiency_ What is the most likely cause of the patient's
▪ 9
symptoms?
• 10
• 11 • A. Vibrio cholera
• 12
• B. Escherichia coli
• 13
• 14 • C. Crohn's disease
• 15
• D. Lactose intolerance
• 16
• 17 • E. Salmonella poisoning
• 18
• F. Campylobacter jejuni
• 19
• 20
• 21
• 22
• 23
• 24
• 25
• 26
• 27
• 28
• 29
• 30
• 31
• 32
• 33
• 34
• 35
• 36
• 37
• 38
• 39
• 40
• 41
• 42 0
Feedback Suspend End Block
1
2 Item: 6 of 44 F' Mark
3 Previous Next Lab Values. Notes Calculator.
4

A 9-year-old Caucasian male complains of fever, sore throat and difficulty swallowing. Small tender lymph
nodes are palpated in the cervical region_ The symptoms subside quickly on penicillin therapy. Ten days
▪ 8 later, the patient presents again with fever, skin rash and fleeting joint pain in the lower extremities. Physical
▪ 9 examination reveals scattered urticaria and palpable lymph nodes in the cervical, axillary and inguinal regions.
• 10 Which of the following is the most likely cause of this patient's current complaints?
• 11
• 12
• 13 • A. Rheumatic fever
• 14 O B. Drug-induced reaction
• 15
• 16 • C. Lymphoproliferative disorder
• 17 • Henoch-Schonlein purpura
• 18
• 19
O E Infective endocarditis
• 20
• 21
• 22
• 23
• 24
• 25
• 26
• 27
• 28
• 29
• 30
• 31
• 32
• 33
• 34
• 35
• 36
• 37
• 38
• 39
• 40
• 41
• 42 0
Feedback Suspend End Block
Item: 7 of 44 F' Mark
Previous Next Lab Values. Notes Calculator.

A 5-year-old girl with chronic renal insufficiency is brought to the physician for a follow-up visit Since birth,
she has had multiple episodes of urinary tract infections, for which she takes trimethoprim-sulfamethoxazole
daily for prophylaxis. She has a history of poor growth and mild hypertension but is otherwise developmentally
9 normal. Her mother lost custody of the girl 3 years ago due to failure to comply with recommended treatment
• 10 and prophylaxis of her infections. The girl has since lived with her grandmother. Examination shows mild
• 11 bilateral lower-extremity edema but no other abnormalities_ Urinalysis shows mild proteinuria but no white
• 12 blood cells or bacteria. Renal scintigraphy with dimercaptosuccinic acid shows bilateral focal parenchymal
• 13 scarring and blunted calyces. Which of the following is the most likely predisposing factor for this patient's
• 14 recurrent infections?
• 15
• 16
• 17
• A. Common variable immunodeficiency
• 18 • B. Neurogenic bladder
• 19
• C. Posterior urethral valves
• 20
• 21 O ID. Polycystic kidney disease
• 22
• E Recurrent sexual abuse
• 23
• 24 O F. Unilateral renal agenesis
• 25
O G. Vesicoureteral reflux
• 26
• 27
• 28
• 29
• 30
• 31
• 32
• 33
• 34
• 35
• 36
• 37
• 38
• 39
• 40
• 41
• 42 0
Feedback Suspend End Block
1
2 Item: 8 of 44 F' Mark
3 Previous Next Lab Values. Notes Calculator.
4-
5
6 A healthy 15-year-old girl comes to the physician for a routine health maintenance examination. She feels well
7 and has no concerns. Her menstrual cycles are regular and last 3-4 days; her last menses was 1 week ago_
8 The patient has been sexually active with one partner for the past year and takes oral contraceptive pills daily.
9 She has no vaginal discharge or pain. She is an honors student in 1 lath grade and plays varsity soccer. The
• 10 patient has tried marijuana ''a few times" but does not use tobacco or alcohol_ Her parents are healthy_ Her
• 11 maternal grandfather died of a myocardial infarction at age 68. The patient's body mass index is 23 kgim2.
• 12 Vital signs and physical examination are normal. What is the best next step in the evaluation of this patient?
• 13
• 14
• 15
• A. Chtarnydia trachornatis testing
• 16 • B. Complete blood count
• 17
• 18
• C. Echocardiography
• 19 • ID. Fasting lipid panel
• 20
• 21
• E Urine culture
• 22 • F. Urine toxicology screen
• 23
• 24
• 25
• 26
• 27
• 28
• 29
• 30
• 31
• 32
• 33
▪ 34
• 35
• 36
• 37
• 38
• 39
• 40
• 41
• 42 0
Feedback Suspend End Block
• 1
• 2 Item: 9 of 44 V- Mark
• 3 Previous Next Lab Values. Notes Calculator.
• 4-
• 5
• 6 A 3-year-old girl is brought to the emergency department with lethargy and fever_ She had diarrhea for several
• 7 days, and her parents say that "she suddenly took a turn for the worse". The girl has refused liquids for the
past 12 hours and has not urinated today. She has no allergies and takes no medications. Her temperature
is 397 C (1113_5 F), blood pressure is 60/28 mm Hg, and pulse is 145/min. On examination, she is lethargic
• 10 and has poor skin turgor. Her capillary refill time is 5 seconds centrally_ Despite numerous attempts, it is not
• 11 possible to start a peripheral intravenous line, and the child's condition continues to deteriorate. Which of the
• 12 following is the best next step in management of this patient?
• 13
• 14
• 15
• A. Attempt arterial line placement
• 16 • R. Attempt central venous catheter placement
• 17
• 18
• C. Attempt intraosseous cannulation
• 19 • ID. Attempt nasogastric tube placement
• 20
• 21
• E Attempt peripheral intravenous line placement
• 22 • F. Transport to intensive care unit for central venous catheter placement
• 23
• 24
• 25
• 26
• 27
• 28
• 29
• 30
• 31
• 32
• 33
• 34
• 35
• 36
• 37
• 38
• 39
• 40
41
42 0
Feedback Suspend End Block
1
2 Item: 10 of 44 V/lark
3 Previous Next Lab Values. Notes Calculator.
4-
5
6 A 5-year-old boy is brought to the physician after he was hit in the head with a baseball_ The incident occurred
7 about two hours ago_ He had no loss of consciousness, but was a little dazed after being hit However, he
8 seemed to improve after a short time. About 30 minutes ago, the patient began to complain of a headache,
vomited twice, and is currently not acting right according to his parents_ On examination, the child is sleepy
and his left pupil is larger than his right A CT scan of his head is shown below_
• 11
• 12
• 13
• 14
• 15
• 16
• 17
• 18
• 19
• 20
• 21
• 22
• 23
• 24
• 25
• 26
• 27
• 28
• 29
• 30
• 31
• 32
• 33
• 34
• 35
• 36
• 37
• 38
• 39
• 40
se
41
42 0
Feedback Suspend End Block
• 1
• 2
3 Previous Lab Values. Notes Calculator.
4-
5
6 7
1:
78
9

1
• 11
• 12
• 13
• 14
• 15
• 16 'I
• 17
• 18
• 19
• 20
• 21
• 22
• 23
• 24
• 25
• 26
• 27
• 28
• 29
• 30
• 31
Which of the following is the most appropriate next step in management?
• 32
• 33
• 34 • A. Cerebral angiogram
• 35
• B. MRI of the brain
• 36
• 37
• C. Repeat CT in 24 hours
• 38 .0 ID. IDexamethasone
• 39
• E Emergent craniotomy
• 40
• 41
• 42 0
Feedback Suspend End Block
• 1
• 2 Item: 11 of 44 V/lark
▪ 3 Previous Next Lab Values. Notes Calculator.
• 4-
▪ 5
▪ 6 A 12-year-old boy is brought to the pediatrician by his mother for a routine examination. The boy has an
▪ 7 eight-year history of episodes in which he utters strange sounds, clears his throat, grimaces, and blinks his
▪ 8 eyes. Recently he has also started to jerk his head during these episodes as well. His medical history is
• 9 otherwise unremarkable and he is earning passing grades in school. Physical examination is unremarkable_
• 10 Which of the following disorders is most likely to develop in this boy?
• 11
• 12
• 13 • A. Conduct disorder
• 14
• B. Obsessive-compulsive disorder
• 15
• C. Oppositional defiant disorder
• 17
• ID. Narcissistic personality disorder
• 18
• 19 • E. Antisocial personality disorder
• 20
• 21
• 22
• 23
• 24
• 25
• 26
• 27
• 28
• 29
• 30 •

• 31
• 32
• 33
• 34
• 35
• 36
• 37
• 38
• 39
• 40
• 41
• 42 0
Feedback Suspend End Block
• 1
• 2 Item: 12 of 44 F' Mark
▪ 3 Previous Next Lab Values. Notes Calculator.
• 4
▪ 5

A 2-year-old boy with cough and difficulty breathing is brought to the emergency department by his mother_
▪ 7 She says that he was well and playing with his toys until 2 hours prior to presentation. He is healthy, but his
▪ 8 6-year old brother has a peanut allergy. The patient's temperature is 36.7 C (98 F), blood pressure is 92148
mm Hg, pulse is 114/min, and respirations are 48/min. The patient's pulse oximetry shows 91% on room air_
• 10 Physical examination shows nasal flaring and grunting with both subcostal and intercostal retractions.
• 11 Wheezing is heard in the right lung field; the left field is clear to auscultation_ No rales or rhonchi are noted.
The remainder of the physical examination is within normal limits. Supplementary oxygen is applied. Chest
x-ray reveals mild hyperinflation of the right lung. Which of the following is the most appropriate next step in
management of this child?

• 17
• A. Bronchoscopy
• 18 • B. Chest computed tomography scan
• 19
• C. Chest physiotherapy
• 20
• 21 • D. Chest tube placement
• 22
• E Intramuscular epinephrine
• 23
• 24 • Nebulized albuterol
• 25
• G. Racemic epinephrine
• 26
• 27
• 28
• 29
• 30
• 31
• 32
• 33
• 34
• 35
• 36
• 37
• 38
• 39
• 40
• 41
• 42 0
Feedback Suspend End Block
• 1
• 2 Item: 13 of 44 V-• Mark
3 Previous Next Lab Values. Notes Calculator.
4 I
5
6 7 A 1-day-old boy is in the newborn nursery with persistent feeding difficulty. He is cyanotic and short of breath
7 when he breastfeeds but turns pink when he cries. Feeding him expressed breast milk from a bottle results in
8 1 the same findings. He is voiding normally and passed meconium. His prenatal, birth, and family histories are
9 unremarkable. His weight is average for gestational age. Vital signs are normal and 4-extremity blood
• 10 M pressures are equal. Examination shows a non-dysmorphic boy with clear lungs and no murmurs_ There are
:
h
11
2
no intercostal retractions or nasal flaring. His peripheral pulses are full and symmetric. Which of the following
is the most likely diagnosis?

• 14
• 15 • A. Choanal atresia
• 115'1
• 17
• 18 I
• Laryngomalacia
• C. Tetralogy of Fallot
• 19 • ID. Tracheoesophageal fistula with esophageal atresia
• 20
• 21 • E Transient tachypnea of the newborn
• 22 • F. Vocal cord paralysis
• 23
• 24
• 25
• 26
• 27
• 28
• 29
• 30
• 31
• 32
• 33
• 34
• 35
• 36
• 37
• 38 I
• 39
• 40
• 41
• 42 411 0
Feedback Suspend End Block
A . 1
3 =1
1
4
Item: 14 of 44 V-• Mark
Previous Next Lab Values. Notes Calculator.

5
6 7 A 3-year-old boy is brought to the emergency department because of severe oral pain along with bleeding and
irritation of his gums_ His symptoms began 2 weeks ago and have progressively worsened. He has a history
8 of numerous sinus infections and episodes of cellulitis, with previous cultures isolating Staphylococcus
9 aureus and Pseuciornorras aerugirrosa. His mother also notes that his umbilical cord did not fall off until 5
• 10 weeks after birth_ On physical examination, the child has a severe, necrotic periodontal infection with
• 11 ulceration.
• 12
• 13 Laboratory results are as follows:
Complete blood count
• 15
Hemoglobin 11.8 WI:IL
• 16 'I
Hematocrit 36%
• 17
Platelets 240,000.1p L
• 18
Leukocyte count 36,GGGip L
• 19
Neutrophils 90%
• 20
Lymphocytes 10%
• 21
Monocytes 2%
• 22
• 23
Nitroblue tetrazolium test is normal.
• 24
• 25
Which of the following defects is most likely present in this patient?
• 26
• 27
• 28 O A. Adenosine deaminase deficiency
• 29 O 13. Complement deficiency
• 30
0 C. Defective formation of mature 13 lymphocytes
• 31
• 32 O ID. Defective formation of mature T lymphocytes
• 33 • E Defective intracellular killing
• 34
• E Development defect of the fourth pharyngeal pouch
• 35
• 36 O G. Impaired leukocyte adhesion
• 37 • Fl. Opsonization defect
• 38
• 39
• 40
• 41
• 42 0
Feedback Suspend End Block
3 =1
1
4
Item: 15 of 44 V-• Mark
Previous Next Lab Values. Notes Calculator.

5
6 7 A 3D-minute-old boy is in the neonatal intensive care unit with central cyanosis_ He was born at 28 weeks
gestation by vaginal delivery to a 16-year-old girl. The pregnancy was complicated by premature labor, and
8 the mother received glucocorticoids and magnesium sulfate shortly after the onset of labor. Membranes
9 ruptured spontaneously 6 hours prior to delivery; amniotic fluid was clear. The boy initially emerged vigorous
• 10 with spontaneous respirations but rapidly developed respiratory distress_ Birth weight was 1 kg (2 lb 4 oz)_
• 11 His temperature is 36 C (96.8 F) under radiant heat, blood pressure is 65/35 mm Hg, pulse is 140imin, and
• 12 respirations are 70/min. Pulse oximetry showed 75% on room air and improves to 90% with 35% Fi02 by
sm
3
continuous positive air pressure ventilation. Physical examination shows grunting, intercostal and subcostal
• '1
retractions, nasal flaring, and shallow respirations. Bilateral breath sounds are diminished. A chest x-ray
shows diffuse, fine, reticular granularity and perihilar linear opacities bilaterally. What is the most likely
•14
• 6 I
diagnosis in this patient?
• -17
• 18
• 19 • A. Congenital diaphragmatic hernia
• 20
• ID-transposition of the great arteries
• 21
• 22 • C. Meconium aspiration syndrome
• 23 • ID. Persistent pulmonary hypertension
• 24
• 25 • E Respiratory distress syndrome
• 26 • Transient tachypnea of the newborn
• 27
• 28
• 29
• 30
• 31
• 32
• 33
▪ 34
• 35
▪ 36
• 37
▪ 38
• 39
• 40
• 41
• 42 0
Feedback Suspend End Block
3 =1
1
4
Item: 16 of 44 V-• Mark
Previous Next Lab Values. Notes Calculator.

5
6 7 An 18-month-old male is brought to the hospital because of fever, dyspnea, and productive cough of two days
7 duration. His mother reports that he just recovered from prolonged diarrhea due to Garda infection_ His past
8 medical history is also significant for pneumonia and recurrent ear infections since 6 months of age. On
9 physical examination, his temperature is 383' C (1G1 .1O F), pulse is 1.4Gimin, and respirations are 40/min.
• 10 Examination reveals a young child in mild respiratory distress and bronchial breath sounds in the right lower
• 11 lung lobe_ Which of the following is the most likely cause of his recurrent infections?
• 12
in
• 13
• 14
• A. Impaired oxidative metabolism
• 15 • B. Complement deficiency

• 17
▪ C. Thymic hypoplasia




13 I
19
▪ 11 Adenosine deaminase deficiency

• E Abnormal B-lymphocyte maturation


• 20
• 21
• 22
• 23
• 24
• 25
• 26
• 27
• 28
• 29
• 30
• 31
• 32
• 33
• 34
• 35
• 36
• 37
• 38
• 39
• 40
• 41
• 42 0
Feedback Suspend End Block
1
2 Item: 17 of 44 V- Mark
3 Previous Next Lab Values. Notes Calculator.
4-
5
6
An 8-year-old girl is being evaluated for short stature. She is at 8th percentile for height and 30th percentile for
7
weight Vital signs are within normal limits. Examination shows a high arched palate and inverted, widely
8
spaced nipples. Karyotyping shows 45 X0_ Which of the following is she most at risk of developing?
9
• 10
• 11 • A. Osteoporosis
• 12
• B. Mitral valve prolapse
• 13
• 14 • C. Mental retardation
• 15
• ID. Bipolar disorder

• E Breast cancer
• 18
• 19
• 20
• 21
• 22
• 23
• 24
• 25
• 26
• 27
• 28
• 29
• 30
• 31
• 32
• 33
• 34
• 35
• 36
• 37
• 38
• 39
• 40
• 41
• 42 0
Feedback Suspend End Block
1
2 Item: 18 of 44 V-• Mark
3 Previous Next Lab Values. Notes Calculator.
4
5
6 7 A 5-year-old boy is brought to the emergency department by his father with fatigue and scrotal swelling. His
7 father noticed the swelling today and is sure that "it wasn't there yesterday." The boy had a low-grade fever
8 1 and cough 2 weeks ago and "hasn't been himself ever since." He has no significant past medical history. On
9 examination, the patient looks ill and lethargic. Mucous membranes are dry and his capillary refill is 3
• 10 seconds. He has a palpable nonblanching rash on his buttocks and lower legs. The right hemi-scrotum is
• 11 slightly swollen and tender to palpation. Abdominal examination shows mild diffuse tenderness to palpation
• 12 without rebound or guarding_ Laboratory results from urinalysis are as follows:
1113
1=
Specific gravity 1.016
• 14
pH 7.0
• 15
Protein +1
• 16 'I
Blood Moderate
• 17
Glucose Negative
i
Ketones Negative
• 19
Leukocyte esterase Negative
•• 20
Nitrites Negative
• 21
• 22
• 23
The boy is admitted to the hospital for further management Which of the following adverse outcomes is he
• 24 most likely to develop?
• 25
• 26 • A. Appendicitis
• 27
• B. Cholecystitis
• 28
• 29 • C. Diverticulitis
• 30 • D. Intussusception
• 31
• 32
• E Meckel diverticulum
• 33 • F. Volvulus
• 34
• 35
• 36
• 37
• 38
• 39
• 40
• 41
• 42 0
Feedback Suspend End Block
• 1
• 2 Item: 19 of 44 V-• Mark
▪ 3 Previous Next Lab Values. Notes Calculator.
• 4-
▪ 5
▪ 6 A 3-year-old African American boy with sickle cell anemia is brought to the emergency department for fever.
▪ 7 He is exhausted and has chills and night sweats. However, he has had no bone pain, cough, or difficulty
▪ 8 breathing. The boy was hospitalized during infancy for dactylitis and last summer for splenic sequestration
▪ 9 crisis_ Other than ibuprofen and acetaminophen for fever and pain, he takes no medications_ His
• 10 vaccinations are up to date. His temperature is 3E4 C (103 F), blood pressure is 78/40 mm Hg, pulse is
• 11 140.imin, and respirations are 22/min_ Examination shows a lethargic boy with mild pallor. Laboratory results
• 12 are as follows:
• 13
Complete blood count
• 14
Hemoglobin 8.2 g/dL
• 15
Hematocrit 24%
• 16
Platelets 325,0001pL
• 17
Leukocytes 18,800IpL
• 18
Neutrophils 80%
Band 6%
• 20
Gram stain Pending
• 21
Blood culture Pending
• 22
• 23
• 24
Which of the following organisms is the most likely cause of this patient's condition?
• 25
• 26 a A. Escherichia col?.
• 27
O B. Haernophdus influerrzae type B
• 28
• 29 O C. Nesseria rnenwrgibares
• 30 O D. Pseudornonas aerugirrosa
• 31
O E Salmonella eritentocfis
• 32
• 33 O F. Staphylococcus aureus
• 34 O G. Streptococcus pneurnoniae
• 35
• 36
• 37
• 30
• 39
• 40
• 41
• 42 0
Feedback Suspend End Block
• 1
• 2 Item: 20 of 44 F' Mark
▪ 3 Previous Next Lab Values. Notes Calculator.
• 4
▪ 5
▪ 6 A previously healthy 5-year-old girl is brought to the emergency department with difficulty swallowing. For the
▪ 7 past week, she has had fever, chills, malaise, muscle aches, and sore throat She now refuses to drink fluids
▪ 8 and spits out any water that is given to her. The girl sustained a bite to her right forearm when she and her
▪ 9 family went camping in Delaware last month_ Family members are unsure of the bite source as it was dark.
• 10 The wound was rinsed with water and has since healed_ The girl takes no medications, has no allergies, and
• 11 is up to date on standard vaccinations_ There is no history of foreign travel_ Vital signs are normal.
• 12 Examination shows an agitated, disoriented girl with copious drool and facial grimacing. The neck has full
• 13 range of motion. A well-healed scar is seen on her right forearm_ Of the following disease reservoirs, which
• 14 is the most likely source of her symptoms?
• 15
• 16
• 17 • A. Mosquito
• 18 • B. Raccoon
• 19
• 20 • C. Rat
• 21 • ID. Spider
• 22
• 23 • E. Squirrel
• 24 • Tick
• 25
• 26
• 27
• 28
• 29
• 30
• 31
• 32
• 33
• 34
• 35
• 36
• 37
• 38
• 39
• 40
• 41
• 42 0
Feedback Suspend End Block
• 1
• 2 Item: 21 of 44 V-• Mark
▪ 3 Previous Next Lab Values. Notes Calculator.
• 4-
▪ 5
▪ 6 An 18-month-old child is brought to the emergency department by his mother with fever, vomiting, and
▪ 7 lethargy_ He developed a fever during the day that initially responded to treatment with
▪ 8 acetaminophen. Throughout the day, he became increasingly lethargic and developed a rash on his lower
▪ 9 extremities that has acutely worsened during the past few hours. The child's immunizations are up to date
• 10 and he is otherwise healthy. On examination he is drowsy and lethargic_ He flexes his hips when his neck is
• 11 flexed. He also has an erythematous, nonblanching pinpoint rash on his trunk and bilateral lower
• 12 extremities. What is the most likely organism causing this patient's symptoms?
• 13
• 14
• 15 • A. Borretia burgdorferi
• 16 • B. Cytomegaroviras
• 17
• 1a • C. Group B Streptococcus
• 19 • Haemophilus inffuenzae
• 20
I 21
• E Herpes simplex virus
• 22 • Listeria monocytogenes
• 23
• 24
• G. Neisseria meningitides
• 25
• 26
• 27
• 28
• 29
• 30
• 31
• 32
• 33
• 34
• 35
• 36
• 37
• 38
• 39
• 40
• 41
• 42 0
Feedback Suspend End Block
• 1
• 2 Item: 22 of 44 V-Mark
• 3 Previous Next Lab Values. Notes Calculator.
• 4-
• 5
• 6 A 2-month-old infant is brought to the clinic for the evaluation of poor feeding. He was born at 32 weeks of
• 7 gestation with a birth weight of 12GG g. The pertinent physical findings are pallor, tachypnea. tachycardia. and
• 8 flow murmurs. The laboratory studies are as follows:
• 9
Hb 7 WI:IL
• 10
WBC 7,DDD!cmm
• 11
Platelets 230,000/cl-rim
• 12
Reticulocytes Low
• 13
• 14
• 15
The peripheral smear shows normocytic normochromic RBC. What is the most likely diagnosis?
• 16 •
• 17

• 19 • A. Alpha thalassemia
• 20 • B. Beta thalassemia
• 21
• 22
• C. Hemolytic disease of newborn
• 23 • ID. Sickle cell anemia
- 24
• E Anemia of prematurity
- 25
• 26 Iron deficiency
• 27 • G. Vitamin B12 deficiency
• 28
• H. Pancytopenia
• 29
• 30 • G-6-PD deficiency
• 31
• 32
• 33
• 34
• 35
• 36
• 37
• 38
• 39
• 40
• 41
• 42 1
11 0
Feedback Suspend End Block
• 1
• 2 Item: 23 of 44 V- Mark
3 Previous Next Lab Values. Notes Calculator.
4
5
6 A 1G-week-old girl is brought to the office by her mother for a well-infant examination after missing her visit 2
7 weeks earlier. She was born at 34 weeks gestation and weighed 2_2 kg (4 lb 14 oz) at birth. She has been
breastfed exclusively since then, and her growth and development are appropriate for her chronological and
9 gestational ages. Her physical examination is notable for mild pallor but is otherwise normal. The mother
• 10 eats a balanced diet that includes meat products. At this time, which of the following is the most appropriate
• 11 nutritional intervention for this patient?
• 12
• 13
• 14
• A. Introduction of eggs and meat to the diet
• 15 • B. Introduction of fruit juices to the diet
• 16
• 17
C. Introduction of pureed vegetables to the diet
• 18 • 11 Iron and vitamin ID supplementation
• 19
• 20
• E No additional supplementation needed
2=
-1 • F. Vitamin 13 12 supplementation
• 22
• G. Vitamin C supplementation
• 24
• 25
• 26 I
• 27
• 28 I
• 29
• 30
• 31
• 32
• 33
• 34
• 35
• 36
• 37
• 38
• 39
• 40
• 41
• 42 0
Feedback Suspend End Block
1
2 Item: 24 of 44 .11 V-• Mark
3 Previous Next Lab Values. Notes Calculator.
4-
5
A previously healthy 8-year-old boy is brought to the office by his mother because he has multiple staring
6
episodes. During these episodes, he is unresponsive to verbal or tactile stimuli, and produces lip-smacking
7
movements. Each episode lasts for a few minutes, after which he remains confused for some time. He has
8
no family history of any seizure disorder. His neurological examination is unremarkable. EEG performed
9
before and after hyperventilation is normal_ Which of the following is the most likely diagnosis?
• 10
• 11 • A. Complex partial seizure
• 12
▪ R. Typical absence seizure
• 13
• 14 • C. Atypical absence seizure
• 15
▪ ID. Juvenile myoclonic epilepsy
• 16
• 17 • E. Lennox-Gastaut syndrome
• 18
• 19
• 20
• 21
• 22
4im

• 25
• 26
• 27
• 28
• 29
• 30
• 31
• 32
• 33
• 34
• 35
• 36
• 37
• 38
• 39
• 40
41
42 0
Feedback Suspend End Block
• 1
• 2 Item: 25 of 44 V- Mark
▪ 3 Previous Next Lab Values. Notes Calculator.
• 4-
▪ 5
▪ 6 A 15-year-old girl is brought to the physician by her mother because she has not begun menstruating_ She is
▪ 7 otherwise healthy and has no medical problems_ Vitals signs are within normal limits. Physical examination
▪ 8 shows absence of breast development and no pubic hair is seen. Examination shows no other
▪ 9 abnormalities. Ultrasound confirms the presence of a uterus. Which of the following is the most appropriate
• 10 next step in management?
• 11
• 12
• 13 O A. Estrogen level
• 14 • B. MRI of pituitary
• 15
• 16 O C. Serum FSH level
• 17 O ID. Kar-yotvping
• 18
• 19 • E GnRH stimulation test
• 20 O F. Reassurance
• 21
• 22
• 23
• 24

• 26
• 27
• 28
• 29
• 30
• 31
• 32
• 33
• 34
• 35
• 36
• 37
• 38
• 39
• 40
• 41
42 0
Feedback Suspend End Block
• 1
• 2 Item: 26 of 44 F' Mark
3 Previous Next Lab Values. Notes Calculator.
4
5
6 An 8-month-old boy is brought to the emergency department with fever, vomiting, and increased sleepiness_
7 He has had a fever for 3 days despite taking acetaminophen and ibuprofen_ Over the past day, he has
8 become increasingly sleepy with significantly decreased oral intake. His temperature is 40 C (104 F), blood
9 pressure is 92/48 mm Hg, pulse is 120/min, and respirations are 30/min_ Pulse oximetry shows 96% on room
• 10 air. The anterior fontanelle is soft and full. The infant is lethargic but cries when attempts are made to flex his
• 11 head. Pupils are equal, round, and reactive to light There are no focal neurologic deficits_ A complete blood
• 12 count and blood culture are pending. Which of the following are the most appropriate next steps in the
• 13 management of this patient?
• 14
• 15
• 16 • A. Head CT scan followed by lumbar puncture
• 17 • B. Intravenous antibiotics followed by head CT scan
• 18
• 19
• C. Intravenous antibiotics followed by hospital admission
• 20 • D. Intravenous antibiotics followed by lumbar puncture
• 21
3=
2.2
• E Lumbar puncture followed by intravenous antibiotics
• 23
• 24
• 25

•s
• 27
• 28 I
• 29
• 30
• 31
• 32
• 33
• 34
• 35
• 36
• 37
• 38
• 39
• 40
• 41
• 42 0
Feedback Suspend End Block
• 1
• 2 Item: 27 of 44 F Mark
3 Previous Next Lab Values. Notes Calculator.
4
5
6 A 16-year-old boy is brought to the pediatrician with pain and limited motion of the right knee. He first noticed
7 knee pain and swelling 6 months earlier, and the pain has gradually worsened since then. He has no history
1 of fever, local erythema, or trauma. The patient reports that his right knee and right ankle have swelled
9 several times before_ He has a history of easy bruising since childhood and had an episode of excessive
• 10 bleeding after a tooth extraction. His uncle had similar problems_ Which of the following is the most likely
• 11 cause of this patient's joint pain?
• 12
• 13
• 14
• A. Avascular necrosis
• 15 • B. Crystal deposition
• 16
• 17
C. Hemosiderin deposition and fibrosis
• 18 • 11 Immunologic tissue injury
• 19
• 20
• E Occult traumatic injury
• 21 • Repetitive microtrauma
22
ffmi
• 23
• G. Vector-borne illness
• 24
• 25
• 26

• 28
• 29
• 30
• 31
• 32
• 33
• 34
• 35
• 36
• 37
• 38
• 39
• 40
• 41
• 42 0
Feedback Suspend End Block
• 1
• 2 Item: 28 of 44 V- Mark
▪ 3 Previous Next Lab Values. Notes Calculator.
• 4-
▪ 5
▪ 6 A 4-day-old boy in the newborn nursery has failed a bilateral hearing screening_ He was born at 37 weeks
▪ 7 gestation by elective cesarean section for intrauterine growth retardation. His mother traveled to Egypt during
▪ 8 the first trimester of pregnancy and developed a brief febrile illness but otherwise felt well during the
▪ 9 pregnancy. The infant's weight, length, and head circumference are <5th percentile. Physical examination
• 10 shows bilateral white pupillary reflexes. The boy does not alert to voice or sound. Auscultation shows a
• 11 continuous systolic-diastolic murmur over the left 2nd intercostal space. The most likely cause of the
• 12 newborn's symptoms is infection with which of the following organisms?
• 13
• 14
• 15
• A. Cytomegalovirus
• 16 • B. Herpes simplex virus
• 17
• 18
• C. Rubella virus
• 19 • D. Toxopfasma gorrarii
• 20
• 21
• E Treponerna palfichlin
• 22 • Varicella zoster virus
• 23
• 24
• 25
- 25
- 27
m
28
• 29
• 30
• 31
• 32
33
34
• 35
• 36
• 37
• 38
• 39
• 40
• 41
42 0
Feedback Suspend End Block
• 1
• 2 Item: 29 of 44 V- Mark
▪ 3 Previous Next Lab Values. Notes Calculator.
• 4-
▪ 5
▪ 6 A 14-year-old boy comes to the physician with two days of fever and nasal discharge_ He has also had
▪ 7 malaise, fatigue, and myalgia_ He has no other medical problems_ Family history is negative for any kidney
▪ 8 disease. His temperature is 38.8° C (101.9° F), blood pressure is 122/74 mm Hg, pulse is 95/min, and
▪ 9 respirations are 15/min. Examination shows no abnormalities_ An incidental urine dipstick testing shows 2+
• 10 proteinuria but no hematuria, pyuria, or active urine sediment Serum creatinine is within normal limits_
• 11 Which of the following is the most appropriate next step in management?
• 12
• 13
• 14
• A_ Repeat dipstick testing on two subsequent occasions
• 15 • 13. Check serum protein and albumin levels
• 16
• 17
C. Order 24-hour urinary collection for protein
• 18 • 11 Order renal ultrasound
• 19
• 20
• E Reassure. with no further workup
• 21 • Renal biopsy
• 22
• 23
• 24
• 25
• 26
• 27
• 28

• 30
• 31
• 32
• 33
• 34
• 35
• 36
• 37
• 38
• 39
• 40
• 41
• 42 41. 0
Feedback Suspend End Block
• 1
• 2 Item: 30 of 44 A V- • Mark
▪ 3 Previous Next . Lab Values. Notes Calculator.
• 4-
▪ 5
▪ 6 A newborn girl is evaluated in the neonatal intensive care unit for cyanosis_ She was born at 36 weeks
▪ 7
gestation by cesarean section for fetal distress_ The girl remained hypoxic despite administration of
▪ 8 supplemental oxygen. On examination, she is cyanotic and tremulous. She has low-set ears, micrognathia,
▪ 9 and a cleft palate_ Chest radiograph shows an absent thymus_
• 10
• 11
• 12
• 13
• 14
• 15
• 16
• 17
• 18
• 19
• 20
• 21
• 22
• 23
• 24
• 25
• 26
- 27
- 28
▪ 29
• 30
31
32
33
34
35
36
37
38
39
• 40 se
41
42 0
Feedback Suspend End Block
• 1
• 2 Item: 30 of 44 'Mark
<=1

▪ 3 Previous Next Lab Values. Notes Calculator.


• 4
▪ 5
▪ 6
▪ 7
▪ 8
▪ 9
• 10
• 11
• 12
• 13
• 14
• 15
• 16
• 17
• 18
• 19
• 20
• 21
• 22
• 23
• 24
• 25
• 26
- 27
- 28
▪ 29 Echocardiography confirms truncus arteriosus. This patient is at greatest risk for which of the following?
• 30
• 31
▪ A. Duodenal atresia
• 32
▪ B. Hypocalcemia
33
• 34 • C. Hypoglycemia
• 35
• D. Infertility
• 36
• 37
• E. Myelomeningocele
▪ 38 ▪ Neutropenia
• 39
• G. Thrombocytopenia
• 40
• 41
• 42 0
Feedback Suspend End Block
• 1
• 2 Item: 31 of 44 F' Mark
3 Previous Next Lab Values. Notes Calculator.
4
5
6 A 52-hour-old girl in the newborn nursery was born at 40 weeks gestation to a 30-year-old woman by
7 cesarean delivery for failure to progress. Apgar scores were 8 and 9 at 1 and 5 minutes, respectively. The
8 patient has been voiding appropriately but has not yet passed meconium. Today she has been spitting up all
9 feeds and now has bilious emesis_ Her abdomen is tense and distended. Rectal examination shows no stool
• 10 in the rectal vault Feeds are held and a nasogastric tube is placed for decompression. Upright abdominal
• 11 x-ray shows multiple dilated loops of small bowel with paucity of air in the large intestine and rectum_ Left
• 12 lateral decubitus view shows free air above the liver. An emergency laparotomy is performed. Viscous
• 13 meconium is irrigated and evacuated, primarily from the ileum with some from the colon. The colon is
• 14 diffusely narrow. Which comorbidity will this patient most likely develop?
• 15
• 16
• 17 • A. Chronic rhinosinusitis
• 18
• B. Early-onset Alzheimer's disease
• 19
• 20 • C. Hypothyroidism
• 21
• D. Infertility from cervical obstruction
• 22
• 23 • E Sensorineural hearing loss
• 24
• 25
• 26
• 27
• 28
29
• 30

• 32
• 33
• 34
• 35
• 36
• 37
• 38
• 39
• 40
• 41
• 42 0
Feedback Suspend End Block
• 1
• 2 Item: 32 of 44 F' Mark
3 Previous Next Lab Values. Notes Calculator.
4
5
6 A newborn boy is evaluated shortly after vaginal delivery for respiratory distress. He was born at 37 weeks
7 gestation to a primigravid mother who had no medical problems. Second-trimester prenatal ultrasound
8 showed bilateral hydronephrosis and oligohydramnios. His temperature is 36.7 C (98 F), pulse is 144/min,
9 and respirations are 50/min. Pulse oximetry is 84% on room air. Examination shows decreased aeration of
• 10 both lungs and intercostal retractions. The child has flattened facies, abdominal distension, and clubfeet
• 11 What is the most likely underlying diagnosis in this patient?
• 12
• 13
• A. Bilateral cryptorchidism
• 14
• 15 • B. Congenital diaphragmatic hernia
• 16
• 17 C. Duodenal atresia
• 18 • D. Hypospadias
• 19
• 20 • E Posterior urethral valves
• 21 • Prostatic hyperplasia
• 22
• 23 • G. Wilms tumor
• 24
• 25
• 26
• 27
• 28
• 29
H
• 31
16
• 31

• 33
• 34
• 35
• 36
• 37
• 38
• 39
• 40
• 41
• 42 0
Feedback Suspend End Block
1
• 2 Item: 33 of 44 .11 V- • Mark
3 Previous Next Lab Values. Notes Calculator.
• 4-
5
6 A 13-year-old girl is brought to the physician for evaluation of sunburn. She just returned from a week-long
7 vacation to the Bahamas with her family. For the past 2 days, the patient has had redness on her face and
8 shoulders despite applying sun protection factor 50 sunscreen just before swimming. This is her first
9 sunburn. Her parents say that "everybody tans easily" in their family. Family history is negative for skin
• 10 cancer. Physical examination shows a well-appearing adolescent with olive skin, brown eyes, and light-brown
• 11 hair. Tender, blanching erythema is present on the face and arms. There is no edema or vesicles. Which of
• 12 the following is the most appropriate recommendation?
• 13
• 14
• 15
• A. Apply sun protection factor 50 sunscreen 30 minutes before sun exposure
• 16 • B. Apply topical corticosteroids to inflamed skin
• 17
• 18
• C. Change to sun protection factor 75 sunscreen
• 19 • Continue current sun protection regimen
• 20
• 21
• E Form a baseline tan with a tanning bed before sun exposure
• 22 • F. Refrain from outdoor activities on cloudless days
• 23
• 24
• G. Wear light-colored clothing while outdoors
• 25
• 26
• 27
• 28
• 29
• 30
• 31
• 32

• 34
• 35
• 36
• 37
• 38
• 39
• 40
• 41
• 42 0
Feedback Suspend End Block
1
• 2 Item: 34 of 44 F' Mark
3 Previous Next Lab Values. Notes Calculator.
• 4-
5
6 A 10-year-old patient with sickle cell disease comes to the physician for a routine visit The patient has a
7 history of multiple pain crises and pneumonias. He was started on hydroxyurea one year ago and has not had
8 any further pain crises since then. He has had no recent illnesses or hospitalizations. His physical
9 examination is unremarkable_ The patient's laboratory results are shown below_
• 10
Complete blood count
• 11
Hemoglobin 9.0 gIL
• 12
Erythrocyte count 2_2 mlnimm
• 13
• 14
MCHC 32%
• 15
MCV 105 fl
• 16
Reticulocytes 2_0%
• 17 Platelets 212,0001mm3
• 18 Leukocyte count 9500/mm3
• 19 Neutrophils 56%
• 20 Eosinophils 3%
• 21 Lymphocytes 36%
• 22 Monocytes 5%
• 23
• 24 Which of the following best describes the role of hydroxyurea in the treatment of patients with sickle cell
• 25 disease?
• 26
• 27
• A. Removes sickled red blood cells from the circulation
• 28
• 29 • B. Lyses microthrombi in the circulation
• 30 • C. Protects against encapsulated bacterial infections
• 31
• D. Increases fetal hemoglobin
• 32
• 33 • E. Chelates iron to prevent iron toxicity

• 35
• 36
• 37
• 38
• 39
• 40
• 41
• 42 0
Feedback Suspend End Block
• 1
• 2 Item: 35 of 44 V- Mark
▪ 3 Previous Next Lab Values. Notes Calculator.
• 4-
▪ 5
▪ 6 A 17-year-old girl is admitted to the hospital for purging behavior, weight loss, and syncope. She is started on
▪ 7 nasogastric feeds and her activities are restricted. On the second day of admission, the patient complains of
▪ 8 severe shortness of breath and has recurrent episodes of nonsustained ventricular tachycardia. Her blood
▪ 9 pressure is 82/55 mm Hg, heart rate is 11 /min, and respirations are 22/min. Her body mass index is 14
• 10 kg/m2. Physical examination shows an emaciated girl with bibasilar crackles and jugular venous distension_
• 11 Serum chemistry results are as follows:
• 12
Sodium 138 mEq/L
• 13
Potassium 2_1 mEq/L
• 14
Chloride 92 mEq!L
• 15
Bicarbonate 28 mEq!L
• 16
Blood urea nitrogen 14 mg/dL
• 17
Creatinine 0_8 mg/dL
• 18
Calcium 8 mg/dL
• 19
Glucose 90 mg/dL
• 20
Magnesium 1 mg/dL
• 21
Phosphorus 0_9 mg/dL
• 22
• 23
• 24
A surge in which of the following hormones best explains this patient's deterioration?
• 25
• 26 • A. Aldosterone
• 27
• B. Cortisol
• 28
• 29 • C. Glucagon
• 30 • D. Insulin
• 31
• E Triiodothyronine
• 32
• 33
• 34

• 36
• 37
• 38
• 39
• 40
• 41
• 42 0
Feedback Suspend End Block
• 1
• 2 Item: 36 of 44 F' Mark U.P
▪ 3 Previous Next Lab Values, Notes Calculator.
• 4-
▪ 5
▪ 6 A 4-year-old boy is brought to physician for evaluation of a rash. A few days ago, the boy had a few small red
▪ 7 bumps on his chin and cheek. The bumps became painful as they filled with fluid. The patient has no other
▪ 8 medical problems and his vaccinations are up to date. He has been playing at the park daily and has had
▪ 9 occasional insect bites_ Vital signs are normal. Physical examination shows multiple pustules on his right
• 10 cheek, nose, and chin. Some lesions are covered with a thick golden-yellow crust The rest of his
• 11 examination is within normal limits_ What is the most appropriate next step in the management of this patient?
• 12
• 13
• 14 • A. Anti-streptolysin 0 antibody titers
• 15 • B. Express pus for culture
• 16
• 17 C. Oral cephalexin
• 18 ID. Swab intact skin for culture

• 19
• 20 • E Topical mupirocin
• 21
• 22
• 23
• 24
• 25
• 26
• 27
• 28
• 29
• 30
• 31
• 32
• 33
• 34
35
I 36
• 37
• 38
• 39
• 40
• 41
• 42 0
Feedback Suspend End Block
• 1
• 2 Item: 37 of 44 .11M Mark
▪ 3 Previous Next Lab Values. Notes Calculator.
• 4-
▪ 5
▪ 6 An 11-month-old girl is brought to the physician for fever. For the past few days, she has had rhinorrhea and
▪ 7 nasal congestion. The fever started 2 days ago and the girl has since been pulling at both ears. The ear pain
▪ 8 seems worse when she lies down. The girl has no medical problems and takes no medications. She attends
▪ 9 day care and her diet consists of infant formula and finger foods. Both parents smoke cigarettes. Her
• 10 temperature is 38.9 C (102 F). Otoscopy shows bulging, erythematous bilateral tympanic membranes with
• 11 decreased mobility on air compression. Crusted rhinorrhea is present at the nares. Her hearing is intact and
• 12 the rest of her examination is normal_ Which of the following is the most appropriate next step in
• 13 management of this patient?
• 14
• 15
• 16 • A. Acetaminophen, follow-up in 2 days
• 17 • B. Myringotomy with tympanostomy tube placement
• 18
• 19
• C. Oral antibiotics
• 20 • ID. Ototopical antibiotics
• 21
• 22
• E. Tympanocentesis and culture
• 23 C. F. Viral nasopharyngeal polymerise chain reaction testing
• 24
• 25
• 26
• 27
• 28
• 29
• 30
• 31
• 32
• 33
• 34
• 35
• 36
a 37
• 38
• 39
• 40
• 41
• 42 0
Feedback Suspend End Block
• 1
• 2 Item: 38 of 44 V-• Mark
▪ 3 Previous Next Lab Values. Notes Calculator.
• 4-
▪ 5
▪ 6 A 12-year-old boy is brought to the emergency department because of severe pain near his left knee. He has
▪ 7 sickle cell disease and has been hospitalized previously for sickle cell crisis_ Vital signs are notable for
▪ 8 persistent fever. Examination of the left lower extremity reveals a normal knee joint with marked tenderness
▪ 9 and swelling over the proximal tibia. Laboratory studies show leukocytosis and elevated ESR_ Imaging
• 10 studies confirm the diagnosis of osteomyelitis. Which of the following organisms is the most likely cause of
• 11 his condition?
• 12
• 13
• 14
• A. Escherichia cob.
• 15 • B. Pseudomorras species
• 16
• 17
▪ C. Salmonella species
• 18 ▪ D. Streptocomis pneumoniae
• 19
• 20
• E Group B streptococcus
• 21
• 22
• 23
• 24
• 25
• 26
• 27
• 28
• 29
• 30
• 31
• 32
• 33
• 34
▪ 35
▪ 36
- 37
• 38
- 39
- 40
▪ 41
42 0
Feedback Suspend End Block
• 1 I Item: 39 of 44
• 2 F' Mark
▪ 3 Previous Next Lab Values. Notes Calculator.
• 4-
▪ 5
▪ 6 A 4-year-old girl is brought to the physician for a rash. She lives in Poland and arrived in the United States last
▪ 7 week to visit her family. The child was in good health until 3 days ago, when she developed nasal congestion
▪ 8 and a low-grade fever. Today she developed a "pink rash" on her face that spread rapidly to her trunk and
▪ 9 extremities. Review of systems is otherwise negative. She completed an antibiotic course for a "throat
• 10 infection" 2 months ago but has otherwise been healthy. She has received no vaccinations and takes no
• 11 medications. Her temperature is 38.3 C(101 F), blood pressure is 1GG/50 mm Hg, pulse is 88/min, and
• 12 respirations are 181min. Physical examination shows a cooperative girl with bilateral nonexudative
• 13 conjunctivitis and patchy erythema on the soft palate. Small, tender lymph nodes are palpable in the
• 14 suboccipital, posterior auricular, and posterior cervical areas_ The nontender rash is shown below_
• 15
• 16
• 17
• 18
• 19
• 20
• 21
• 22
• 23
• 24
• 25
• 26
• 27
• 28
• 29
• 30
• 31
• 32
• 33
• 34

se

11. 0
I Feedback, Suspend End Block
1
• 2 Item: 39 of 44 V/lark
3 Previous Next Lab Values. Notes Calculator.
• 4- respirations are 181min_ Physical examination shows a cooperative girl with bilateral nonexudative
5 conjunctivitis and patchy erythema on the soft palate. Small, tender lymph nodes are palpable in the
6 suboccipital, posterior auricular, and posterior cervical areas_ The nontender rash is shown below.
▪ 7
▪ 8
▪ 9
• 10
• 11
• 12
• 13
• 14
• 15
• 16
• 17
• 18
• 19
• 20
• 21
• 22
• 23
• 24
• 25
• 26
• 27
• 28
• 29
• 30
• 31
• 32
• 33
• 34
35
36
37
38

se

0
Feedback Suspend End Block
• 1
• 2 Item: 39 of 44 V- Mark U.P
▪ 3 Previous Next Lab Values, Notes Calculator.
• 4-
▪ 5
▪ 6
▪ 7
▪ 8
▪ 9
• 10
• 11
• 12
• 13
• 14
• 15
• 16
• 17
• 18
• 19
• 20
• 21
• 22
• 23
• 24
• 25
• 26
• 27
• 28
• 29
• 30
• 31
• 32
• 33
• 34

The remainder of the examination is normal. ..1..rhich of the following is the most likely cause of this patient's
condition?

111 0
Suspend End Block
Feedback
• 1
Item: 39 of 44 I -4(1 I>- El70
• 2 Mark
'

Previous Next Lab Values Notes Calculator.


▪ 3
. 4
▪ 5
6
7
8
▪ 9
• 10
• 11
• 12
• 13
• 14
• 15
• 16
• 17
• 18
• 19
• 20
• 21
• 22
• 23
• 24
• 25
• 26
The remainder of the examination is normal Which of the following is the most likely cause of this patient's
• 27
condition?
• 28
• 29
• 30 0 A. Erythema multiforme
• 31 • B. Exanthematous drug eruption
. 32
O C. Kawasaki disease
• 33
• 34 0 D. Measles
• 35 O E. Mumps
• 36
• F. Rubella
• 37
• 3-8 O G. Scarlet fever
• 39
O H. Varicella
• 40
• 41
0
• 42 Feedback Suspend End Block
A'7
• 1 I Item: 40 of 44
• 2 F' Mark
▪ 3 Previous Next Lab Values. Notes Calculator.
• 4-
▪ 5
▪ 6 A 1-year-old girl is brought to the emergency department because she is not moving her right arm. Her father
▪ 7 says that the girl acted normally when she awoke this morning. He was swinging her by the forearms when
▪ 8 she suddenly started crying and refused to use her right arm. He has swung her by the arms previously and it
▪ 9 usually makes her laugh. The girl lives with her father and her mother, who is currently out of town_ Physical
• 10 examination shows a well-nourished child who holds her right arm in pronation against the chest She avoids
• 11 any movement of her right arm. No bruises or other abnormalities are seen_ Which of the following is the
• 12 most appropriate next step in management of this patient?
• 13
• 14
• 15 • A. Contact the child protection agency
• 16
• 17
B. Closed reduction and casting of the right arm
• 18 • C. Gently hyperpronate the right forearm
• 19
• 20
• D. Order a complete skeletal survey
• 21 • E Order an x-ray of the right arm
• 22
• 23
• Refer the child to an orthopedic surgeon
• 24
• 25
• 26
• 27
• 28
• 29
• 30
• 31
• 32
• 33
• 34
• 35
• 36
• 37
• 38
• 39

• 41
42 0
Feedback Suspend End Block
A'S
1
• 2 Item: 41 of 44 RI M Mark
3 Previous Next Lab Values. Notes Calculator.
• 4-
5
6 A 9-month-old infant is brought to the emergency department with lethargy and tachypnea_ He was healthy
7 before developing fever and diarrhea four days ago. He has been taking some formula, but has had two to
8 three episodes of diarrhea with each bottle. He has lost three pounds (1.4 kg) since his routine check-up two
9 weeks ago. He has had one wet diaper in the past twenty four hours. On examination, his temperature is
• 10 102.5° F (39.1' C), pulse is 200/min, respiratory rate is 42/min, and blood pressure is 70/45 mm Hg_ He is
• 11 lethargic with decreased tone and decreased deep tendon reflexes. His mucous membranes are dry.
• 12 Cardiopulmonary exam reveals tachycardia and tachypnea. His abdominal exam is unremarkable. Capillary
• 13 refill is four seconds. Laboratory results are shown below.
• 14
Chemistry panel
• 15
Serum sodium 165 mEq/L
• 16
Serum potassium 4.5 mEq!L
• 17
Chloride 1 08 mEq/L
• 18
Bicarbonate 14 mEq/L
• 19
Blood urea nitrogen (BUN) 20 mgldL
• 20
Serum creatinine 0.8 mg/I:IL
• 21
Calcium 10.0 mg/dL
• 22
Blood glucose 98 mg/dL
• 23
• 24
• 25
Which of the following fluids should be used as a bolus in the resuscitation of this infant?
• 26
• 27 • A. 0.9% saline
• 28
• B. 0.45% saline
• 29
• 30 • C. 5% dextrose
• 31 • ID. 5% albumin
• 32
• E Packed red blood cells
• 33
• 34
• 35
• 36
• 37
• 38
• 39
• 40

42 0
Feedback Suspend End Block
• 1
• 2 Item: 42 of 44 A V- Mark
▪ 3 Previous Next Lab Values. Notes Calculator.
• 4-
▪ 5
▪ 6 A newborn infant is brought to the nursery for evaluation after delivery. The mother reports that the pregnancy
▪ 7 was uncomplicated, but she had only two prenatal visits. The infant was born via normal spontaneous vaginal
▪ 8 delivery and required no resuscitation. The infant has hepatosplenomegaly on examination. While in the
▪ 9 hospital, the infant requires treatment for anemia and hyperbilirubinemia. On subsequent examinations, the
• 10 infant has clear rhinorrhea and ulcerative lesions on his feet Which of the following congenital infections is
• 11 most likely in this patient?
• 12
• 13
• 14 • A. Toxoplasmosis
• 15 • E. Syphilis
• 16
• 17 • C. Rubella
• 18 • D. Cytomegalovirus infection
• 19
• 20 • E Human immunodeficiency virus infection
• 21
• 22
• 23
• 24
• 25
• 26
• 27
• 28
• 29
• 30
• 31
• 32
• 33
• 34
• 35
• 36
• 37
• 38
• 39
• 40
• 41
0
Feedback Suspend End Block
• 1
• 2 Item: 43 of 44 F' Mark
3 Previous Next Lab Values. Notes Calculator.
• 4
▪ 5
▪ 6 A 15-year-old Caucasian male is brought to the office by his mother for the evaluation of a six-month history of
▪ 7 unstable gait and speech difficulty which are getting worse over time_ His past medical history is insignificant
▪ 8 He is not taking any medications, and denies smoking or alcohol consumption. His blood pressure is 120/10
▪ 9 mmHg and pulse is 80/min. Musculoskeletal examination showed scoliosis and feet deformity with 'hammer
• 10 toes.' The neurologic examination showed dysarthria, dysmetria, nystagmus, and absence of deep plantar
• 11 reflexes on lower extremities. What is the most common cause of death in this patient population?
• 12
• 13
• 14
• A. Cardiomyopathy
• 15 • B. Renal failure
• 16
• 17
▪ C. Diabetes-related complications
• 18 ▪ D. Malignancy
• 19
• 20
• E Septic shock
• 21
• 22
• 23
• 24
• 25
• 26
• 27
• 28
• 29
• 30
• 31
• 32
• 33
• 34
• 35
• 36
• 37
• 38
• 39
• 40
• 41
•42 41. 0
Feedback. Suspend End Block
• 1
• 2 Item: 44 of 44 F Mark
▪ 3 Previous Next Lab Values. Notes Calculator.
• 4-
▪ 5
▪ 6 A previously healthy 6-year-old boy is brought to the physician after the sudden appearance of bruises
▪ 7 throughout his body. He has had no bleeding or recent trauma Three weeks ago, he had an upper
▪ 8 respiratory tract infection that resolved uneventfully. He takes no medications, has no allergies, and his
▪ 9 immunizations are up to date_ Family history is negative for bleeding or clotting disorders. Vital signs are
• 10 normal. Physical examination shows a cooperative, well-appearing child with scattered petechiae and
• 11 ecchymoses over the trunk and extremities. The rest of the examination is unremarkable. Laboratory results
• 12 are as follows:
• 13
Hemoglobin 13.5gidL
• 14
Platelets 1 0, CEDDip L
• 15
Leukocytes 1,000.1pL
• 16
• 17
• 18
Peripheral smear shows a few large platelets. What is the most appropriate next step in management of this
• 19
patient?
• 20
• 21 • A. Bone marrow biopsy
• 22
• B. Intravenous antibiotics
• 23
• 24 • C. Intravenous immunoglobulin
• 25
• ID. Observation
• 26
• 27 • E Platelet transfusion
• 28 • F. Splenectomy
• 29
• 30
• 31
• 32
• 33
• 34
• 35
• 36
• 37
• 38
• 39
• 40
• 41
• 42 0
Feedback Suspend End Block
• 2 Item: 1 of 44 V-• Mark
• 3 Previous Next Lab Values. Notes Calculator.
• 4
• 5
• 6 A 16-year-old girl comes to the physician for a routine examination. She is wearing a sweater despite the
• 7 warm weather and says that she often feels cold. She also complains of difficulty with weight loss_ The
• 8 patient wants to lose about 4.5 kg (10 Ib) because she believes she is "enormous." For the past year, she
• 9 has been on a vegetarian diet because meat makes her feel bloated. Menarche was at age 12 and her last
• 10 menstrual period was 2 months ago. The patient is an honors student and involved in ballet and
• 11 cross-country running. Her mother has Graves' disease. Temperature is 36_2 C (97.2 F), blood pressure is
• 12 88/58 mm Hg, pulse is 42/min, and respirations are 16/min. Her body mass index is 15 kg/m2. The patient is
• 13 alert and cooperative but appears tired. Her skin is dry with lanugo. Laboratory results are as follows:
• 14
Serum chemistry
• 15
Sodium 136 mEdiL
• 16
Potassium 2.9 mEq/L
• 17
Chloride 102 mEq/L
• 18
Bicarbonate 24 mEq/L
• 19
Blood urea nitrogen 18 mg/dL
• 20
Creatinine mg/dL
• 21
Calcium 8 mg/dL
• 22
Glucose 78 mg/dL
• 23
Magnesium 2.3 mg/dL
• 24
Phosphorus 2 mg/dL
• 25
• 26
Thyroid function tests
• 27
Thyroid-stimulating hormone 4.5 pU/mL
• 28
Free T4 ng/dL
• 29
Triiodothyronine (T3) 65 ngfdL
• 30
• 31
Urine human chorionic gonadotropin Negative
• 32
• 33
• 34
What is the most appropriate next step in management of this patient's symptoms?
• 35
• 36 A. Administer bupropion [1%]
• 37
B. Administer fluoxetine [59C]
• 38
• 39 C. Administer levothyroxine [14%]
• 40 11 Ariminktpr nlAn7Anine 11041
• 41
• 42 End Block
Feedback
• 2 Item: 1 of 44 V-Mark
-4(1

• 3 Previous Next Lab Values, Notes Calculator.


141L1J114--J14-11 I I IIILJL
• 4
Phosphorus 2 mg/dL
▪ 5
▪ 6
Thyroid function tests
▪ 7
Thyroid-stimulating hormone 4.5 pLlimL
▪ 8
Free T4 0.4 ngidL
▪ 9
Triiodothyronine (T3) 65 ngldL
• 10
• 11
Urine human chorionic gonadotropin Negative
• 12
• 13
• 14
What is the most appropriate next step in management of this patient's symptoms?
• 15
• 16 A. Administer bupropion [1%]
• 17
B. Administer fluoxetine [5%]
• 18
• 19 C. Administer levothyroxine [141C]
• 20 ID. Administer olanzapine [1%]
• 21
E Hospitalization [67%]
• 22
• 23 F. Order serum 25-hydroxy-vitamin D [1'3C]
• 24 G. Order serum cobalamin [19C]
• 25
H.Refer for cognitive behavioral therapy, with follow-up in 2 weeks [10%]
• 26
• 27
• 28 Explanation:
• 29
• 30
• 31 Anorexia versus bulimia nervosa
• 32 . -
• 33 Diagnosis Clinical features Treatment
• 34
• 35 • Cognitive-behavioral
• 36 • BMI <18. 5
therapy
• 37 Anorexia • Intense fear of weight gain
• 38 • Nutritional rehabilitation
nervosa
• 39 • Distorted views of body
• Olanzapine, if no response se
• 40 weight and shape


41
42
Feedback
a
End Block
: 1 of 44 E F'Mark
▪ 3 Previous , Next Lab Values. Notes Calculator.
• 4- 1 • ulanzapine, 11 no iesponse
weight and shape
▪ 5 to above
▪ 6
▪ 7
• Recurrent episodes of
▪ 8
▪ 9 binge eating
• 10 • Binge eating followed by • Cognitive-behavioral
• 11 compensatory behavior therapy
• 12 Bulimia to prevent weight gain • Nutritional rehabilitation
• 13 nervosa
• 14 • Excess worrying about • SSRI medication, often in
• 15 body shape and weight combination with above
• 16
• Maintains normal body
• 17
weight (BMI 18_5-30)
• 18
• 19 Li5MLEwotldr LLC
• 20
• 21 Anorexia nervosa is an eating disorder that usually begins in adolescence. It is characterized by distorted
• 22 body image, determined dieting, and phobic avoidance of many foods, resulting in unsafe weight loss. Risk
• 23 factors include high-achieving "type-A" personalities and participation in activities in which there is pressure to
• 24 be slender (eg, ballet, running). To lose weight, patients may fast and/or exercise excessively (restricting
• 25 subtype) or compensate for binge eating with laxatives or vomiting (purging subtype). Many patients also have
• 26 secondary amenorrhea. This patient meets the DSM-V diagnostic criteria for anorexia nervosa (Table).
• 27
• 28 Nutritional rehabilitation and psychotherapy are the cornerstones of treatment for anorexia nervosa, with
• 29 the primary goal of weight gain_ Hospitalization and acute stabilization are highly recommended for
• 30 dehydration, electrolyte disturbances (eg, hypokalemia, hypophosphatemia), bradycardia, or severe weight
• 31 loss, as seen in this patient. Meals should be supervised, and some patients require nasogastric tube
• 32 feeding. During the onset of anabolism, patients require close monitoring for refeeding syndrome:
• 33 electrolyte depletion, arrhythmias, and heart failure, which can result from fluid and electrolyte shifts_ Vitamin
• 34 deficiencies should be assessed and supplemented if deficiencies are identified (Choices F and G).
• 35
• 36 (Choice A) Bupropion is contraindicated in patients with eating disorders as it can provoke seizures.
• 37
• 38 (Choice B) Clear evidence supports the use of selective serotonin reuptake inhibitors (SSR1s) to achieve
• 39 remission from binging and purging in bulimia nervosa However, SSRIs have not been shown to aid weight
• 40 gain in anorexia nervosa and are therefore not recommended.
• 41
• 42 End Block
Feedback
Item: 1 of 44 F'Mark
▪ 3 Previous Next Lab Values. Notes Calculator.
• 4- be slender (eg, ba-llet, running). To lose weight, patients may fast and/or exercise excessively (restricting
▪ 5 subtype) or compensate for binge eating with laxatives or vomiting (purging subtype). Many patients also have
▪ 6 secondary amenorrhea. This patient meets the DSM-V diagnostic criteria for anorexia nervosa (Table).
▪ 7
▪ 8 Nutritional rehabilitation and psychotherapy are the cornerstones of treatment for anorexia nervosa, with
▪ 9 the primary goal of weight gain_ Hospitalization and acute stabilization are highly recommended for
• 10 dehydration, electrolyte disturbances (eg, hypokalemia, hypophosphatemia), bradycardia, or severe weight
• 11 loss, as seen in this patient Meals should be supervised, and some patients require nasogastric tube
• 12 feeding. During the onset of anabolism, patients require close monitoring for refeeding syndrome:
• 13 electrolyte depletion, arrhythmias, and heart failure, which can result from fluid and electrolyte shifts_ Vitamin
• 14 deficiencies should be assessed and supplemented if deficiencies are identified (Choices F and G).
• 15
• 16 (Choice A) Bupropion is contraindicated in patients with eating disorders as it can provoke seizures.
• 17
• 18 (Choice B) Clear evidence supports the use of selective serotonin reuptake inhibitors (SSR1s) to achieve
• 19 remission from binging and purging in bulimia nervosa However. SSRIs have not been shown to aid weight
• 20 gain in anorexia nervosa and are therefore not recommended.
• 21
(Choice C) Starvation and malnutrition can decrease serum thyroxine-binding globin and albumin
• 22
concentrations in anorexia nervosa, resulting in euthyroid hypothyroxinemia. Levothyroxine is not
• 23
recommended and treatment should be directed at nutritional rehabilitation instead.
• 24
• 25
(Choice ID) Olanzapine is the most widely studied antipsychotic for anorexia nervosa and can be considered
• 26
in patients who fail to gain weight despite initial nutritional rehabilitation and cognitive behavioral therapy.
• 27
• 28 (Choice H) Cognitive behavioral therapy is very important but this patient requires prompt comprehensive
• 29 inpatient management, rather than outpatient referral.
• 30
• 31 Educational objective:
• 32 Patients with anorexia nervosa require hospitalization when they have unstable vital signs, severe bradycardia
• 33 or cardiac dysrhythmias, and electrolyte derangements. Goals of treatment include nutritional rehabilitation
• 34 and weight gain. Patients should be monitored closely for refeeding syndrome.
• 35
• 36 References:
• 37
• 38 1. Identification and management of eating disorders in children and adolescents.
• 39 2. Refeeding hypophosphatemia in adolescents with anorexia nervosa: a systematic review.
• 40
• 41
• 42 End Block
Feedback
• 2 Item: 1 of 44 F' Mark
• 3 Previous Next Lab Values. Notes Calculator.
• 4
▪ 5 Nutritional rehabilitation and psychotherapy are the cornerstones of treatment for anorexia nervosa, with
▪ 6 the primary goal of weight gain_ Hospitalization and acute stabilization are highly recommended for
▪ 7 dehydration, electrolyte disturbances (eg, hypokalemia, hypophosphatemia), bradycardia, or severe weight
▪ 8 loss, as seen in this patient Meals should be supervised, and some patients require nasogastric tube
▪ 9 feeding. During the onset of anabolism, patients require close monitoring for refeeding syndrome:
• 10 electrolyte depletion, arrhythmias, and heart failure, which can result from fluid and electrolyte shifts. Vitamin
• 11 deficiencies should be assessed and supplemented if deficiencies are identified (Choices F and G).
• 12
• 13 (Choice A) Rupropion is contraindicated in patients with eating disorders as it can provoke seizures.
• 14
• 15 (Choice B) Clear evidence supports the use of selective serotonin reuptake inhibitors (SSR1s) to achieve
• 16 remission from binging and purging in bulimia nervosa. However, SSRIs have not been shown to aid weight
• 17 gain in anorexia nervosa and are therefore not recommended.
• 18
• 19
(Choice C) Starvation and malnutrition can decrease serum thyroxine-binding globin and albumin
• 20
concentrations in anorexia nervosa, resulting in euthyroid hypothyroxinemia. Levothyroxine is not
• 21
recommended and treatment should be directed at nutritional rehabilitation instead.
• 22
(Choice ID) Olanzapine is the most widely studied antipsychotic for anorexia nervosa and can be considered
• 23
in patients who fail to gain weight despite initial nutritional rehabilitation and cognitive behavioral therapy.
• 24
• 25
(Choice H) Cognitive behavioral therapy is very important but this patient requires prompt comprehensive
• 26
inpatient management, rather than outpatient referral.
• 27
• 28 Educational objective:
• 29 Pi
Patients with anorexia nervosa require hospitalization when they have unstable vital signs, severe bradycardia
• 30 or cardiac dysrhythmias, and electrolyte derangements. Goals of treatment include nutritional rehabilitation
• 31 and weight gain. Patients should be monitored closely for refeeding syndrome.
• 32
• 33
References:
• 34
• 35 1. Identification and management of eating disorders in children and adolescents.
• 36 2. Refeeding hypophosphatemia in adolescents with anorexia nervosa: a systematic review.
• 37
• 38
• 39
Copyright © Morld Last updated: [10/13/2014]
• 40
• 41
• 42 End Block
Feedback
2 Item: 2 of 44 F' Mark
3 Previous Next Lab Values. Notes Calculator.
• 4 1.
5
6 A 4-year-old boy is brought to the physician for evaluation of speech that is "difficult to understand." His
▪ 7 parents state that "he doesn't seem to listen" and describe him as a slow learner who only uses single words
▪ 8 and frequently mumbles. The boy prefers to play by himself and is obsessed with building towers with
▪ 9 blocks_ He becomes extremely distressed if prevented from performing this activity. He was born at term
• 10 from an uncomplicated pregnancy and delivery, and his physical growth has been normal. His older brother
• 11 was aggressive as a young child and diagnosed with attention deficit hyperactivity disorder. In the office
• 12 waiting room, the boy plays quietly with blocks and seems oblivious to the other children. When his mother
• 13 calls him to come to her, he turns toward her once and then resumes playing. His physical examination and
• 14 hearing are normal. Which of the following is the most appropriate statement to tell his parents?
• 15
• 16
• 17
A. "He most likely has a developmental language disorder and would benefit from speech therapy."
• 18
[2%]
• 19 H. "His behavior is most likely due to the inattentive subtype of attention deficit hyperactivity disorder."
• 20 [1%]
• 21
C. "I am concerned that his difficulties may indicate intellectual disability." [1%]
• 22
• 23
ID_ understand your concerns; a brain MRI will help establish a definitive diagnosis." [0%]
• 24 E. understand your concerns; a comprehensive evaluation for autism spectrum disorder will be
• 25 helpful." [95%]
• 26
• 27
• 28 Explanation:
• 29
• 30
• 31 DM-5 criteria for autism spectrum disorder
• 32
• 33
• Multiple, persistent deficits in social communication & interactions currently
• 34
• 35
or by history involving:
• 36 • Social-emotional reciprocity
• 37 • Nonverbal communicative behaviors
• 38 • Developing, maintaining & understanding relationships
• 39
• Restricted, repetitive patterns of behavior currently or by history:
• 40
• 41
• 42 End Block
Feedback
1
2 Item: 2 of 44 r. Mark
▪ 3 Previous Next Lab Values. Notes Calculator.
• 4-
▪ 5
▪ 6 DSM-5 criteria for autism spectrum disorder
▪ 7
▪ 8
▪ 9 • Multiple, persistent deficits in social communication & interactions currently
• 10 or by history involving:
• 11 • Social-emotional reciprocity
• 12
• Nonverbal communicative behaviors
• 13
• Developing, maintaining & understanding relationships
• 14
• 15 • Restricted, repetitive patterns of behavior currently or by history:
• 16 • Repetitive motor movements
• 17 • Insistence on sameness or inflexible adherence to routines
• 18
• Fixated interests of abnormal intensity or focus
• 19
• 20
• Adverse responses to sensory input
• 21 • Symptoms occur in the early developmental period & cause impairment
• 22
• Not better explained exclusively by intellectual disability or another condition
• 23
• 24
4.15M LEWurl d. LLC
• 25
• 26 This patient's impaired social interaction and communication, restricted interests, insistence on
• 27 routine, and delayed language development raise concerns about autism spectrum disorder. Autism
• 28 spectrum disorder can occur with or without language impairment. Language deficits can range from stilted
• 29 speech to language delays (seen in this case) or to complete lack of intelligible speech. Varying degrees of
• 30 intellectual impairment can also accompany autism.
• 31
• 32 The DM-5 diagnosis of autism spectrum disorder encompasses high-functioning autism (Asperger
• 33 syndrome), childhood disintegrative disorder, and any pervasive developmental disorder not otherwise
• 34 specified_ Diagnosis is based on history and behavioral observations (Table}_ The physician should
• 35 empathically listen to the parents' concerns and complete a comprehensive evaluation before making a
• 36 definitive diagnosis. This evaluation includes structured assessments of social, language, and intellectual
• 37 development in addition to hearing, vision, and genetic (eg, Fragile X syndrome) testing.
• 38
• 39 (Choice A) Varying degrees of language impairment frequently accompany autism spectrum disorders.
• 40 Howpvp.r isolated lannuaoa disorripm am not nharantpri7p.el by elpfinits in non-verbal communination sonial
• 41
• 42 End Block
Feedback
1
2 Item: 2 of 44 V-Mark
3 Previous Next Lab Values. Notes Calculator.
• 4 intellectual impairment can also accompany autism.
5
6 The DM-5 diagnosis of autism spectrum disorder encompasses high-functioning autism (Asperger
▪ 7 syndrome), childhood disintegrative disorder, and any pervasive developmental disorder not otherwise
▪ 8 specified_ Diagnosis is based on history and behavioral observations (Table). The physician should
▪ 9 empathically listen to the parents' concerns and complete a comprehensive evaluation before making a
• 10 definitive diagnosis. This evaluation includes structured assessments of social, language, and intellectual
• 11 development in addition to hearing, vision, and genetic (eg, Fragile X syndrome) testing.
• 12
• 13 (Choice A) Varying degrees of language impairment frequently accompany autism spectrum disorders.
• 14 However, isolated language disorders are not characterized by deficits in non-verbal communication, social
• 15 reciprocity, and lack of imaginative play.
• 16
• 17
(Choice B) Even though this patient is inattentive and does not listen to his mother, this is likely due to the
• 18
lack of social reciprocity seen in autism spectrum disorders. Diagnosis of attention deficit hyperactivity
• 19
disorder requires the presentation of multiple inattentive symptoms (eg, distractibility, forgetfulness, avoidance
• 20
of tasks, lack of follow through, careless mistakes); these are not exhibited by this patient
• 21
(Choice C) Intellectual disability may accompany autism but would not explain this patient's impaired social
• 22
communication and interactions.
• 23
• 24
(Choice 0) In the absence of focal neurologic findings, routine neuroimaging is not recommended for
• 25
patients with suspected autism spectrum disorder. These patients often require sedation for MRI, and this is
• 26
an unnecessary risk when imaging will likely be low yield.
• 27
• 28 Educational objective:
• 29 Autism spectrum disorder should be suspected in children with impaired social communication/interactions
• 30 and restricted repetitive interests or behaviors. Comprehensive evaluation includes structured assessments
• 31 of social, language, and intellectual development in addition to hearing, vision, and genetic testing.
• 32
• 33
References:
• 34
• 35 1. Diagnostic evaluation of autism spectrum disorders.
• 36 2. Autism spectrum disorders.
• 37
• 38
• 39
Copyright @ UWorld Last updated: [8/11/2014]
• 40
• 41
• 42 End Block
Feedback
1
Item: 3 of 44 V- Mark
Previous Next Lab Values. Notes Calculator.

▪ 5
▪ 6 A 12-year-old boy is accused of setting his neighbor's house on fire. His parents say, 'We can't believe he
▪ 7 would do anything like this; he has always been hyperactive but is basically a good kid_ The past year he has
▪ 8 been a little more irritable and argumentative with us, but that's just typical of preteens." They add that he has
▪ 9 wanted to be a firefighter since a young age and spends hours on the internet watching videos about fires and
• 10 firefighting_ The boy's grades are average, and his teachers describe him as a loner who has difficulty making
• 11 friends at school_ Two years ago he was caught setting the interior of his father's car on fire_ The boy has
• 12 also been linked to several suspicious fires in the neighborhood, although no criminal charges have been
• 13 brought against him. Rased on this information, which of the following is the most likely diagnosis?
• 14
• 15
• 16
A. Antisocial personality disorder [2%]
• 17 13. Attention deficit hyperactivity disorder [1°,6]
• 18
C. Conduct disorder [16°,6]
• 19 I
• 20
ID. Normal developmental experimentation [0%]
• 21 E Oppositional defiant disorder [2%]
• 22
• E Pyromania [80%]
• 23
• 24
• 25 Explanation:
• 26
• 27
• 28 Pyromania
• 29
• 30
DSM-5 diagnosis
• 31
• 32 • Deliberate fire setting on more than 1 occasion
• 33
• Tension, arousal prior to act
• 34
• 35 • Fascination with fire & its consequences
• 36
• Pleasure or relief when settingiwitnessing fires
• 37
• 38 • No external gain, revenge, or political motivation; not done to attract attention
• 39
• Not better explained bar conduct disorder, manic episode, psychosis, antisocial
• 40 •
• 41
• 42 End Block
Feedback
1
Item: 3 of 44 Mark
Previous Next Lab Values. Notes Calculator.

▪ 5
▪ 6
Pyromania
▪ 7
▪ 8 DSM-5 diagnosis
▪ 9
• 10
• Deliberate fire setting on more than 1 occasion
• 11 • Tension, arousal prior to act
• 12
• Fascination with fire & its consequences
• 13
• 14 • Pleasure or relief when settinglwitnessing fires
• 15
• No external gain, revenge, or political motivation; not done to attract attention
• 16
• 17 • Not better explained by conduct disorder, manic episode, psychosis, antisocial
• 18 personality disorder, or impaired judgment (neurocognitive disorder, substance
• 19 intoxication)
• 20
• 21 Cr]USMLEWorld. LLC
• 22
• 23 This child's fascination with fire and history of repetitive, intentional fire setting suggest a diagnosis of
• 24 pyromania, a rare impulse control disorder more commonly found in males. DM-5 describes pyromania as
• 25 intentional fire setting on more than one occasion to relieve internal tension. There is no obvious motive (eg,
• 26 profit, revenge, political statement, recognition). Individuals with this condition tend to be fascinated by fire and
• 27 anything related to it (eg, fire stations, firefighters). They deliberately start fires to reduce tension and feel
• 28 pleasure or relief afterward. Pyromania must be differentiated from fire setting associated with a manic
• 29 episode, psychosis, or substance intoxication_
• 30
• 31 Fire setting is also a symptom of conduct disorder (Choice C), a psychiatric condition of childhood and
• 32 adolescence characterized by a repetitive and persistent pattern of violating major societal rules or the rights
• 33 of others. The diagnosis of conduct disorder requires at least 3 of 15 behaviors that fall into 4 categories:
• 34 aggression toward people and animals, deceitfulness or theft, destruction of property (includes fire setting),
• 35 and serious violation of rules. This patient exhibits only 1 of 15 possible behaviors seen in conduct
• 36 disorder. A separate diagnosis of pyromania is not indicated when a patient who sets fires meets the criteria
• 37 for conduct disorder.
• 38
(Choice A) The diagnosis of antisocial personality disorder is given to individuals MB years. It involves a
• 39
pattern of violation of basic societal rules and the rights of others and requires a history with some symptoms se
• 40
• 41
• 42 End Block
Feedback
1
Item: 3 of 44 [Ell V-• Mark
Previous Next Lab Values. Notes Calculator.
pyromania, a rare impulse control disorder more commonly tound in males. IJSM-5 describes pyromania as
▪ 5 intentional fire setting on more than one occasion to relieve internal tension. There is no obvious motive (eg,
▪ 6 profit, revenge, political statement, recognition). Individuals with this condition tend to be fascinated by fire and
▪ 7 anything related to it (eg, fire stations, firefighters). They deliberately start fires to reduce tension and feel
▪ 8 pleasure or relief afterward. Pyromania must be differentiated from fire setting associated with a manic
▪ 9 episode, psychosis, or substance intoxication_
• 10
• 11 Fire setting is also a symptom of conduct disorder (Choice C), a psychiatric condition of childhood and
• 12 adolescence characterized by a repetitive and persistent pattern of violating major societal rules or the rights
• 13 of others. The diagnosis of conduct disorder requires at least 3 of 15 behaviors that fall into 4 categories:
• 14 aggression toward people and animals, deceitfulness or theft, destruction of property (includes fire setting),
• 15 and serious violation of rules. This patient exhibits only 1 of 15 possible behaviors seen in conduct
• 16 disorder. A separate diagnosis of pyromania is not indicated when a patient who sets fires meets the criteria
• 17 for conduct disorder.
• 18
(Choice A) The diagnosis of antisocial personality disorder is given to individuals X18 years. It involves a
• 19
pattern of violation of basic societal rules and the rights of others and requires a history with some symptoms
• 20
of conduct disorder before age 15.
• 21
• 22
(Choices B and E) Although this boy's parents describe him as hyperactive, irritable, and argumentative,
• 23
there is insufficient information to substantiate a diagnosis of attention deficit hyperactivity disorder (ADHD) or
• 24
oppositional defiant disorder (ODD). Children with ADHD are inattentive, hyperactive, and impulsive, resulting
• 25
in academic difficulties and impaired social relationships_ amp is characterized by a pattern of angry/irritable
• 26
mood and argumentative/defiant behavior toward authority figures_ Children with ADHD or ODD do not
• 27
typically engage in fire setting_
• 28
• 29 (Choice ID) Children are naturally curious about fire, and accidental or occasional fire setting by unsupervised
• 30 children may occur_ This happens most commonly in children age 5-10 who do not understand the dangers
• 31 of playing with fire_ In contrast, pyromania involves intentional, recurrent fire setting to relieve tension.
• 32
• 33 Educational objective:
• 34 Pyromania is characterized by intentional and repeated fire setting with no obvious motive_ Individuals with
• 35 conduct disorder can also have a history of fire setting, but other features (eg, lying, theft, cruelty to others)
• 36 are also present
• 37
• 38 References:
• 39
• 40 1. Clinical characteristics and psychiatric el:morbidity of pyromania se
• 41
• 42 End Block
Feedback
• 1
2 Item: 3 of 44 VI/lark
Previous Next Lab Values. Notes Calculator.
• 4
• 5 Fire setting is also a symptom of conduct disorder (Choice C), a psychiatric condition of childhood and
• 6 adolescence characterized by a repetitive and persistent pattern of violating major societal rules or the rights
. 7 of others. The diagnosis of conduct disorder requires at least 3 of 15 behaviors that fall into 4 categories:
• 8 aggression toward people and animals, deceitfulness or theft, destruction of property (includes fire setting),
• 9 and serious violation of rules. This patient exhibits only 1 of 15 possible behaviors seen in conduct
• 10 disorder. A separate diagnosis of pyromania is not indicated when a patient who sets fires meets the criteria
• 11 for conduct disorder.
• 12
• 13 (Choice A) The diagnosis of antisocial personality disorder is given to individuals L18 years. It involves a
• 14 pattern of violation of basic societal rules and the rights of others and requires a history with some symptoms
• 15 of conduct disorder before age 15.
• 16
(Choices B and E) Although this boy's parents describe him as hyperactive, irritable, and argumentative,
• 17
there is insufficient information to substantiate a diagnosis of attention deficit hyperactivity disorder (ADHD) or
• 18
oppositional defiant disorder (ODD). Children with ADHD are inattentive, hyperactive, and impulsive, resulting
• 19
in academic difficulties and impaired social relationships. ODD is characterized by a pattern of angry/irritable
• 20
mood and argumentative/defiant behavior toward authority figures_ Children with ADHD or ODD do not
• 21
typically engage in fire setting_
• 22
• 23
(Choice 0) Children are naturally curious about fire, and accidental or occasional fire setting by unsupervised
• 24
children may occur_ This happens most commonly in children age 5-10 who do not understand the dangers
• 25
of playing with fire_ In contrast, pyromania involves intentional, recurrent fire setting to relieve tension.
• 26
• 27 Educational objective:
• 28 Pyromania is characterized by intentional and repeated fire setting with no obvious motive_ Individuals with
• 29 conduct disorder can also have a history of fire setting, but other features (eg, lying, theft, cruelty to others)
• 30 are also present.
• 31
• 32
References:
• 33
. 34 1. Clinical characteristics and psychiatric comorbidity of pyromania
• 35 2. Prevalence and correlates of fire-setting in the United States: results from the National
• 36 Epidemiologic Survey on Alcohol and Related Conditions (NESARC).
• 37
• 38
• 39
Copyright © LIWorld Last updated: [11/27/2014]
• 40
• 41
• 42 End Block
Feedback
• 1
2 Item: 4 of 44 V-Mark

• 3 Previous Next Lab Values. Notes Calculator.
4 A

. 5

• 6 A 10-year-old boy is brought to an urgent care center by his mother with a 1-day history of "rash' on his lower
. 7 extremities following a viral illness 7 days earlier_ He has also had abdominal pain and nausea for 24 hours.
• 8 He has no travel history and no sick contacts. The patient has no past medical history and takes no
• 9 medications. Vital signs are within normal limits for his age_ The abdomen is soft with mild diffuse
• 10 tenderness to deep palpation; there is no rebound tenderness or guarding. Bowel sounds are normal.
• 11 Bilateral lower extremities have nonblanching palpable purplish lesions below the knee and mild swelling of the
• 12 feet The remainder of the examination is normal_
• 13
• 14 Laboratory results are as follows:
• 15 Complete blood count
• 16 Hemoglobin 13.0 WI:IL
• 17 Hematocrit 40%
• 18 Platelets 140,000/u L
• 19 Leukocytes 7,500/4
• 20
• 21 Urinalysis
• 22 Specific gravity 1.025
• 23 pH
• 24 Protein +1
• 25 Blood Moderate
• 26 Glucose Negative
• 27 Ketones Negative
• 28 Leukocyte esterase Negative
• 29 Nitrites Negative
• 30 Bacteria None
• 31 White blood cells 1-2/hpf
• 32 Red blood cells 20-30/hpf
• 33 Casts None
• 34 Crystals None
• 35
• 36
Which of the following is the most likely cause of this child's skin manifestations?
• 37
• 38
• 39 A. Bacterial infection [5%]
• 40 IRnnp r-r-irrriutr infiltmtinn 1-1 041 so
• 41
• 42 End Block
Feedback
• 1
2 Item: 4 of 44 F'Mark

. 3 Previous Next Lab Values. Notes Calculator.
4 Crystals None
. 5
6
• Which of the following is the most likely cause of this child's skin manifestations?
. 7

• 8
• 9 A. Bacterial infection [5%]
• 10 B. Bone marrow infiltration [1%]
• -1 -1
C. Clotting factor deficiency [3%]
• 12
• 13 D. Nutritional deficiency [G%]
• 14 • E Vasculitis [85%]
• 15
E Viral infection [7%]
• 16
• 17
• 18 Explanation:
• 19
• 20
• 21 lienoch-Schanlein purpura
• 22
• 23
• 24 • Immune-mediated leukocAoclastic va5culitis
Pathogenesis
• 25 • Associated with IgA deposition in affected organs
• 26
• 27
• 28
• Palpable purpura with normal platelet count &
• 29 coagulation studies
• 30 ▪ Arthritis/arthralgia
• 31
Clinical • Abdominal pain
• 32
• 33 manifestations • Gastrointestinal bleeding
• 34 • Intussusception
• 35
* Renal disease similar to IgA nephropathy
• 36
• 37 • Scrotal pain & swelling
• 38
• 39
- Normal platelet count & coagulation studies v
• 40 Lab-oratory
• 41
• 42 End Block
Feedback
• 1
• 2 Item: 4 of 44 F'Mark
• 3 Previous Next . Lab Values, Notes Calculator.
4
• 5
Henoch-Schonlein purpura
• 6
• 7
• 8 • Immune-mediated leukocytoclastic vasculitis
0 Pathogenesis
• Associated with IgA deposition in affected organs
• 10
11
• 12 . Palpable purpura with normal platelet count &
13
coagulation studies
• 14
• 15
• Arthritis/a rthralgia
• 16 Clinical • Abdominal pain
17 manifestations • Gastrointestinal bleeding
• 18
• Intussusception
• 19
• 20 • Renal disease similar to IgA nephropathy
• 21 • Scrotal pain & swelling
• 22 ,
!
• 23
• 24 • Normal platelet count & coagulation studies
Laboratory
• 25 • Normal to elevated creatinine 1

• 26 findings
• Urinalysis: Hematuria, red cell casts & mild proteinuria
• '7
Z,

• 28
• 20 . Supportive management (hydration & pain control
• 30 with nonsteroidal anti-inflammatory drugs)
• 31
• 32
• Hospitalization indicated for severe abdominal pain,
• 33 renal insufficiency inability to tolerate oral intake, or
Treatment
• 34 altered mental status

• Systemic glucocorticoids in patients with severe
• 36
• abdominal pain unresponsive to nonsteroldal anti-
• 38 inflammatory drugs
• 39
• 40 L..uSMLEwadd,LL.0 se
• .41
• 42 End Block,
Feedback
• 1
• 2 Item: 4 of 44 l 'Mark
• 3 Previous Next . Lab Values. Notes Calculator.
A. OM IA ■ .1.1 I 1.1.
altered mental status
• 5
• Systemic glucocorticoids in patients with severe
• 6
• 7 abdominal pain unresponsive to nonsteroidal anti-
• 8 inflammatory drugs
• 9
• 10 kZi OhtLE wodld. 1.K
• 11
• 12 This child's presentation of hematuria, abdominal pain, and a purpuric rash on the lower extremities without
• 13 thrombocytopenia is most consistent with a vasculttfs, specifically Henoch-SchOnlein purpura (HSP). HSP,
• 14 which is immune (IgA) mediated, is the most common systemic vasculitis of childhood. It typically follows an
• 15 infection (often minor) and presents with the classic tetrad of palpable purpura on the lower extremities,
• 16 arthralgias, abdominal pain, and renal disease. Abdominal pain is typically colicky and is thought to be
• 17 secondary to localized vasculitis of the bowel wall. Arthralgias are most common in the ankles and knees.
• 18 Renal manifestations include hematuria, non-nephrotic-range proteinuria, and a mildly elevated serum
• 19
creatinine. The rash in HSP, which may be purpuric or petechial, typically involves the buttocks, thighs, and
• 20
legs and is distributed symmetrically.
• 21
HSP can be diagnosed clinically in children presenting with classic signs and symptoms_ The diagnosis
• 22
requires the presence of lower-extremity purpura or petechiae with at least one of the following: arthritis or
• 23
arthralgia, renal involvement, abdominal pain, or positive histopathology. Laboratory testing is not required but
• 24
typically demonstrates a normal platelet count and a mildly elevated creatinine. In children with atypical
• 25
presentations, a renal biopsy may be required to confirm the diagnosis; deposition of IgA in the mesangium
• 26
will be seen. Treatment is primarily supportive and consists of hydration and pain control with nonsteroidal
• 27
anti-inflammatory drugs.
• 28
• 29
(Choice A) Purpura fulminans is a life-threatening condition seen with bacterial infections (eg, Neisseria
• 30
rnerringitidis, Streptococcus przeurnornae) that presents with blue or black hemorrhagic, purpuric lesions.
• 31
These patients are typically very ill with fever, hypotension, and evidence of disseminated intravascular
• 32
coagulation_
• 33
• 34 (Choice B) Bone marrow infiltration by leukemic cells can cause purpura. However, patients with leukemia
• 35 often have an elevated white blood cell count and thrombocytopenia, neither of which is present in this patient
• 36
• 37 (Choice C) Common clotting factor deficiencies that cause purpura in children include von Willebrand
• 38 disease, hemophilia A (factor VIII deficiency), and hemophilia B (factor IX deficiency). However, these
• 39 conditions do not present with other systemic findings, such as abdominal pain or renal disease_
• 40
se
• 41
• 42 End Block
Feedback
• 1
• 2 Item: 4 of 44 V-Mark
• 3 Previous Next Lab Values. Notes Calculator.
4
• 5
(Choice A) Purpura fulminans is a life-threatening condition seen with bacterial infections (eg, Neisseria
rnerringitidis, Streptococcus pneurnoniae) that presents with blue or black hemorrhagic, purpuric lesions.
• 6
• 7
These patients are typically very ill with fever, hypotension, and evidence of disseminated intravascular
• 8
coagulation_
9
(Choice B) Bone marrow infiltration by leukemic cells can cause purpura. However, patients with leukemia
• 10
often have an elevated white blood cell count and thrombocytopenia, neither of which is present in this patient.
• 11
• 12 (Choice C) Common clotting factor deficiencies that cause purpura in children include von Willebrand
• 13 disease, hemophilia A (factor VIII deficiency), and hemophilia B (factor IX deficiency). However, these
• 14 conditions do not present with other systemic findings, such as abdominal pain or renal disease.
• 15
• 16 (Choice 0) Nutritional deficiencies, such as vitamin C deficiency (scurvy), can cause petechiae, purpura, or
• 17 ecchymoses. However, these would be extremely unlikely in a previously healthy child without any dietary
• 18 restrictions.
• 19
• 20 (Choice F) Viral exanthems are typically macular, maculopapular, or vesicular rather than purpuric. Rocky
• 21 Mountain spotted fever and atypical measles can have lesions that progress to generalized purpura.
• 22 However, these illnesses often present with fever and additional systemic symptoms, neither of which this
• 23 child has_
• 24
• 25 Educational objective:
• 26 Henoch-SchOnlein purpura (HSP) is an IgA-mediated vasculitis of childhood that presents with palpable
• 27 purpura on the lower extremities, abdominal pain, arthralgias, and renal involvement Children with HSP
• 28 have a normal platelet count, unlike those with other causes of purpura_
• 29
• 30 References:
• 31
1. Henoch-Schonlein Purpura
• 32
• 33 2. EULARIPRINTO/PRES criteria for Henoch-Schonlein purpura, childhood polyarteritis
• 34 nodosa, childhood Wegener granulomatosis and childhood Takayasu arteritis: Ankara
• 35 2008. Part II: Final classification criteria.
• 36 3. Clinical practice: Diagnosis and management of Henoch-Schanlein purpura.
• 37
• 38
• 39
Copyright © UWorld Last updated: [7/20/2014] se
• 40
• 41
• 42 End Block
Feed back
. 1
2 Item: 5 of 44 F' Mark
'
▪ 3 Previous Next Lab Values. Notes Calculator.
. 4

A 1G-year-old boy is brought to the emergency department due to abdominal pain and bloody diarrhea. The
▪ 6 mother says that he was "fine" a few days ago, and then suddenly became ill_ Physical examination shows a
▪ 7 pale and jaundiced child_ There is diffuse abdominal tenderness and 2+ pedal edema_ Laboratory studies
▪ 8 show anemia, thrombocytopenia and renal insufficiency. What is the most likely cause of the patient's
▪ 9 symptoms?
• 10
• 11
A. Vibrio cholera [1%]
• 12 0 B. Escherichia coli [79%]
• 13
C. Crohn's disease [2%]
• 14
• 15
ID. Lactose intolerance [1%]
• 16 E Salmonella poisoning [7%]
• 17
F. Campylobacter jejuni [9%]
• 18
• 19
• 20 Explanation:
• 21
• 22 This patient has hemolytic-uremic syndrome, which is caused by a toxin released by Escherichia coli. The
• 23 verotoxin invades and destroys the colonic epithelial lining, and produces a bloody diarrhea. Subsequent
• 24 activation of the coagulation system and red cell hemolysis results in jaundice. HUS is more common in
• 25 children aged one- to four-years-old, and the initial presentation includes abdominal pain and diarrhea. The
• 26 classic triad of HUS is uremia, thrombocytopenia and hemolytic anemia Investigations include CB C, platelet
• 27 count, blood smear, urinalysis, BUN and creatinine. Treatment is generally supportive, and involves
• 28 plasmapheresis, dialysis if necessary, and steroids. Antibiotics are not indicated_
• 29
• 30 (Choice A) Cholera usually presents during an epidemic with an abrupt, watery, painless diarrhea There
• 31 may be vomiting, and most patients are severely dehydrated. The treatment is fluid replacement
• 32
(Choice C) Crohn's disease can affect any part of the GI tract It can present with fever, abdominal pain,
• 33
diarrhea (sometimes bloody), weight loss and rectal fissures_ It generally has a gradual course, and is not
• 34
seen in the very young. There is no hemolysis or jaundice_
• 35
• 35
(Choice 0) Lactose intolerance has features of chronic diarrhea (not bloody). It can present with abdominal
• 37
pain or cramps, and is usually associated with ingestion of dairy products. It is more common in Asians and
• 38
Africans. The diagnosis is suggested by: (1) improvement of symptoms after abstaining from dairy
• 39
products, (2) acid pH of the stools, and (3) presence of positive reducing sugars. The diagnosis is confirmed v
• 40
• 41
• 42 End Block
Feedback
• 1
• 2 Item: 5 of 44 V-Mark
▪ 3 Previous Next Lab Values. Notes Calculator.
• 4
Explanation:
▪ 6
This patient has hemolytic-uremic syndrome, which is caused by a toxin released by Eschend-ria cob.. The
▪ 7
verotoxin invades and destroys the colonic epithelial lining, and produces a bloody diarrhea. Subsequent
▪ 8
activation of the coagulation system and red cell hemolysis results in jaundice. HUS is more common in
▪ 9
children aged one- to four-years-old, and the initial presentation includes abdominal pain and diarrhea. The
• 10
classic triad of HUS is uremia, thrombocytopenia and hemolytic anemia Investigations include CBC, platelet
• 11
count, blood smear, urinalysis, BUN and creatinine. Treatment is generally supportive, and involves
• 12
plasmapheresis, dialysis if necessary, and steroids. Antibiotics are not indicated_
• 13
• 14 (Choice A) Cholera usually presents during an epidemic with an abrupt, watery, painless diarrhea There
• 15 may be vomiting, and most patients are severely dehydrated. The treatment is fluid replacement.
• 16
• 17 (Choice C) Crohn's disease can affect any part of the GI tract It can present with fever, abdominal pain,
• 18 diarrhea (sometimes bloody), weight loss and rectal fissures. It generally has a gradual course, and is not
• 19 seen in the very young. There is no hemolysis or jaundice_
• 20
• 21 (Choice ID) Lactose intolerance has features of chronic diarrhea (not bloody). It can present with abdominal
• 22 pain or cramps, and is usually associated with ingestion of dairy products. It is more common in Asians and
• 23 Africans. The diagnosis is suggested by: (1) improvement of symptoms after abstaining from dairy
• 24 products, (2) acid pH of the stools, and (3) presence of positive reducing sugars. The diagnosis is confirmed
• 25 by a positive hydrogen breath test
• 26
• 27 (Choice E) Salmonella poisoning is usually due to ingestion of contaminated water, food or milk_ It can
• 28 present with general malaise, fever, and an erythematous rash on the abdomen (Rose spots). Most
• 29 individuals have diarrhea for several weeks. Stool cultures are diagnostic. If untreated, the disease can
• 30 progress to involve the brain, bone, kidney and joints.
• 31
(Choice F) Even though Campytobacterjejurri can cause bloody diarrhea, it does not cause
• 32
thrombocytopenia_ Renal failure can be seen with any diarrhea secondary to dehydration and prerenal
• 33
azotemia.
• 34
• 35
Educational Objective:
• 35
Suspect hemolytic uremic syndrome in a child with sudden-onset abdominal pain, bloody diarrhea, and triad
• 37
of anemia, thrombocytopenia, and renal failure.
• 38
• 39
Copyright © UWorld Last updated: [8/22/2014]
• 40
• 41
• 42 End Block
Feedback
1
2 Item: 6 of 44 V-Mark
3 Previous Next Lab Values. Notes Calculator.
4

A s-year-old Caucasian male complains of fever, sore throat and difficulty swallowing. Small tender lymph
nodes are palpated in the cervical region_ The symptoms subside quickly on penicillin therapy. Ten days
▪ 8 later, the patient presents again with fever, skin rash and fleeting joint pain in the lower extremities. Physical
▪ 9 examination reveals scattered urticaria and palpable lymph nodes in the cervical, axillary and inguinal regions.
• 10 Which of the following is the most likely cause of this patient's current complaints?
• 11
• 12
A. Rheumatic fever [37%]
• 13
• 14 • R. Drug-induced reaction [31%]
• 15 C. Lymphoproliferative disorder [891)]
• 16
ID. Henoch-Schonlein purpura [15136]
• 17
• 18 E Infective endocarditis [21:k]
• 19
• 20
Explanation:
• 21
• 22
This young boy's initial presentation is suggestive of an upper respiratory tract infection. The onset of fever,
• 23
rash and joint pain 10 days after penicillin therapy, however, suggests a separate drug-induced reaction, most
• 24
likely serum sickness-like reaction_ Serum sickness-like reaction is a hypersensitivity reaction that occurs
• 25
one to two weeks after administration of certain drugs, such as penicillin, amoxicillin, IMP-SMX and cefaclor
• 26
(most classic association}_ It most commonly occurs in young children following the treatment of viral
• 27
infections with antibiotic agents. The mechanism of serum sickness-like reaction is not fully understood, but,
• 28
like true serum sickness, symptoms of fever, urticaria and arthralgias begin one to two weeks after
• 29
administration of the offending agent Serum sickness-like reactions are distinct from immediate
• 30
hypersensitivity reactions, which present with rapid-onset symptoms.
• 31
• 32
(Choice A) Rheumatic fever may occur in cases of streptococcal pharyngitis that are not appropriately
• 33
treated with antibiotics. Patients may present with a combination of polyarthritis, carditis, erythema
• 34
marginatum rash, subcutaneous nodules and chorea. Treatment with penicillin makes rheumatic fever highly
• 35
unlikely in this patient
• 36
• 37
(Choice C) Lymphoproliferative disorders such as leukemia may present with symptoms such as fever,
• 38
lymphadenopathy and joint pains. Complete blood count, blood smear and bone marrow biopsy are useful in
• 39
establishing the diagnosis. This patient's history is more suggestive of a drug reaction. v
• 40
• 41
• 42 End Block
Feedback
Item: 6 of 44 V-Mark
-4(1
Previous Next Lab Values. Notes Calculator.

This young boy's initial presentation is suggestive of an upper respiratory tract infection. The onset of fever,
rash and joint pain 10 days after penicillin therapy, however, suggests a separate drug-induced reaction, most
likely serum sickness-like reaction_ Serum sickness-like reaction is a hypersensitivity reaction that occurs
one to two weeks after administration of certain drugs, such as penicillin, amoxicillin, TMF-SMX and cefaclor
• 8
(most classic association)_ It most commonly occurs in young children following the treatment of viral
▪ 9
infections with antibiotic agents. The mechanism of serum sickness-like reaction is not fully understood, but,
• 10
like true serum sickness, symptoms of fever, urticaria and arthralgias begin one to two weeks after
• 11
administration of the offending agent Serum sickness-like reactions are distinct from immediate
• 12
hypersensitivity reactions, which present with rapid-onset symptoms.
• 13
• 14
(Choice A) Rheumatic fever may occur in cases of streptococcal pharyngitis that are not appropriately
• 15
treated with antibiotics. Patients may present with a combination of polyarthritis, carditis, erythema
• 16
marginatum rash, subcutaneous nodules and chorea. Treatment with penicillin makes rheumatic fever highly
• 17
unlikely in this patient
• 18
• 19
(Choice C) Lymphoproliferative disorders such as leukemia may present with symptoms such as fever,
• 20
lymphadenopathy and joint pains. Complete blood count, blood smear and bone marrow biopsy are useful in
• 21
establishing the diagnosis. This patient's history is more suggestive of a drug reaction.
• 22
• 23
(Choice ID) Henoch-Schonlein purpura is a systemic IgA-mediated vasculitis occurring after viral upper
• 24
respiratory infection. It presents with arthralgias. purpuric rash of the lower extremities, abdominal pain and
• 25
renal disease.
• 26
• 27
(Choice E) Infective endocarditis may cause fever, bacteremia, heart murmur, Osler nodes (nodules on the
• 28
fingers and toes), Janeway lesions (erythematous macules), splinter hemorrhages and Roth spots (retinal
• 29
hemorrhages).
• 30
• 31
Educational objective:
• 32
Serum sickness-like reaction may occur one to two weeks after administration of certain drugs, such as
• 33
penicillin, amoxicillin or cefaclor in the setting of a viral illness. Prominent symptoms include fever, urticarial
• 34
rash, polyarthralgia and lymphadenopathy_ The condition derives its name from its similarities to true serum
• 35
sickness, an immune-complex mediated hypersensitivity reaction to non-human proteins. Serum-sickness
• 36
like reaction is a clinical diagnosis, and should resolve with withdrawal of the offending agent; it does not
• 37
represent a true drug allergy.
• 38
• 39
Copyright © UWorld Last updated: [8/22/2014]
• 40
• 41
• 42 End Block
Feedback
Item: 7 of 44 F' Mark
Previous Next Lab Values. Notes Calculator.

A 5-year-old girl with chronic renal insufficiency is brought to the physician for a follow-up visit Since birth,
she has had multiple episodes of urinary tract infections, for which she takes trimethoprim-sulfamethoxazole
daily for prophylaxis. She has a history of poor growth and mild hypertension but is otherwise developmentally
9 normal. Her mother lost custody of the girl 3 years ago due to failure to comply with recommended treatment
• 10 and prophylaxis of her infections. The girl has since lived with her grandmother. Examination shows mild
• 11 bilateral lower-extremity edema but no other abnormalities_ Urinalysis shows mild proteinuria but no white
• 12 blood cells or bacteria. Renal scintigraphy with dimercaptosuccinic acid shows bilateral focal parenchymal
• 13 scarring and blunted calyces. Which of the following is the most likely predisposing factor for this patient's
• 14 recurrent infections?
• 15
• 16
• 17
A. Common variable immunodeficiency [1%]
• 18 B. Neurogenic bladder [1%]
• 19
C. Posterior urethral valves [119C]
• 20
• 21
ID. Polycystic kidney disease [5%]
• 22 E Recurrent sexual abuse [0%]
• 23
• Unilateral renal agenesis [0*]
• 24
• 25
• G. Vesicoureteral reflux [82%]
• 26
• 27
Explanation:
• 28
• 29
• 30 Vesicoureteral reflux
• 31
• 32
• 33
• 34
• 35
• 36
• 37
• 38
• 39
• 40
• 41
• 42 End Block
Feedback
Item: 7 of 44 F'Mark
-4(1
U.P
Previous Next . Lab Values, Notes Calculator.

Vesicoureteral reflux

• 10
• -1 -1
• 12
• 13
• 14
• 15
• 115
• 17
• 18
• 19
• 20
• 21
• 22
• 23 Normal Grade I Grade II Grade III Grade IV Grade V
• 24
• 25
• 26 Grade Description
• 27
• 28
I Into a nondilated ureter
• 29
• 30
II Into the pelvis & calyces without dilation
• 31
• 32
Mild to moderate dilation of the ureter; renal pelvis & calyces;
• 33 III
with minimal blunting of the fornices
• 34
• 35
IV Moderate ureteral tortuosity & dilation of the pelvis & calyces
• 36
• 37
Gross dilation of the ureter, pelvis & calyces; loss of papillary
• 38
impressions ureteral tortuosity
• 39
• 40
• 41 .
• 42 End Block
Feedback
Item: 7 of 44 Mark
-<1
Previous Next Lab Values. Notes Calculator.

ly Moderate ureterar tortuosity & dilation of the pelvis & calyces

Gross dilation of the ureter, pelvis & calyces; loss of papillary


V
impressions: ureteral tortuosity
9 •
• 10
• 11
©LIS LEWo rld, LLC
• 12
• 13 Recurrent urinary tract infections (UTIs) in infants and children are a serious problem as they often involve
• 14 the kidney and signify a congenital urinary tract anomaly. One of the most common abnormalities is primary
• 15 vesicoureteral reflux (VUR). Normal urine should have unidirectional flow from the kidneys, ureters,
• 16 bladder, and out the urethra. Patients with severe VUR have urinary reflux from the bladder into the kidney,
• 17 and the regurgitant urine causes dilation of the ureters (hydroureter) and kidneys (hydronephrosis).
• 18
• 19 The definitive diagnosis of VUR is made by contrast voiding cystourethrogram. Renal ultrasound is
• 20 performed to screen for hydronephrosis. Recurrent and/or chronic pyelonephritis can lead to blunting of
• 21 calices (calyceal clubbing) and focal parenchymal scarring. Renal scintigraphy with dimercaptosuccinic acid
• 22 is the preferred modality for long-term evaluation for renal scarring. Renal function should be followed by
• 23 serial creatinine. Patients should be monitored closely for complications of chronic renal insufficiency,
• 24 such as hypertension and anemia.
• 25
• 26 (Choice A) Patients with common variable immunodeficiency typically have recurrent sinopulmonary or
• 27 gastrointestinal infections_
• 28
• 29 (Choice B) Neurogenic bladder can cause recurrent UTIs due to urine stasis and secondary reflux from
• 30 inadequate voiding. It is unlikely due to lack of other neurologic deficits in this patient.
• 31
(Choice C) Posterior urethral valves are the most common cause of chronic renal insufficiency/failure in
• 32
children_ This distal urinary tract obstruction can cause secondary urinary reflux but the condition affects only
• 33
boys.
• 34
• 35
(Choice ID) Autosomal recessive polycystic kidney disease manifests in infancy as large flank masses,
• 36
respiratory distress from pulmonary hypoplasia, and Potter faces (flattened ears/nose, micrognathia from
• 37
oligohydramnios). Autosomal dominant polycystic kidney disease is usually asymptomatic in childhood_
• 38
• 39 (Choice E) Although frequent sexual intercourse is a risk factor for recurrent UTIs in women, urinary tract
• 40
v
• 41
• 42 End Block
Feedback
• 1
Item: 7 of 44 VI/lark
3 Previous Next Lab Values. Notes Calculator.

is the preferred modality for long-term evaluation for renal scarring. Renal function should be followed by
serial creatinine. Patients should be monitored closely for complications of chronic renal insufficiency,
such as hypertension and anemia

• 8 (Choice A) Patients with common variable immunodeficiency typically have recurrent sinopulmonary or
• 9 gastrointestinal infections_
• 10
• 11 (Choice B) Neurogenic bladder can cause recurrent UTIs due to urine stasis and secondary reflux from
• 12 inadequate voiding. It is unlikely due to lack of other neurologic deficits in this patient.
• 13
• 14 (Choice C) Posterior urethral valves are the most common cause of chronic renal insufficiency/failure in
• 15 children. This distal urinary tract obstruction can cause secondary urinary reflux but the condition affects only
• 16 boys.
• 17
• 18
(Choice ID) Autosomal recessive polycystic kidney disease manifests in infancy as large flank masses,
• 19
respiratory distress from pulmonary hypoplasia, and Potter faces (flattened ears/nose, micrognathia from
• 20
oligohydramnios). Autosomal dominant polycystic kidney disease is usually asymptomatic in childhood_
• 21
(Choice E) Although frequent sexual intercourse is a risk factor for recurrent UTIs in women, urinary tract
• 22
anomalies are the principal cause in infants and children.
• 23
• 24
(Choice F) Most patients with a solitary kidney are asymptomatic.
• 25
• 26 Educational objective:
• 27 Severe vesicoureteral reflux can cause recurrent or chronic pyelonephritis. Complications include
• 28 parenchymal scarring, hypertension, and renal insufficiency. Definitive diagnosis is made by voiding
• 29 cystourethrogram_
• 30
• 31
References:
• 32
• 33 1. Summary of the AUA guideline on management of primary vesicoureteral reflux in
• 34 children.
• 35 2. Urinary tract infection: clinical practice guideline for the diagnosis and management of
• 36 the initial UTI in febrile infants and children 2 to 24 months.
• 37
• 38
• 39
Copyright © Morld Last updated: [8/26/2G14]
• 40
• 41
• 42 End Block
Feedback
Item: 7 of 44 .111M 'Mark a'
Previous Next Lab Values Notes Calculator
Media Exhibit

Bilateral vesicoureteral reflux

9
• 10
• -1 -1
• 12
• 13
• 14 Bilateral grade V
• 15 vesicoureteral
• 16 reflux and
• 17
hydroureter
• 18
• 19
• 20
• 21
• 22
• 23
• 24
• 25
• 26
• 27
• 28
• 29
• 30
• 31
• 32
• 33
• 34
• 35
• 36
• 37
• 38
• 39
• 40
• 41
• 42 End Block
Feedback
2 Item: 7 of 44 F'Mark Ff.
3 Previous Next Lab Values. Notes Calculator
4 Media Exhibit
5
6 Posterior urethral valves
7
8
9
Posterior urethral valves
• 10
• -1 -1
• 12
• 13
• 14
• 15
• 16
• 17 Hydro-
• 18 nephrosi5
• 19
• 20
• 21
• 22
• 23
Dilated
• 24 ureters
• 25
• 26
• 27 Reflux
• 28
• 29
• 30
• 31 Distended
• 32 bladder
• 33
• 34
• 35
• 36
• 37
• 38
• 39
• 40
• 41
0
• 42 End Block
Feedback
2 Item: 7 of 44 .11M 'Mar k
3 Previous Next Lab Values Notes Calculator
4 Media Exhibit
5
6 Posterior urethral valveq
7
8
9
• 10
• 11
Hydro-
• 12 nephrosis
• 13
• 14
• 15
• 16 Dilated
• 17
ureters
• 18
• 19
• 20
• 21
Reflux
• 22
• 23
• 24
Distended
• 25
• 26
bladder
• 27
• 28
• 29
• 30
• 31
• 32
• 33
• 34
Urethra obstructed by congenital
• 35
• 36 posterior urethral membrane
01.15MLEWorkl, LLC
• 37
• 38
• 39
• 40 I

• 41
• 42
1
2 Item: 8 of 44 V-Mark
3 Previous Next Lab Values. Notes Calculator.
4-
5
6 A healthy 15-year-old girl comes to the physician for a routine health maintenance examination. She feels well
7 and has no concerns. Her menstrual cycles are regular and last 3-4 days; her last menses was 1 week ago_
8 The patient has been sexually active with one partner for the past year and takes oral contraceptive pills daily.
9 She has no vaginal discharge or pain. She is an honors student in 10th grade and plays varsity soccer. The
• 10 patient has tried marijuana ''a few times" but does not use tobacco or alcohol_ Her parents are healthy_ Her
• 11 maternal grandfather died of a myocardial infarction at age 68. The patient's body mass index is 23 kgim2.
• 12 Vital signs and physical examination are normal. What is the best next step in the evaluation of this patient?
• 13
• 14
• 15
▪ A. Chtarnydia trachornatis testing [68%]
• 16 B. Complete blood count [15%]
• 17
C. Echocardiography [11C]
• 18
• 19
ID. Fasting lipid panel [8%]
• 20 E Urine culture [2%]
• 21
F. Urine toxicology screen [7%]
• 22
• 23
• 24 Explanation:
• 25
• 26 The purpose of screening tests is to identify individuals at increased risk for a particular disease who would
• 27 benefit from diagnosis and treatment. All sexually active women age . 24 should undergo testing for
• 28 Chlarnyaria trachornatis and Neisseria gorrorrhoeae, 2 of the most common sexually transmitted
• 29 infections. Screening is also recommended for any person with a new partner in past 2 months, multiple
• 30 partners, history of sexually transmitted infections, illicit drug use, incarceration, and contact with sex workers.
• 31
• 32 Cer-vicitis is a common manifestation of chlamydia and gonorrhea, but many patients are
• 33 asymptomatic. Undetected and untreated infection can lead to pelvic inflammatory disease and its
▪ 34 associated complications (eg, infertility, ectopic pregnancy, chronic pelvic pain). In addition, these chlamydial
• 35 and gonorrheal infections can facilitate HIV transmission.
• 36
The best screening test is the nucleic acid amplification test, which has high sensitivity and specificity. The
• 37
test can be performed on urine, endocervical, vaginal, or urethral specimens with similar accuracy. Patients
• 38
diagnosed with infection should receive antibiotics immediately and refrain from sexual intercourse until
• 39
treatment is complete and symptoms have resolved. All sex partners from the preceding 2 months should V
• 40
• 41
• 42 End Block
Feedback
1
2 Item: 8 of 44 V- Mark
-4Z1

3 Previous Next Lab Values. Notes Calculator.


4- dbbULIdLeU 1,1MT1p114,dLIU116 key, irlieruilLy, eL.LupiL preyridriLy, 4,1- 11- U1114, peiviu pairq_ in dUUILIU11,

5 and gonorrheal infections can facilitate HIV transmission.


6
The best screening test is the nucleic acid amplification test, which has high sensitivity and specificity. The
7
test can be performed on urine, endocervical, vaginal, or urethral specimens with similar accuracy_ Patients
8
diagnosed with infection should receive antibiotics immediately and refrain from sexual intercourse until
9
treatment is complete and symptoms have resolved. All sex partners from the preceding 2 months should
• 10
also be tested and treated for infection.
• 11
• 12
(Choice B) Patients who have no symptoms (eg, fatigue, pallor, dyspnea) and no menorrhagia do not require
• 13
routine complete blood count testing.
• 14
• 15 (Choice C) Electrocardiogram, echocardiography, and exercise testing should be performed in athletes at
• 16 risk for sudden cardiac death. High-risk patients include those with a history of Marfan syndrome, chest pain,
• 17 or dyspnea on exertion; family history of cardiomyopathy or long-QT syndrome; and premature cardiac death
• 18 or disability in a close relative age <50_ Routine screening is otherwise not recommended due to risk of false-
• 19 positive results and lack of cost efficiency.
• 20
• 21 (Choice ID) Universal screening for dyslipidemia is recommended at age 9-11 and at age 17-21, as lipid
• 22 levels are relatively stable just prior to and after puberty. Screening outside of these periods should occur in
• 23 patients at high risk for cardiovascular disease (eg, history of obesity/diabetes mellitus/tobacco exposure,
• 24 family history of premature coronary disease) and men age
• 25
• 26 (Choice E) Routine screening for asymptomatic bacteriuria is not recommended in men and nonpregnant
• 27 women. However, pregnant women should be screened for asymptomatic bacteriuria due to the increased
• 28 incidence of pyelonephritis and low birth weight
• 29
• 30 (Choice F) A basic drug test can detect amphetamine, cocaine, marijuana, opioids, and phencyclidine that
• 31 have been used within a few days of the test Testing may be useful if the patient is in a drug abuse
• 32 rehabilitation or pain management program or is receiving psychiatric care_ Random drug screening is not
• 33 recommended, but adolescents should be counseled on the increased risk of unintentional injuries, motor
▪ 34 vehicle crashes, abuse, and dependence.
• 35
Educational objective:
• 36
All sexually active women ages24 should be screened for Chlarnydia trachornatis and Neissena gonorriroeae
• 37
by nucleic acid amplification testing. Athletes with risk factors for sudden death should undergo cardiac
• 38
evaluation, but routine screening is not recommended otherwise_ Random urine toxicology is generally not
• 39
recommended, but the risks of short- and-long term drug use should be discussed.
• 40
• 41
• 42 End Block
Feedback
1
2 Item: 8 of 44 V-Mark
3 Previous Next Lab Values. Notes Calculator.
4
(Choice C) Electrocardiogram, echocardiography, and exercise testing should be performed in athletes at
5
risk for sudden cardiac death_ High-risk patients include those with a history of Mar-fan syndrome, chest pain,
6
or dyspnea on exertion; family history of cardiomyopathy or long-QT syndrome; and premature cardiac death
7
or disability in a close relative age <50_ Routine screening is otherwise not recommended due to risk of false-
8
positive results and lack of cost efficiency.
9
• 10 (Choice 0) Universal screening for dyslipidemia is recommended at age 9-11 and at age 17-21, as lipid
• 11 levels are relatively stable just prior to and after puberty. Screening outside of these periods should occur in
• 12 patients at high risk for cardiovascular disease (eg, history of obesity/diabetes mellitus/tobacco exposure,
• 13 family history of premature coronary disease) and men age
• 14
• 15 (Choice E) Routine screening for asymptomatic bacteriuria is not recommended in men and nonpregnant
• 16 women. However, pregnant women should be screened for asymptomatic bacteriuria due to the increased
• 17 incidence of pyelonephritis and low birth weight.
• 18
• 19 (Choice F) A basic drug test can detect amphetamine, cocaine, marijuana, opioids, and phencyclidine that
• 20 have been used within a few days of the test. Testing may be useful if the patient is in a drug abuse
• 21 rehabilitation or pain management program or is receiving psychiatric care_ Random drug screening is not
• 22 recommended, but adolescents should be counseled on the increased risk of unintentional injuries, motor
• 23 vehicle crashes, abuse, and dependence.
• 24
• 25 Educational objective:
• 26 All sexually active women ages 4 should be screened for Chlarnyciria trachornatis and Neissena gonorriloeae
• 27 by nucleic acid amplification testing_ Athletes with risk factors for sudden death should undergo cardiac
• 28 evaluation, but routine screening is not recommended otherwise_ Random urine toxicology is generally not
• 29 recommended, but the risks of short- and-long term drug use should be discussed.
• 30
• 31 References:
• 32
1. USPSTF recommendations for STI screening.
• 33
• 34 2. Results of random drug testing in an adolescent substance abuse program.
• 35 3. Screening for asymptomatic bacteriuria in adults: evidence for the U.S. Preventive
• 36 Services Task Force reaffirmation recommendation statement.
• 37
• 38
• 39
Copyright © LIWorld Last updated: [11/24/2014]
• 40
• 41
• 42 End Block
Feedback
1
• 2 Item: 9 of 44 F' Mark
• 3 Previous Next Lab Values. Notes Calculator.
• 4- 11.
• 5
6 A 3-year-old girl is brought to the emergency department with lethargy and fever_ She had diarrhea for several
7 days, and her parents say that "she suddenly took a turn for the worse''. The girl has refused liquids for the
past 12 hours and has not urinated today. She has no allergies and takes no medications. Her temperature
is 397 C (103.5 F), blood pressure is 60/28 mm Hg, and pulse is 145/min. On examination, she is lethargic
• 10 and has poor skin turgor. Her capillary refill time is 5 seconds centrally_ Despite numerous attempts, it is not
• 11 possible to start a peripheral intravenous line, and the child's condition continues to deteriorate. Which of the
• 12 following is the best next step in management of this patient?
• 13
• 14
• 15
A. Attempt arterial line placement [2°4]
• 16 B. Attempt central venous catheter placement [12%]
• 17
C. Attempt intraosseous cannulation [69'36]
• 18
• 19
ID. Attempt nasogastric tube placement [1%]
• 20 E. Attempt peripheral intravenous line placement [116]
• 21
F. Transport to intensive care unit for central venous catheter placement [15%]
• 22
• 23
• 24 Explanation:
• 25
• 26 This child's presentation is concerning for hypovolemic or septic shock and requires emergency fluid
• 27 resuscitation. When intravenous access cannot be obtained in emergency cases, intraosseous (10)
• 28 access should be attempted immediately. 10 access requires less skill and practice than central line
• 29 placement, and clinical trials have demonstrated 10 lines to be safer and faster than central lines. 10
• 30 catheters provide a cannula large enough to deliver fluids and medications rapidly and to obtain blood samples
• 31 for laboratory testing_
• 32
• 33 The most common site for 10 access is the proximal tibia due to its wide, flat surface and distance from the
• 34 sternum in case cardiopulmonary resuscitation is performed simultaneously. However, any large bone can
• 35 be used. la catheters can be placed manually or with a driver. Contraindications to 10 placement include
• 36 infection (eg, cellulitis) overlying the access site, fracture or previous 10 attempts in the chosen extremity, or
• 37 bone fragility (eg, osteogenesis imperfecta).
• 38
(Choice A) Arterial lines are used for continuous blood pressure monitoring and to draw laboratory studies.
• 39
They are not used for fluid resuscitation and should be placed after the patient has been resuscitated as the V
• 40
• 41
• 42 End Block
Feedback
1
2 Item: 9 of 44 V-• Mark
3 Previous Next Lab Values. Notes Calculator.
I I 14-. I I 4,4-01111114-011 .J11-4-. I LI 14-. r.11 6,..nallual 1.11411;1 4,14,14-. LU ILA L.11.JLE-.111.......-. II UI II LI 14-.
4- A
sternum in case cardiopulmonary resuscitation is performed simultaneously. However, any large bone can
5
be used_ 10 catheters can be placed manually or with a driver_ Contraindications to 10 placement include
6
infection (eg, cellulitis) overlying the access site, fracture or previous la attempts in the chosen extremity, or
7
bone fragility (eg, osteogenesis imperfecta).

(Choice A) Arterial lines are used for continuous blood pressure monitoring and to draw laboratory studies_
• 10
They are not used for fluid resuscitation and should be placed after the patient has been resuscitated as the
• 11
procedure takes time and great skill_
• 12
• 13 (Choices B and F) Central catheters take longer to place than la lines and require a higher amount of
• 14 procedural skill and practice_ This child needs immediate access for fluid resuscitation, and transporting her
• 15 within the hospital will cause delays. A central line can be placed later if necessary when the patient is stable.
• 16
• 17 (Choice 0) Although nasogastric fluids are preferred for mild to moderate dehydration associated with
• 18 gastrointestinal disease, they are not recommended in shock, when the splanchnic circulation is constricted_
• 19 At this point, absorption of enteral fluids will not be rapid enough to correct the patient's hypovolemia_
• 20
• 21 (Choice E) Several attempts at peripheral intravenous insertion have already failed_ Given that the child is in
• 22 uncompensated shock, access should be escalated to 10 placement.
• 23
• 24 Educational objective:
• 25 lntraosseous lines can be placed rapidly when emergency access is necessary and peripheral access cannot
• 26 be obtained_ lntraosseous access can be performed with less required skill and practice than central venous
• 27 access_
• 28
• 29 References:
• 30
1. Comparison of umbilical venous and intraosseous access during simulated neonatal
• 31
resuscitation.
• 32
• 33 2. Comparison of intraosseous versus central venous access in adults under resuscitation
• 34 in the emergency department with inaccessible peripheral veins
• 35 3. Intraosseous versus intravenous vascular access during out-of-hospital cardiac arrest: a
• 36 randomized controlled trial.
• 37
• 38
• 39
Copyright © UWorld Last updated: [8/27/2014]
• 40
• 41
• 42 End Block
Feedback
1
2 Item: 10 of 44 F' Mark
3 Previous Next Lab Values. Notes Calculator.
4-
5
6 A 5-year-old boy is brought to the physician after he was hit in the head with a baseball. The incident occurred
7 about two hours ago_ He had no loss of consciousness, but was a little dazed after being hit However, he
8 seemed to improve after a short time. About 30 minutes ago, the patient began to complain of a headache,
vomited twice, and is currently not acting right according to his parents_ On examination, the child is sleepy
and his left pupil is larger than his right A CT scan of his head is shown below.
• 11
• 12
• 13
• 14
• 15
• 16
• 17
• 18
• 19
• 20
• 21
• 22
• 23
• 24
• 25
• 26
• 27
• 28
• 29
• 30
• 31
• 32
• 33
• 34
• 35
• 36
• 37
• 38
• 39
• 40
se
• 41
• 42 End Block
Feedback
1
2 Item: 10 of 44 Mark
3 Previous Next Lab Values. Notes Calculator.
4-
5
6
7
8

• 11
• 12
• 13 V.Thich of the following is the most appropriate next step in management?
• 14
• 15 A. Cerebral angiogram [1%]
• 16
MRI of the brain [1%]
• 17
• 18 C. Repeat CT in 24 hours [2%]
• 19 ID. IDexamethasone [6%]
• 20
• 21
E Emergent craniotomy [89%]
• 22
• 23
Explanation:
• 24
• 25 Epidural hematomas are usually caused by injury to meningeal blood vessels, and do not usually cross suture
• 26 lines because of the better dural attachment at the sutures. The classic history of an epidural hematoma
• 27 consists of a direct head injury followed by a lucid interval and then rapid neurologic deterioration with
• 28 headache, vomiting, seizures, confusion, and lethargy. Affected patients can progress to coma and death
• 29 due to herniation if not treated.
• 30
• 31 Physical examination findings vary by age_ Infants with open fontanelles may present with bulging fontanelles,
• 32 irritability, seizures, and hypotonia. Older children and adolescents usually present with more classic
• 33 symptoms of headache, vomiting, and deteriorating mental status_
• 34
• 35 A CT scan of the head is the diagnostic test of choice. The key to evaluating a child with an epidural
• 36 hematoma is to search for the presence of clinical features that necessitate an emergent craniotomy, which
• 37 include any of the following: GCS < 8, signs of increased ICF, pupillary abnormalities, hemiparesis, or
• 38 cerebellar signs. The patient in this vignette has pupillary abnormalities, vomiting, and a decreased level of
• 39 consciousness, and should therefore have a neurosurgical evaluation for an emergent craniotomy.
• 40
se
• 41
• 42 End Block
Feedback
• 1
• 2 Item: 10 of 44 V-Mark
-4Z1

3 Previous Next Lab Values. Notes Calculator.


4- Ilfle6 UeLdU6e UI Lilt UeLLel UUldl dadLAIHIefIL dL Lilt 6ULUE e6. I He L-Id6611, riibLury UI dfl epiuurdi fief EldLUElld

5 consists of a direct head injury followed by a lucid interval and then rapid neurologic deterioration with
6 7 headache, vomiting, seizures, confusion, and lethargy. Affected patients can progress to coma and death
due to herniation if not treated.

Physical examination findings vary by age_ Infants with open fontanelles may present with bulging fontanelles.
irritability, seizures. and hypotonia. Older children and adolescents usually present with more classic
symptoms of headache, vomiting, and deteriorating mental status.
• 11
• 12
A CT scan of the head is the diagnostic test of choice. The key to evaluating a child with an epidural
• 13
hematoma is to search for the presence of clinical features that necessitate an emergent craniotomy, which
• 14
include any of the following: GCS < 8, signs of increased ICF, pupillary abnormalities, hemiparesis, or
• 15
cerebellar signs. The patient in this vignette has pupillary abnormalities, vomiting, and a decreased level of
• 16
consciousness, and should therefore have a neurosurgical evaluation for an emergent craniotomy.
• 17
• 18 (Choice A) An angiogram may be helpful when an epidural hematoma is suspected to be secondary to an
• 19 arteriovenous malformation (AVM). This patient, however, has a clear mechanism of injury resulting in his
• 20 epidural hematoma, making an AVM extremely unlikely_
• 21
• 22 (Choice B) A MRI of the brain is not useful in the acute evaluation of an epidural hematoma because it takes
• 23 too long to obtain, which delays appropriate intervention.
• 24
• 25 (Choice C) In patients with no neurologic signs and a small epidural hematoma, a follow-up CT in 24 hours
• 26 may be appropriate.
• 27
• 28 (Choice ID) Steroids have no role in the acute management of an epidural hematoma.
• 29
Educational objective:
• 30
Epidural hematomas are often associated with a lucid interval followed by rapid neurological deterioration_
• 31
They appear as a biconvex mass on CT scan of the head. and patients with features suggestive of
• 32
deteriorating neurological status or increased IP require an emergent craniotomy.
• 33
• 34
• 35 References:
• 36 1. Extradural haematomas in children: a 10-year review.
• 37
• 38
• 39
Copyright © LIWorld Last updated: [12/29/2014]
• 40
• 41
• 42 End Block
Feedback
• 1
• 2 Item: 11 of 44 VI/lark
▪ 3 Previous Next Lab Values. Notes Calculator.
• 4-
▪ 5
▪ 6 A 12-year-old boy is brought to the pediatrician by his mother for a routine examination. The boy has an
▪ 7 eight-year history of episodes in which he utters strange sounds, clears his throat, grimaces, and blinks his
▪ 8 eyes. Recently he has also started to jerk his head during these episodes as well. His medical history is
▪ 9 otherwise unremarkable and he is earning passing grades in school. Physical examination is unremarkable_
• 10 Which of the following disorders is most likely to develop in this boy?
• 11
• 12
• 13
A. Conduct disorder [71C]
• 14 ko B. Obsessive-compulsive disorder [81%]
• 15
C. Oppositional defiant disorder [6%]

• 17
ID. Narcissistic personality disorder [2%]
• 18 E Antisocial personality disorder [4%]
• 19
• 20
• 21 Explanation:
• 22
Occurring more frequently in males, Tourette syndrome is characterized by multiple motor and one or more
• 23
vocal tics that present before the age of 18. The tics occur many times a day (frequently in bouts) nearly every
• 24
day or at regular intervals for at least one year. The motor tics frequently observed include grimacing, eye
• 25
blinking, nose twitching, head jerking, and shoulder shrugging. The vocal tics include barking, grunting,
• 26
squeaking, coughing, and throat clearing_ In a minority of cases, coprolalia may be observed_ Symptoms are
• 27
exacerbated by stress and tend to subside during sleep_
• 28
• 29

Frequent comorbid conditions in this patient population include attention deficit hyperactivity disorder (60
• 30
percent) and obsessive-compulsive disorder (27 percent). Obsessive-compulsive disorder (O ID) develops
• 31
within 3-6 years after the tics first appeared. It may peak in late adolescence or in early adulthood at a time
• 32
when the tics are waning. Less common comorbid conditions include anxiety, depression, and impulse
• 33
control disorders.
• 34
• 35 (Choices A, C, D and E) There is an increased incidence of conduct disorder, oppositional defiant disorder,
• 36 and antisocial personality disorder in patients suffering from Tourette syndrome, but attention deficit
• 37 hyperactivity disorder and obsessive-compulsive disorder are more common comorbid conditions.
• 38
• 39 Educational Objective:
• 40 ID afro ni-c. 1* rifin Tni i rai-Fa c,...nr-4rnrna F, at en a c i n n if i r a ni-hr inr rna c nr-1 ri c lr of tint en lnninn nn r if H1 ma ran fiariizr e
s

• 41
• 42 End Block
Feedback
• 1
• 2 Item: 11 of 44 V-• Mark -4Z1

▪ 3 Previous Next Lab Values. Notes Calculator.


ciyi 1113LLII y UI Cl...113LiLIC3 II 1 VV1114,11 11C ULLC13 3L1 01 iyc 3LJUI 1413, I...ICC!! 3 1113 L111 LiCIL, yi II I IC74.GJ , CII1U L.111111‘3 1113
• 4- A
5
eyes. Recently he has also started to jerk his head during these episodes as well. His medical history is
otherwise unremarkable and he is earning passing grades in school. Physical examination is unremarkable_
▪ 6
▪ 7
Which of the following disorders is most likely to develop in this boy?
▪ 8
▪ 9 A. Conduct disorder [7%]
• 10
B. Obsessive-compulsive disorder [81%]
• 11
• 12
C. Oppositional defiant disorder [65.6]
• 13 ID. Narcissistic personality disorder [2%]
• 14
E. Antisocial personality disorder [4%]
• 15

• 17 Explanation:
• 18
• 19 Occurring more frequently in males, Tourette syndrome is characterized by multiple motor and one or more
• 20 vocal tics that present before the age of 18. The tics occur many times a day (frequently in bouts) nearly every
• 21 day or at regular intervals for at least one year. The motor tics frequently observed include grimacing, eye
• 22 blinking, nose twitching, head jerking, and shoulder shrugging. The vocal tics include barking, grunting,
• 23 squeaking, coughing, and throat clearing_ In a minority of cases, coprolalia may be observed_ Symptoms are
• 24 exacerbated by stress and tend to subside during sleep_
• 25
• 26 Frequent comorbid conditions in this patient population include attention deficit hyperactivity disorder (&J
• 27 percent) and obsessive-compulsive disorder (27 percent). Obsessive-compulsive disorder (O D} develops
• 28 within 3- years after the tics first appeared. It may peak in late adolescence or in early adulthood at a time
• 29 when the tics are waning. Less common comorbid conditions include anxiety, depression, and impulse
• 30 • control disorders_
• 31
(Choices A, C, D and E) There is an increased incidence of conduct disorder, oppositional defiant disorder,
• 32
and antisocial personality disorder in patients suffering from Tourette syndrome, but attention deficit
• 33
hyperactivity disorder and obsessive-compulsive disorder are more common comorbid conditions.
• 34
• 35
Educational Objective:
• 36
Patients with Tourette syndrome have a significantly increased risk of developing attention deficit hyperactivity
• 37
disorder or obsessive-compulsive disorder.
• 38
• 39
Copyright © LIWorld Last updated: [1611212014]
• 40
• 41
• 42 End Block
Feedback
• 1
• 2 Item: 12 of 44 V- Mark
▪ 3 Previous Next Lab Values. Notes Calculator.
• 4-
5
▪ 6 A 2-year-old boy with cough and difficulty breathing is brought to the emergency department by his mother_
▪ 7 She says that he was well and playing with his toys until 2 hours prior to presentation. He is healthy, but his
8 s-year old brother has a peanut allergy. The patient's temperature is 3E7 C (98 F), blood pressure is 92148
9 mm Hg, pulse is 1141min, and respirations are 48/min. The patient's pulse oximetry shows 91% on room air_
• 10 Physical examination shows nasal flaring and grunting with both subcostal and intercostal retractions.
• 11 Wheezing is heard in the right lung field; the left field is clear to auscultation. No rales or rhonchi are noted.
The remainder of the physical examination is within normal limits. Supplementary oxygen is applied. Chest
x-ray reveals mild hyperinflation of the right lung. Which of the following is the most appropriate next step in
management of this child?

A. Bronchoscopy [81%]
• 17
• 18 B. Chest computed tomography scan [3%]
• 19 C. Chest physiotherapy [0%]
• 20
D. Chest tube placement [1%]
• 21
• 22 E Intramuscular epinephrine [4%]
• 23 F. Nebulized albuterol [7%]
• 24
G. Racemic epinephrine [3%]
• 25

• 26
• 27 Explanation:
• 28
• 29 Respiratory distress is one of the most common presenting chief complaints in the pediatric emergency
• 30 department Although the differential diagnosis for respiratory distress is broad, this child's clinical
• 31 presentation of sudden-onset respiratory distress without a preceding illness and focal findings on pulmonary
• 32 examination is most consistent with foreign body aspiration (FBA). FBA is most common in children age 1-3
• 33 years. Commonly aspirated FBs include foods such as peanuts and popcorn and pieces of toys. More than
• 34 half of aspirated FBs end up in the right mainstem bronchus; laryngeal and tracheal FBs are far less common_
• 35
• 36 Clinical features of FBA are shown in the table.
• 37
• 38
• 39
Clinical manifestations of foreign body aspiration
V
• 40
• 41
• 42 End Block
Feedback
-1
2 Item: 12 of 44 I'Mark U.P
3 Previous Next Lab Values, Notes Calculator.
4- A

5
6
Clinical manifestations of foreign body aspiration
7
8
9 . History of choking (80%-90% of cases)
• 10 . Coughing
• 11 Signs &
•• Sudden-onset respiratory distress
symptoms
• 13 •• Cyanosis
• 14
6 15
. Altered mental status

• 17
• 18
• Focal monophonic wheezing on affected side
• 19 i Diminished aeration on affected side
• 20 Physical
• 21
• Generalized wheezing
examination
• 22 • Inspiratory stridor
• 23
findings
• 24
• Hoarseness
• 25 • Respiratory distress
• 26
.. ,
• 27
• 28 Radiographic . Hyperinflation or atelectasis of affected side
• 29
findings . Visualization of foreign body
• 30
• 31
USN1L EWorld. t LC
• 32
• 33
• 34 Although chest radiographs are often obtained in patients with suspected FBA, they are normal in
• 35 approximately 2/3 of cases given that most aspirated objects are radiolucent. If an FE causes partial
• 36 obstruction, with air trapping during expiration, hyperinflated lungs are seen on imaging. In contrast, complete
• 37 obstruction can result in atelectasis, post-obstructive pneumonia, and/or localized bronchiectasis (late
• 30 feature).
• 39
• 40
The standard of care for both diagnosis and treatment of FBA is immediate bronchoscopy.
• 41
• 42 End Block
Feedback
1
2 Item: 12 of 44 F' Mark U.P
3 Previous Next Lab Values, Notes Calculator.
- I 11%.1c.111
4- A

5 ▪ Respiratory distress
6
7
8 Radiographic • Hyperinflation or atelectasis of affected side
9
findings • Visualization of foreign body
• 10
• 11
1,JSMILEWorld.ILC

Although chest radiographs are often obtained in patients with suspected FBA, they are normal in
approximately 2/3 of cases given that most aspirated objects are radiolucent. If an FE causes partial
obstruction, with air trapping during expiration, hyperinflated lungs are seen on imaging. In contrast, complete
• 17 obstruction can result in atelectasis, post-obstructive pneumonia, and/or localized bronchiectasis (late
• 18 feature}_
• 19
• 20
The standard of care for both diagnosis and treatment of FBA is immediate bronchoscopy.
• 21
(Choice B) Although an aspirated FE may be seen on computed tomography scan of the chest, a scan is
• 22
not indicated as it requires patient cooperation and only delays the diagnosis.
• 23
• 24
(Choice C) Chest physiotherapy is helpful in removing tenacious secretions and mucous plugs. It is
• 25
indicated in patients with bronchiectasis.
• 26
• 27 (Choice 0) Chest tube placement is indicated for patients with respiratory distress secondary to a large
• 28 pneumothorax, hemothorax, or pleural effusion_ It is not indicated in this patient.
• 29
• 30 (Choice E) Intramuscular epinephrine is the first-line treatment for anaphylaxis. Anaphylaxis typically
• 31 presents with respiratory distress, involvement of the skin-mucosal tissue (eg, generalized hives, itching,
• 32 flushing, swollen lips and tongue), gastrointestinal symptoms (eg, crampy abdominal pain, vomiting), and
• 33 hypotension. Although the patient's sibling has a peanut allergy, his clinical presentation is not consistent with
• 34 anaphylaxis.
• 35
• 36 (Choice F) Although this patient has wheezing on examination, it is focal rather than generalized, as seen
• 37 with asthma. Eronchodilators are not helpful in patients with FBA given that the underlying etiology is a fixed
• 38 obstruction in the bronchus.
• 39
• 40
(Choice G) Nebulized racemic epinephrine is used for laryngotracheobronchitis (croup), which typically
• 41
• 42 End Block
Feedback
• 1
• 2 Item: 12 of 44 F' Mark
▪ 3 Previous Next Lab Values. Notes Calculator.
• 4 indicated in patients with bronchiectasis.
▪ 5
(Choice ID) Chest tube placement is indicated for patients with respiratory distress secondary to a large
▪ 7 pneumothorax, hemothorax, or pleural effusion_ It is not indicated in this patient
8
(Choice El Intramuscular epinephrine is the first-line treatment for anaphylaxis_ Anaphylaxis typically
• 10 presents with respiratory distress, involvement of the skin-mucosal tissue (eg, generalized hives, itching,
• 11 flushing, swollen lips and tongue), gastrointestinal symptoms (eg, crampy abdominal pain, vomiting), and
hypotension_ Although the patient's sibling has a peanut allergy, his clinical presentation is not consistent with
anaphylaxis_

(Choice F) Although this patient has wheezing on examination, it is focal rather than generalized, as seen
with asthma. Eronchodilators are not helpful in patients with FBA given that the underlying etiology is a fixed
• 17 obstruction in the bronchus_
• 18
(Choice G) Nebulized racemic epinephrine is used for laryngotracheobronchitis (croup), which typically
• 19
presents with mild upper respiratory symptoms. a "barky'r cough, and inspiratory stridor. This child has none
• 20
of these_
• 21
• 22
Educational objective:
• 23
Sudden-onset respiratory distress in a toddler with focal findings on physical examination is most consistent
• 24
with foreign body aspiration_ Chest x-ray is of limited help as most objects are radiolucent A history of
• 25
choking, if witnessed, is very helpful in diagnosis_ Immediate bronchoscopy is indicated to remove the foreign
• 26
body_
• 27
• 28
References:
• 29
• 30 1. Indications for flexible versus rigid bronchoscopy in children with suspected
• 31 foreign-body aspiration.
• 32
2. Extraction of tracheobronchial foreign bodies in children and adults with rigid and
• 33
flexible bronchoscopy.
• 34
• 35 3. Foreign body aspiration in children: Experience from 2624 patients.
• 36 4. Tracheobronchial aspiration of foreign bodies and rigid bronchoscopy in children.
• 37
• 38
• 39
Copyright © LIWorld Last updated: [1G/13/2014]
• 40
• 41
• 42 End Block
Feedback
• 1
• 2 Item: 13 of 44 F' Mark
▪ 3 Previous Next Lab Values. Notes Calculator.
• 4
▪ 5
▪ 6 7 A 1-day-old boy is in the newborn nursery with persistent feeding difficulty. He is cyanotic and short of breath
▪ 7 when he breastfeeds but turns pink when he cries. Feeding him expressed breast milk from a bottle results in
▪ 8 the same findings. He is voiding normally and passed meconium. His prenatal, birth, and family histories are
▪ 9 unremarkable. His weight is average for gestational age. Vital signs are normal and 4-extremity blood
• 10 pressures are equal. Examination shows a non-dysmorphic boy with clear lungs and no murmurs_ There are
• 11 no intercostal retractions or nasal flaring. His peripheral pulses are full and symmetric. Which of the following
• 12 is the most likely diagnosis?
13
• 14
• 15
A. Choanal atresia [6416]
B. Laryngomalacia [996]
• 17
C. Tetralogy of Fallot [8%]
• 18
• 19
D. Tracheoesophageal fistula with esophageal atresia [14N
• 20 E. Transient tachypnea of the newborn [5%]
• 21
F. Vocal cord paralysis [0%]
• 22
• 23
• 24 Explanation:
• 25
• 26
• 27
• 28
• 29
• 30
• 31
• 32
• 33
• 34
• 35
• 36
• 37
▪ 38
• 39
• 40
se
41
42 End Block
Feedback
• 1
• 2 Item: 13 of 44 F'Mark
3 Previous Next Lab Values. Notes Calculator.
4 I A

5
6 7
7
8 1
9
• 10 M

:
h
11
2

• 14
• 15
• 11151
• 17
• 18 I
• 19
• 20
• 21
• 22
• 23
• 24
• 25
• 26
• 27
• 28
• 29
• 30
• 31 Choanal atresia should be suspected in a newborn with cyanosis that is aggravated by feeding and relieved
• 32 by crying_ The congenital nasal malformation is caused by failure of the posterior nasal passage to canalize
• 33 completely, leaving either a bony (90%) or membranous (10%) obstruction_ The condition may be isolated or
• 34 part of a syndrome (ie, CHARGE syndrome: Coloboma, Heart Defects, Atresia of the choanae, Renal
• 35 anomalies, Growth impairment, and Ear abnormalitiesIdeafness).
• 36
• 37 The clinical severity depends on the infant's ability to breathe through the mouth and whether 1 or both
• 38 I choanae are obstructed. Bilateral obstruction classically presents with cyclic cyanosis that worsens when
• 39 the infant cannot breathe through the nose during feeding but recovers during crying. Unilateral choanal
• 40 rema in i inriinnnQari !Intl! the infant riaidolnnQ his nr her first i inner raQnirtnnf infortinn so
• 41
• 42 Feedback End Block
A . 1
• 1
• 2 Item: 13 of 44 V- Mark
3 Previous Next Lab Values. Notes Calculator.
4 I part of a syndrome (ie, CHARGE syndrome: Coloboma, Heart Defects, Atresia of the choanae, Renal
5 anomalies, Growth impairment, and Ear abnormalities/deafness).
6 7
7 The clinical severity depends on the infant's ability to breathe through the mouth and whether 1 or both
8 1 choanae are obstructed. Bilateral obstruction classically presents with cyclic cyanosis that worsens when
9 the infant cannot breathe through the nose during feeding but recovers during crying. Unilateral choanal
• 10 M atresia may remain undiagnosed until the infant develops his or her first upper respiratory infection.

:
h
11
2 Failure to pass a catheter through the nose into the oropharynx is suggestive of choanal atresia. The
diagnosis is confirmed by CT scan (Image), which shows a narrowing at the level of the pterygoid plate in the
• 14 posterior nasal cavity (green arrows). In severe cases, air-fluid levels (red arrow) may develop at the
• 15 obstruction site_ The first step in management consists of placing an oral airway and gavage feeding_
• 11151 Definitive treatment involves repairing the obstruction with surgery or endoscopy.
• 17
• 18 I (Choice B) Laryngomalacia classically presents with inspiratory stridor that is exacerbated by exertion or
• 19 distress. Symptoms appear within the first few weeks of life. Cyanosis is uncommon unless the obstruction
• 20 is severe_ However, this infant has no stridor or tachypnea.
• 21
(Choice C) Patients who have Tetralogy of Fallot (TOF) with mild obstruction to pulmonary blood flow are
• 22
asymptomatic at rest and become cyanotic when stressed. These desaturations are known as "ter spells,
• 23
which may be confused with the cyanosis seen in choanal atresia. However, the cyanosis in choanal atresia
• 24
is not triggered by stress but by anything that will keep the infant's mouth closed (ie, during feedings).
• 25
Patients with TOF will also have a systolic ejection murmur of pulmonary stenosis and holosystolic murmur of
• 26
ventricular septal defect These are not seen in this patient, making TOF diagnosis unlikely.
• 27
• 28
(Choice 0) Tracheoesophageal fistula with esophageal atresia can cause feeding problems immediately
• 29 after birth as feeds cannot pass the esophagus and end up in the airway. These patients typically have
• 30
coughing, respiratory distress, and adventitious lung sounds in addition to cyanosis.
• 31
• 32 (Choice E) Transient tachypnea of the newborn is characterized by tachypnea immediately after birth that is
• 33 unrelated to crying. Additional symptoms include nasal flaring, subcostallintercostal retractions and expiratory
• 34 grunting.
• 35
• 36 (Choice F) Vocal cord paralysis is associated with birth injury or central nervous system insults. Infants may
• 37 have unilateral or bilateral paralysis and generally have stridor and respiratory distress_ It cannot be clinically
• 38 I differentiated from laryngomalacia; direct visualization of the upper airway via endoscope is necessary.
• 39 Cyanosis is uncommon unless the obstruction is severe, and this infant has no stridor or tachypnea.
• 40 so
• 41
• 42 Feedback End Block
A . 1
1 i
2. Item: 13 of 44 'Mark
3 Previous Next Lab Values. Notes Calculator.
4 I (Choice B) Laryngomalacia classically presents with inspiratory stridor that is exacerbated by exertion or
5 distress. Symptoms appear within the first few weeks of life. Cyanosis is uncommon unless the obstruction
6 7 is severe. However. this infant has no stridor or tachypnea.
7
8 1 (Choice C) Patients who have Tetralogy of Fallot (TOF) with mild obstruction to pulmonary blood flow are
9 asymptomatic at rest and become cyanotic when stressed. These desaturations are known as 'let" spells,
• 11:1 M which may be confused with the cyanosis seen in choanal atresia. However, the cyanosis in choanal atresia
is not triggered by stress but by anything that will keep the infant's mouth closed (ie, during feedings).
:
h
12
1 Patients with TOF will also have a systolic ejection murmur of pulmonary stenosis and holosystolic murmur of
ventricular septal defect These are not seen in this patient, making TOF diagnosis unlikely.
• 14
• 15
(Choice ID) Tracheoesophageal fistula with esophageal atresia can cause feeding problems immediately
after birth as feeds cannot pass the esophagus and end up in the airway. These patients typically have


11151
17
18 I
coughing, respiratory distress, and adventitious lung sounds in addition to cyanosis.

(Choice E) Transient tachypnea of the newborn is characterized by tachypnea immediately after birth that is
• 19
unrelated to crying. Additional symptoms include nasal flaring, subcostallintercostal retractions and expiratory
• 20
grunting.
• 21
• 22
(Choice F) Vocal cord paralysis is associated with birth injury or central nervous system insults. Infants may
• 23
have unilateral or bilateral paralysis and generally have stridor and respiratory distress_ It cannot be clinically
• 24
differentiated from laryngomalacia; direct visualization of the upper airway via endoscope is necessary_
• 25
Cyanosis is uncommon unless the obstruction is severe, and this infant has no stridor or tachypnea.
• 26 I
• 27 Educational objective:
• 28 I Choanal atresia should be suspected in an otherwise well-appearing infant with intermittent cyanosis and
I
• 29 distress during feeding that is relieved by crying. Failure to pass a catheter through the nose into the
• 30 oropharynx is suggestive of the diagnosis. A CT scan shows narrowing at the level of the pterygoid plate in
• 31 the posterior nasal cavity.
• 32
• 33
References:
• 34
• 35 1. Choanal atresia: current concepts and controversies
• 36 I 2. Nasal obstruction in newborns
• 37
• 38 I
• 39
Copyright © LIWorld Last updated: [9/4/2014]
• 40 I
• 41
• 42 End Block
Feedback
3 =1
1
4
Item: 14 of 44 F Mark
Previous Next Lab Values. Notes Calculator.

5
6 7 A 3-year-old boy is brought to the emergency department because of severe oral pain along with bleeding and
irritation of his gums_ His symptoms began 2 weeks ago and have progressively worsened. He has a history
8 of numerous sinus infections and episodes of cellulitis, with previous cultures isolating Staphylococcus
9 aureus and Pseudornonas aerugirrosa. His mother also notes that his umbilical cord did not fall off until 5
• 10 weeks after birth_ On physical examination, the child has a severe, necrotic periodontal infection with
• 11 ulceration.
• 12
• 13 Laboratory results are as follows:
Complete blood count
• 15 Hemoglobin 11.8 gfdL
• 16 Hematocrit 36%
• 17 Platelets 24G,COGipL
• 18 Leukocyte count 36,DDD!pL
• 19 Neutrophils 90%
• 20 Lymphocytes 10%
• 21 Monocytes 2%
• 22
• 23
Nitroblue tetrazolium test is normal.
• 24
• 25 Which of the following defects is most likely present in this patient?
• 26
• 27
A. Adenosine deaminase deficiency [4%]
• 28
• 29 B. Complement deficiency [3%]
• 30 C. Defective formation of mature 13 lymphocytes [5%]
• 31
D. Defective formation of mature T lymphocytes [2%]
• 32
• 33 E Defective intracellular killing [6%]
• 34 • Development defect of the fourth pharyngeal pouch [1%]
• 35
G. Impaired leukocyte adhesion [16%]
• 36
• 37 H. Opsonization defect [3%]
• 38
• 39
• 40
Explanation: V

• 41
• 42 End Block
Feedback
1
Item: 14 of 44 -461
II 7 Mark
3 Previous Next Lab Values, Notes Calculator.
A

Manifestations of
6
▪ 7 leukocyte adhesion deficiency type I
• 8
• 9
Clinical
- Absence of bus formation at sites of infection
- Delayed umbilical cord separation (> 30 days)
- Poor wound healing

• 15
- Recurrent skin & mucosa) bacterial infections
• 16 ▪ Periodontitis, often necrotizing
• 17
• 18
• 19 Laboratory
• 20 ▪ Leukocytosis with neutrophil predominance
• 21
- Cultures often show S oureus & Gram-negative bacilli
• 22
• 23 - Biopsy of infected tissue shows inflammatory infiltrate
• 24 devoid of neutrophils

USPALEWor Id, LLC

Fl.

• 25
27
• 28 This patient with recurrent bacterial infections. delayed umbilical cord separation. and a necrotic periodontal
• 29 infection is most likely suffering from leukocyte adhesion deficiency type I. This syndrome is caused by
• 30 deficient expression of CD18, an essential component of certain integrins present on the surface of
.31 leukocytes. It is characterized by the inability of leukocytes (particular neutrophils) to exit the vasculature and
• 32 migrate to areas of infection or inflammation. This results in leukocytosis with a neutrophil predominance and
.33 the complete absence of neutrophils in inflamed or infected tissues.
• 34
• (Choices A, C, and D) Adenosine deaminase deficiency is an autosomal recessive form of severe combined
i 36 immunodeficiency that leads to deficient formation of mature B and T lymphocytes due to toxic accumulation
• of adenosine_ Most patients present with life-threatening infections. failure to thrive. and persistent diarrhea.
• 38 Laboratory studies show severe lymphopenia.
• 39
• 40 (Choice 13) Patients with complement deficiencies are at increased risk of bacterial infections, particularly v

• 41
• 42
Feedback, End Block .
• 1
• 2 Item: 14 of 44 rMark
▪ 3 Previous Next Lab Values. Notes Calculator.
• 4- devoid of neutrophils A

▪ 5
▪ 6 LISMLEftrIel, L I C.

▪ 7
▪ 8 This patient with recurrent bacterial infections, delayed umbilical cord separation, and a necrotic periodontal
▪ 9 infection is most likely suffering from leukocyte adhesion deficiency type I. This syndrome is caused by
• 10 deficient expression of CD18, an essential component of certain integrins present on the surface of
• 11 leukocytes. It is characterized by the inability of leukocytes (particular neutrophils) to exit the vasculature and
• 12 migrate to areas of infection or inflammation. This results in leukocytosis with a neutrophil predominance and
• 13 the complete absence of neutrophils in inflamed or infected tissues_

• 15 (Choices A, C, and D) Adenosine deaminase deficiency is an autosomal recessive form of severe combined
• 16 immunodeficiency that leads to deficient formation of mature 13 and T lymphocytes due to toxic accumulation
• 17 of adenosine_ Most patients present with life-threatening infections, failure to thrive, and persistent diarrhea.
• 18 Laboratory studies show severe lymphopenia_
• 19
• 20
(Choice B) Patients with complement deficiencies are at increased risk of bacterial infections, particularly
• 21
with encapsulated bacteria such as Streptococcus pneumoniae, Haernophilus influenza, and Neisseria
• 22
meningitides_ They are also at risk for autoimmune conditions.
• 23
(Choice E) Chronic granulomatous disease is characterized by defective intracellular killing due to impaired
• 24
respiratory burst from activated phagocytes. In this condition, the classic nitroblue tetrazolium test is negative
• 25
(abnormal). The dihydrorhodamine 123 test is more sensitive and can quantify the severity of illness.
• 26
• 27
(Choice F) Developmental defects in the pharyngeal arch system (including the fourth pharyngeal pouch) are
• 28
seen in DiGeorge syndrome, which is characterized by T-cell deficiency_ DiGeorge syndrome typically has
• 29
other associated physical examination findings, including facial and cardiac anomalies along with thymic
• 30
hypoplasia.
• 31
• 32 (Choice H) Opsonization defects can result from asplenia, as the spleen plays a crucial role in the
• 33 production of opsonizing antibody, which is required for the clearance of encapsulated organisms. This
• 34 patient has no history of infection by encapsulated organisms.
• 35
• 36 Educational objective:
• 37 Patients with leukocyte adhesion defect type 1 suffer from delayed separation of the umbilical cord, recurrent
• 38 bacterial infections of the skin and mucosal surfaces, and necrotic periodontal infections_ Additional findings
• 39 include leukocytosis with a neutrophil predominance and the complete absence of neutrophils in inflamed or
• 40 infp.nte.ri tissups
• 41
• 42 End Block
Feedback
• 1
• 2 Item: 14 of 44 V-Mark
3 Previous Next Lab Values. Notes Calculator.
II 1 II 1 IUI iuuciit ici iti LI ILL icrau.3 Lu uciit.ici IL iui I I IULIUI I 1...11 I I !ULU! c 1_7 ai IU I iyi i !pi iuti. JLc 3 uuu w LUP!.11., at.t.ui I lUIULIUI I
• 4-
of adenosine. Most patients present with life-threatening infections, failure to thrive, and persistent diarrhea.
5
Laboratory studies show severe lymphopenia_
6
7
(Choice B) Patients with complement deficiencies are at increased risk of bacterial infections, particularly
8
with encapsulated bacteria such as Streptococcus pneurnomae, Haernophifus influenza, and Neisseria
9
meningitides_ They are also at risk for autoimmune conditions.
• 10
• 11 (Choice El Chronic granulomatous disease is characterized by defective intracellular killing due to impaired
• 12 respiratory burst from activated phagocytes. In this condition, the classic nitroblue tetrazolium test is negative
• 13 (abnormal). The dihydrorhodamine 123 test is more sensitive and can quantify the severity of illness.

• 15 (Choice F) Developmental defects in the pharyngeal arch system (including the fourth pharyngeal pouch) are
• 16 seen in DiGeorge syndrome, which is characterized by T-cell deficiency DiGeorge syndrome typically has
• 17 other associated physical examination findings, including facial and cardiac anomalies along with thymic
• 18 hypoplasia.
• 19
• 20 (Choice H) Opsonization defects can result from asplenia, as the spleen plays a crucial role in the
• 21 production of opsonizing antibody. which is required for the clearance of encapsulated organisms. This
• 22 patient has no history of infection by encapsulated organisms.
• 23
• 24 Educational objective:
• 25 Patients with leukocyte adhesion defect type 1 suffer from delayed separation of the umbilical cord, recurrent
• 26 bacterial infections of the skin and mucosal surfaces, and necrotic periodontal infections_ Additional findings
• 27 include leukocytosis with a neutrophil predominance and the complete absence of neutrophils in inflamed or
• 28 infected tissues.
• 29
• 30 References:
• 31
1. Hematologically important mutations: leukocyte adhesion deficiency.
• 32
• 33 2. Periodontal manifestation of leukocyte adhesion deficiency type I.
• 34 3. Leukocyte adhesion deficiency type 1: an important consideration in the clinical
• 35 differential diagnosis of prepubertal periodontitis. A case report and review of the
• 36 literature.
• 37
• 38
• 39
Copyright © UWorld Last updated: [8/10/2014]
• 40
• 41
• 42 End Block
Feedback
• 1 I
• 2 Item: 15 of 44 F' Mark -4(1

▪ 3 Previous Next Lab Values. Notes Calculator.


• 4-
▪ 5
▪ 6 A 33-minute-old boy is in the neonatal intensive care unit with central cyanosis_ He was born at 28 weeks
▪ 7 gestation by vaginal delivery to a 16-year-old girl. The pregnancy was complicated by premature labor, and
▪ 8 the mother received glucocorticoids and magnesium sulfate shortly after the onset of labor. Membranes
▪ 9 ruptured spontaneously 6 hours prior to delivery; amniotic fluid was clear. The boy initially emerged vigorous
• 10 with spontaneous respirations but rapidly developed respiratory distress_ Birth weight was 1 kg (2 lb 4 oz)_
• 11 His temperature is 36 C (96.8 F) under radiant heat, blood pressure is 65/35 mm Hg, pulse is 143imin, and
• 12 respirations are 70/min. Pulse oximetry showed 75% on room air and improves to 90% with 35% FiO, by
• 13
continuous positive air pressure ventilation. Physical examination shows grunting, intercostal and subcostal
• 14
retractions, nasal flaring, and shallow respirations. Bilateral breath sounds are diminished. A chest x-ray
shows diffuse, fine, reticular granularity and perihilar linear opacities bilaterally. What is the most likely
• 16
diagnosis in this patient?
• 17
• 18
• 19 A. Congenital diaphragmatic hernia [0%]
• 20 B. ID-transposition of the great arteries [3%]
• 21
C. Meconium aspiration syndrome [3%]
• 22
• 23 ID. Persistent pulmonary hypertension [3%]
• 24 • E Respiratory distress syndrome [88%]
• 25
F. Transient tachypnea of the newborn [3%]
• 26
• 27
• 28 Explanation:
• 29
• 30
• 31
I Common causes of neonatal respirator), distress
• 32
• 33
Diagnosis Pathophysiology Clinical features
• 34
• 35

• 36 Inadequate alveolar • Tachyphea that begins shortly after
Transient birth & resolves by day 2 of life
• 37 fluid clearance at birth
tachypnea of
• 38 results in mild • Chest x-ray: Bilateral perihilar linear
• 39 the newborn
pulmonary edema 1 streaking
• 40
• 41
• 42
■ Feedback End Block


2 Item: 15 of 44 11 r.Mark
3 Previous Next Lab Values. Notes Calculator.
4 1
5
6 7 Common causes of neonatal respiratory distress

8 1
Diagnosis Pathophysiology Clinical features
9
• 10 • Tachypnea that begins shortly after
Inadequate alveolar
• 11 Transient birth & resolves by day 2 of life
fluid clearance at birth
• 12 tachypnea of
13
•-h1= results in mild • Chest x-ray: Bilateral perihilar linear
the newborn
• 14 pulmonary edema streaking

• 16
• Severe respiratory distress &
• 17
cyanosis after premature birth
• 18
• 19 Surfactant deficiency • Chest x-ray:
Respiratory
• 20 results in alveolar • Diffuse, reticulogranular,
• 21
distress
collapse & diffuse ground-glass appearance
• 22 syndrome
atelectasis
• 23 • Air bronchograms
• 24
• 25 • Low lung volumes
• 26
• 27 High pulmonary • Tachypnea & severe cyanosis
• 28 Persistent
vascular resistance
• 29 pulmonary • Chest x-ray: Clear lungs with
results in right-to-left
• 30 hypertension decreased pulmonary vascularity
shunting & hypoxia
• 31
• 32 OUSAiLEWorkl,LLC
• 33
• 34 Respiratory distress syndrome (RDS), formerly known as hyaline membrane disease, is a common problem
• 35 in premature neonates and very low birth weight (<1500 g [3.3 Ib]) infants. The incidence is inversely
• 36 related to gestational age, and it occurs in almost all extremely preterm (28 weeks) neonates. The primary
• 37 problem is surfactant deficiency; the immature alveoli are unable to produce sufficient surfactant to reduce
• 33 1 alveolar surface tension. Affected neonates develop the following symptoms within minutes to hours after
• 39 birth in an attempt to compensate for diffuse alveolar collapse:
• 40 se
• 41
• 42 End Block
Feedback
• 1
• 2 Item: 15 of 44 F' Mark
▪ 3 Previous Next Lab Values. Notes Calculator.

• 4- Respiratory distress syndrome (I-WS), tormerly known as hyaline membrane disease, is a common problem A
▪ 5 in premature neonates and very low birth weight (<1500 g [3.3 Ib]) infants. The incidence is inversely
▪ 6 related to gestational age, and it occurs in almost all extremely preterm (28 weeks) neonates. The primary
▪ 7 problem is surfactant deficiency; the immature alveoli are unable to produce sufficient surfactant to reduce
▪ 8 alveolar surface tension. Affected neonates develop the following symptoms within minutes to hours after
▪ 9 birth in an attempt to compensate for diffuse alveolar collapse:
• 10
• Tachypnea (respiratory rate >601min)
• 11
• Grunting (to increase end-expiratory pressure)
• 12
• Nasal flaring (decreases nasal airway resistance)
• 13
• Retractions (intercostal muscles contract and pull in the compliant chest wall)
• 14
• Hypoxia and cyanosis (reflects significant atelectasis)
• 16
• 17 This patient has the classic x-ray findings of RDS. The risk of RDS can be reduced by administration of
• 18 maternal antenatal glucocorticoids, which stimulate fetal surfactant synthesis and secretion_ Neonatal
• 19 treatment includes early continuous positive air pressure ventilation_ Intubation, mechanical ventilation,
• 20 and exogenous surfactant therapy are reserved for severe cases.
• 21
• 22
(Choice A) Classic x-ray findings of congenital diaphragmatic hernia include loops of bowel in the
• 23
hemithorax, a displaced cardiac silhouette, and a gasless abdomen. Patients with congenital diaphragmatic
• 24
hernia would worsen with noninvasive positive-pressure ventilation as air pumped into the gastrointestinal
• 25
tract can compress the lungs and should instead be intubated_
• 26
(Choice B) D-transposition of the great arteries can cause cyanosis due to systemic circulation of
• 27
deoxygenated blood. Newborns are usually tachypneic without other signs of respiratory
• 28
distress. Administration of supplemental oxygen would not improve oxygen saturation_
• 29
• 30
(Choice C) Meconium aspiration syndrome occurs in term or post-term infants born through meconium-
• 31
stained fluid. Meconium obstructs the airways and causes respiratory distress. X-ray would show patchy
• 32
infiltrates, coarse streaking of both lung fields, and flattening of the diaphragm. This patient's history of
• 33
prematurity and clear amniotic fluid makes this diagnosis unlikely.
• 34
• 35 (Choice 0) Persistent pulmonary hypertension of the newborn should be suspected in all term and post-term
• 36 neonates with cyanosis. High pulmonary vascular resistance results in right-to-left shunting of deoxygenated
• 37 blood through the foramen ovale and ductus arteriosus, resulting in hypoxia. The diagnosis is rare in very low
• 38 birth weight infants. In addition, the x-ray findings in this patient are very typical of RDS.
• 39
• 40 irrhnirta F1 TrAnqiinnt tArhunnin2 is thin mnqt rnmmnn rai icr3 of rinqnirAtnrur r-litrinqq in fiill_tprm infant nil
• 41
• 42 End Block
Feedback
• 1
• 2 Item: 15 of 44 V-• Mark -<=1

▪ 3 Previous Next Lab Values. Notes Calculator.


11.6011LPII,W PL) Uy III IUII 14. J UI .6,6011WW1111.1:11 1.11C114111101111;11.1., 11=1111CI II 14.IUUG IUUl.JJ UI L.IIJYY1 III LI IG
• 4 A
hemithorax, a displaced cardiac silhouette, and a gasless abdomen. Patients with congenital diaphragmatic
▪ 5
▪ 6 7 hernia would worsen with noninvasive positive-pressure ventilation as air pumped into the gastrointestinal
tract can compress the lungs and should instead be intubated_
▪ 7
▪ 8
(Choice B) 0-transposition of the great arteries can cause cyanosis due to systemic circulation of
▪ 9
deoxygenated blood. Newborns are usually tachypneic without other signs of respiratory
• 10
distress. Administration of supplemental oxygen would not improve oxygen saturation_
• 11
• 12 (Choice C) Meconium aspiration syndrome occurs in term or post-term infants born through meconium-
• 13 stained fluid. Meconium obstructs the airways and causes respiratory distress. X-ray would show patchy
• 14 infiltrates, coarse streaking of both lung fields, and flattening of the diaphragm. This patient's history of
prematurity and clear amniotic fluid makes this diagnosis unlikely.
• 16
• 17 (Choice 0) Persistent pulmonary hypertension of the newborn should be suspected in all term and post-term
• 18 neonates with cyanosis. High pulmonary vascular resistance results in right-to-left shunting of deoxygenated
• 19 blood through the foramen ovale and ductus arteriosus, resulting in hypoxia. The diagnosis is rare in very low
• 20 birth weight infants. In addition, the x-ray findings in this patient are very typical of RDS.
• 21
• 22 (Choice F) Transient tachypnea is the most common cause of respiratory distress in full-term infants and
• 23 has no long-term sequelae. Respiratory distress in the setting of surfactant deficiency, prematurity, and
• 24 diffuse reticular granularities on chest x-ray rules out benign transient tachypnea.
• 25
• 26 Educational objective:
• 27 Respiratory distress syndrome should be suspected when a premature infant presents with grunting. flaring,
• 28 and retractions immediately after birth_ Chest-x ray includes characteristic fine reticular granularity of the
• 29 lungs_ Treatment includes early continuous positive air pressure ventilation_
• 30
• 31 References:
• 32
1. Respiratory distress in the newborn.
• 33
• 34 2. The role of surfactant treatment in preterm infants and term newborns with acute
• 35 respiratory distress syndrome.
• 36 3. Respiratory support in preterm infants at birth.
• 37
▪ 38
• 39
Copyright © LIWorld Last updated: [8/8/2014]
• 40 Ii
• 41
• 42 End Block
Feedback
Item: 15 of 44
3 Previous Next Lab Values Notes Calculator
4- Media Exhibit
5
6 eonaiml respiratory distress syndrome
7
8
9
• 10
• 11
• 12
• 13
• 14
Endotracheal,
tube
• 16
• 17
• 18
• 19
$111111111116"
_;
• 20
• 21
• 22
• 23
• 24
round-glass"
• 25 opacities Air
• 26 bronchograms
• 27
• 28
• 29
• 30
• 31
• 32
• 33
• 34
• 35
Orogastric
• 36 tube
• 37 V
• 33
• 39
• 40
• 41
• 42 End Block
Feedback
• -1
• 2 Item: 15 of 44 .11M Mark
▪ 3 Previous Next Lab Values Notes Calculator
• 4- Media Exhibit
▪ 5
▪ 6 Congenital diaphragmatic hernia
▪ 7
▪ 8
▪ 9
• 10
• 11
• 12
• 13
• 14

• 16
• 17
• 18
• 19
• 20
• 21
• 22
• 23
• 24
• 25
• 26
• 27
• 28
• 29
• 30
• 31
• 32
• 33
• 34 Gasless abdomen
• 35
• 36
• 37 V
• 38
• 39
• 40
• 41
• 42
1
2 Item: 16 of 44 'MY-Mark
3 Previous Next Lab Values. Notes Calculator.
4
5
6 7 An 18-month-old male is brought to the hospital because of fever, dyspnea, and productive cough of two days
7 duration. His mother reports that he just recovered from prolonged diarrhea due to Garda infection_ His past
8 1 medical history is also significant for pneumonia and recurrent ear infections since 6 months of age. On
9 physical examination, his temperature is 383' C (1G1 F), pulse is 1.4Gimin, and respirations are 40/min.
• 10 Examination reveals a young child in mild respiratory distress and bronchial breath sounds in the right lower
• 11 lung lobe_ Which of the following is the most likely cause of his recurrent infections?
• 12
r
i 13
=
• 14
A. Impaired oxidative metabolism [11%]
• 15 13. Complement deficiency [9%]
C. Thymic hypoplasia [7%]
• 17
• 18 D. Adenosine deaminase deficiency [21%]
• 19 E. Abnormal B-lymphocyte maturation [5156]
• 20
• 21
• 22 Explanation:
• 23
This infant has a history of Garda and recurrent sinopulmonary infections since six months of age,
• 24
suggesting a genetic defect in 0-lymphocyte maturation (Choice E). Genetic 0-cell deficiencies present after
• 25
6 months of age because this is when the newborn's levels of maternal antibodies drop. Affected infants are
• 26
predisposed to recurrent sinopulmonary infections with encapsulated organisms like H. influenzae and S.
• 27
pneurnorriae because immune destruction of encapsulated organisms depends upon the humoral immune
• 28
response. Deficiency in IgA, which inhabits the GI tract, is the specific deficiency that predisposes to Garda
• 29
infection.
• 30
• 31
(Choice A) Chronic granulomatous disease (CGD) is a condition of impaired oxidative metabolism. In CGD,
• 32
a defect in the NADPH-oxidase system of phagocytic cells results in defective intracellular killing_ Patients are
• 33
therefore prone to abscesses secondary to catalase-producing organisms like Aspergdfus and
• 34
Staphylococcus.
• 35
• 36 (Choice B) Gonococcal and meningococcal infections are common in patients with complement
• 37 deficiency. S. pneurnoniae and H. influenzae infections do occur in this patient population as well, but
• 38 infections with Giardia are not typically seen_
• 39
• 40 nna clamant of nir4artrria c-trnrIrr.rna Tina re.c.1111-1.-.1- Trail rlafirianrw

• 41
• 42 End Block
Feedback
• 1
• 2 Item: 16 of 44 V-Mark
▪ 3 Previous Next Lab Values. Notes Calculator.
• 4-
ID. Adenosine deaminase deficiency [2116]
▪ 5
wI E Abnormal B-lymphocyte maturation [51%]
▪ 6
▪ 7
▪ 8
Explanation:
▪ 9
• 10 This infant has a history of Garda and recurrent sinopulmonary infections since six months of age,
• 11 suggesting a genetic defect in B-lymphocyte maturation (Choice Ey Genetic B-cell deficiencies present after
• 12 6 months of age because this is when the newborn's levels of maternal antibodies drop. Affected infants are
• 13 predisposed to recurrent sinopulmonary infections with encapsulated organisms like H. intluenzae and S.
• 14 pneurnomae because immune destruction of encapsulated organisms depends upon the humoral immune
• 15 response. Deficiency in IgA, which inhabits the GI tract, is the specific deficiency that predisposes to Garda
infection.
• 17
• 18 (Choice A) Chronic granulomatous disease (COD) is a condition of impaired oxidative metabolism. In COD,
• 19 a defect in the NADPH-oxidase system of phagocytic cells results in defective intracellular killing_ Patients are
• 20 therefore prone to abscesses secondary to catalase-producing organisms like Aspergllus and
• 21 Staphylococcus.
• 22
• 23 (Choice B) Gonococcal and meningococcal infections are common in patients with complement
• 24 deficiency. S. pneunioniae and H. ifitluenzae infections do occur in this patient population as well, but
• 25 infections with Guardia are not typically seen.
• 26
• 27 (Choice C) Thymic hypoplasia is one element of DiGeorge syndrome. The resultant T-cell deficiency puts
• 28 patients at high risk for viral and fungal infections.
• 29
(Choice ID) Adenosine deaminase deficiency is commonly the genetic defect underlying severe combined
• 30
immunodeficiency (SCID). Patients with SCID have a deficiency of both B- and T-cells_ The T-cell deficiency
• 31
predisposes to viral and fungal infections and the B-cell deficiency predisposes to infection by bacteria.
• 32
• 33
Educational objective:
• 34
Patients with genetic B-cell deficiencies begin to develop recurrent infections after passing 6 months of
• 35
age. The deficient humoral immune response in these patients impairs the body's ability to destroy
• 36
encapsulated organisms. Hence, recurrent sinopulmonary infections with H. influenzae and S. pneurnoniae
• 37
are common_ Lack of IgA also predisposes to Garda infection_
• 38
• 39
Copyright © LIWorld Last updated: [12/29/2014]
• 40
• 41
• 42 End Block
Feedback
1
2 Item: 17 of 44 V-Mark
3 Previous Next Lab Values. Notes Calculator.
4-
5
6
An 8-year-old girl is being evaluated for short stature. She is at 8th percentile for height and 30th percentile for
7
weight Vital signs are within normal limits. Examination shows a high arched palate and inverted, widely
8
spaced nipples. Karyotyping shows 45 X0_ Which of the following is she most at risk of developing?
9
• 10
• 11 A. Osteoporosis [&J%]
• 12 B. Mitral valve prolapse [22%]
• 13
C. Mental retardation [13%]
• 14
• 15 ID. Bipolar disorder [1%]
E Breast cancer [3%]

• 18
• 19 Explanation:
• 20
• 21 This patient's short height, high arched palate, widely spaced nipples, and 45 XO karyotype are all consistent
• 22 with a diagnosis of Turner syndrome_ Patients with Turner syndrome are prone to the development of
• 23 osteoporosis which increases the risk of bone fracture_ This increased risk is in part due to low estrogen
• 24 levels from gonadal dysgenesis_ There is also thought to be increased risk from having only one copy of X
• 25 chromosome genes that may be involved in bone metabolism_ Estrogen replacement therapy is given to
• 26 nearly all patients to promote normal maturation, but it also has the effect of reducing the risk of osteoporosis.
• 27
• 28 (Choice B) The most common cardiovascular abnormalities in Turner syndrome are coarctation of the aorta
• 29
and aortic valve abnormalities. Mitral valve prolapse is not very common.
• 30
(Choice C) Most patients with Turner syndrome have normal cognitive abilities.
• 31
• 32
(Choice 13) There is no higher risk for development of bipolar disorder in patients with Turner syndrome.
• 33
• 34 (Choice E) Patients with Turner syndrome do not have any additional risk of breast cancer_ Turner syndrome
• 35 patients have decreased estrogen levels, while elevated estrogen levels are a risk factor for breast cancer_
• 36
• 37 Educational objective:
• 38 Short height, high arched palate, widely spaced nipples, and 45 XO karyotype are all features of Turner
• 39 syndrome. Patients with Turner syndrome have a higher risk of osteoporosis due to lower estrogen levels
• 40 and nnlv hmiinn nnp rnnv of rhrnmnsnmp npnps invnlvprl in hemp nlethnlisrn se
• 41
• 42 End Block
Feedback
• 1
• 2 Item: 17 of 44 .11V-• Mark
3 Previous Next Lab Values. Notes Calculator.
4- An 8-year-old girl is being evaluated for short stature. She is at 8 — percentile for height and 30`" percentile for
5 weight Vital signs are within normal limits. Examination shows a high arched palate and inverted, widely
6 spaced nipples. Karyotyping shows 45 X0_ Which of the following is she most at risk of developing?
7
8
A. Osteoporosis [60%]
9
• 10
B. Mitral valve prolapse [22%]
• 11 C. Mental retardation [13%]
• 12
ID. Bipolar disorder [1°,.0]
• 13
• 14
E Breast cancer [3%]
• 15

Explanation:

• 18 This patient's short height, high arched palate, widely spaced nipples, and 45 XO karyotype are all consistent
• 19 with a diagnosis of Turner syndrome_ Patients with Turner syndrome are prone to the development of
• 20 osteoporosis which increases the risk of bone fracture_ This increased risk is in part due to low estrogen
• 21 levels from gonadal dysgenesis_ There is also thought to be increased risk from having only one copy of X
• 22 chromosome genes that may be involved in bone metabolism. Estrogen replacement therapy is given to
• 23 nearly all patients to promote normal maturation, but it also has the effect of reducing the risk of osteoporosis.
• 24
• 25 (Choice B) The most common cardiovascular abnormalities in Turner syndrome are coarctation of the aorta
• 26 and aortic valve abnormalities. Mitral valve prolapse is not very common.
• 27
• 28 (Choice C) Most patients with Turner syndrome have normal cognitive abilities.
• 29
• 30 (Choice ID) There is no higher risk for development of bipolar disorder in patients with Turner syndrome.
• 31
(Choice E) Patients with Turner syndrome do not have any additional risk of breast cancer. Turner syndrome
• 32
patients have decreased estrogen levels, while elevated estrogen levels are a risk factor for breast cancer_
• 33
• 34
Educational objective:
• 35
Short height, high arched palate, widely spaced nipples, and 45 XO karyotype are all features of Turner
• 36
syndrome. Patients with Turner syndrome have a higher risk of osteoporosis due to lower estrogen levels
• 37
and only having one copy of X chromosome genes involved in bone metabolism.
• 38
• 39
Copyright © UWorld Last updated: [8/22/2014]
• 40
• 41
• 42 End Block
Feedback
3 =1
1
4
Item: 18 of 44 F' Mark
Previous Next Lab Values. Notes Calculator.

5
6 7 A 5-year-old boy is brought to the emergency department by his father with fatigue and scrotal swelling. His
7 father noticed the swelling today and is sure that 'it wasn't there yesterday.'" The boy had a low-grade fever
8 and cough 2 weeks ago and 'hasn't been himself ever since." He has no significant past medical history_ On
examination, the patient looks ill and lethargic. Mucous membranes are dry and his capillary refill is 3
• io seconds. He has a palpable nonblanching rash on his buttocks and lower legs. The right hemi-scrotum is
• ii slightly swollen and tender to palpation. Abdominal examination shows mild diffuse tenderness to palpation
• 12 without rebound or guarding_ Laboratory results from urinalysis are as follows:
. 1=3
Specific gravity 1.016
• u
pH 7.8
• -15
Protein +1
• 16
Blood Moderate
• 17
Glucose Negative
Ketones Negative
• 19
Leukocyte esterase Negative
• 20
Nitrites Negative
• 21
• 22
The boy is admitted to the hospital for further management Which of the following adverse outcomes is he
• 23
most likely to develop?
• 24
• 25
• 26 A. Appendicitis [694)]
• 27 B. Cholecystitis [81C]
• 28
C. Diverticulitis [39C]
• 29
• 30 ID. Intussusception [66%]
• 31 E Meckel diverticulum [3%]
• 32
F. Volvulus [15%]
• 33
• 34
• 35
Explanation:
• 36
• 37
Henoch-Schonlein purpura
• 38
• 39
V
• 40
• 41
• 42 End Block
Feedback
1
-

• 2 Item: 18 of 44 F'Mark
3 Previous Next Lab Values. Notes Calculator.
• 4- A
5
Fienoch-Schonlein purpura
6
7
8 • Immune-mediated leukocytoclastic vasculitis
9 Pathogenesis
• Associated with IgA deposition in affected organs
• 10
• -1 -1
• 12 • Palpable purpura with normal platelet count &
• 13 coagulation studies
• 14
• Arthritisiarthralgia
• 15
• 16 Clinical • Abdominal pain
• 17 manifestations • Gastrointestinal bleeding
• Intussusception
• 19
• 20 • Renal disease similar to IgA nephropathy
• 21 • Scrotal pain & swelling
• 22
• 23
• 24
• Normal platelet count & coagulation studies
Laboratory
• 25 • Normal to elevated creatinine
• 26
findings
• Urinalysis: Hematuria, red cell casts & mild proteinuria
• 27
• 28
• 29 • Supportive management (hydration & pain control
• 30 with nonsteroidal anti-inflammatory drugs)
• 31
• Hospitalization indicated for severe abdominal pain,
• 32
• 33 renal insufficiency, inability to tolerate oral intake, or
Treatment
• 34 altered mental status
• 35
• Systemic glucocorticoids in patients with severe
• 36
• 37
abdominal pain unresponsive to nonsteroidal anti-
• 33 inflammatory drugs
• 39
• 40 uSIALVicw121.4K se
• 41
• 42 End Block
Feedback
• 1
• 2 Item: 18 of 44 Al Mark
▪ 3 Previous Next . Lab Values. Notes Calculator.
• 4- altered mental status A

▪ 5
• Systemic glucocorticoids in patients with severe
▪ 6
▪ 7
abdominal pain unresponsive to nonsteroidal anti-
▪ 8 inflammatory drugs
▪ 9
Q.) u$M,I,EWadkl, ci,C
• 10
• 11
• 12 Henoch-Schbnlein purpura (HSP) is an immune-mediated vasculitis of childhood that often occurs after mild
• 13 illnesses such as upper respiratory tract infections_ HSP is more common in boys and occurs more
• 14 frequently in the fall and winter months. Classic manifestations include abdominal pain, palpable purpura on
• 15 the lower extremities, arthralgias, and renal disease. In rare cases, patients can have scrotal pain and
• 16 swelling as the initial presenting symptoms_
• 17
18 Although the majority of patients with HSP develop abdominal pain, the presence of severe abdominal pain
• 19
should prompt further workup for gastrointestinal hemorrhage or intussusception, both of which are known
• 20
complications_ Intussusception, which occurs in up to 4% of cases, presents with severe episodic
• 21
abdominal pain and "currant jelly" or bloody stools. The increased risk for intussusception is due to bowel
• 22
wall edema and localized hemorrhage, which can act as lead-points for the intussusception.
• 23
Unlike most cases of intussusception in children, which are ileo-colic, intussusceptions in HSP are more likely
• 24
to be small-bowel or ileo-ileal (60% of cases). Because of their location, small-bowel intussusceptions cannot
• 25
be seen on contrast enema and are diagnosed by the presence of a "target" sign on ultrasound. Ileocolic
• 26
intussusceptions can be treated with air or contrast enema, but ileo-ileal intussusceptions that do not reduce
• 27
spontaneously often require surgical management
• 28
• 29
(Choice A) Appendicitis is no more common in HP than in normal children. Appendicitis typically presents
• 30
with fever, anorexia, vomiting, and right lower-quadrant pain_
• 31
• 32 (Choice B) Cholecystitis is rare in otherwise healthy children and is not associated with HS13. Risk factors
• 33 for cholecystitis in children include sickle cell anemia, hereditary spherocytosis, and obesity.
• 34
• 35 (Choice C) Diverticulitis, which is caused by inflammation of colonic divertula, is exceedingly rare in children
• 36 and typically presents with left lower-quadrant pain_ An increased incidence of colonic diverticula and
• 37 abdominal walliinguinal hernias is seen with autosomal dominant polycystic kidney disease.
▪ 38
• 39 (Choice El Meckel diverticulum usually presents with painless rectal bleeding in young children_ It is not
• 40 associated with HSP. The presence of Meckel diverticulum, however, is associated with an increased risk of
• 41
• 42 End Block
Feedback
1
2. Item: 18 of 44 V-Mark
3 Previous Next Lab Values. Notes Calculator.
4 I oe seen on contrast enema and are diagnosed oy me presence or a - target sign on uitrasouno_ iieocoiic
5 intussusceptions can be treated with air or contrast enema, but ileo-ileal intussusceptions that do not reduce
6 7 spontaneously often require surgical management
7
(Choice A) Appendicitis is no more common in HP than in normal children_ Appendicitis typically presents
8
with fever, anorexia, vomiting, and right lower-quadrant pain_
9
• 10
(Choice B) Cholecystitis is rare in otherwise healthy children and is not associated with HSF. Risk factors
• 11
for cholecystitis in children include sickle cell anemia, hereditary spherocytosis, and obesity_
• 12
1113
1=
(Choice C) Diverticulitis, which is caused by inflammation of colonic divertula, is exceedingly rare in children
• 14 and typically presents with left lower-quadrant pain_ An increased incidence of colonic diverticula and
• 15 abdominal wall/inguinal hernias is seen with autosomal dominant polycystic kidney disease.
• 16
• 17 (Choice El Pdeckel diverticulum usually presents with painless rectal bleeding in young children. It is not
i associated with HSP. The presence of Meckel diverticulum, however, is associated with an increased risk of
• 19 recurrent intussusception.
•• 20
• 21 (Choice F) Malrotation with midgut volvulus is a very common cause of intestinal obstruction in the neonatal
• 22 period, and it can also occur later in childhood or adolescence. Over 90% of patients with midgut volvulus
• 23 present with vomiting (often bilious). Volvulus is not a recognized complication of HSP.
• 24
• 25 Educational objective:
• 26 Henoch-Schdrilein purpura (HSP) is an immune-mediated vasculitis that is most common in children age
• 27 <15. Typical features include palpable purpura, hematuria, abdominal pain. arthralgias and occasionally
• 28 scrotal swelling_ Children with HSP are at increased risk for ileo-ileal intussusception.
• 29
• 30 References:
• 31
1. Intra-abdominal manifestations of Henoch-Schbnlein purpura.
• 32
• 33 2. Henoch Schonlein purpura in childhood: epidemiological and clinical analysis of 150
• 34 cases over a 5-year period and review of literature.
• 35
3. Henoch Schtinlein purpura in childhood: clinical analysis of 254 cases over a 3-year
• 36
period.
• 37
• 38
• 39
Copyright © LIWorld Last updated: [12/31/2014]
• 40
• 41
• 42 End Block
Feedback
• -1
• 2 Item: 18 of 44 Al I - Mark
V"'

▪ 3 Previous Next Lab Values Notes Calculator.


• 4- Media Exhibit
▪ 5
▪ 6 Bloody stools in intussusizeption
▪ 7
▪ 8
▪ 9
• 10
• -1 -1
• 12
• 13
• 14
• 15
• 16
• 17

• 19
• 20
• 21
• 22
• 23
• 24
• 25
• 26
• 27
• 28
• 29
• 30
• 31
• 32
• 33
• 34
• 35
• 36

=Pr
• 37
• 33
• 39
• 40
• 41
• 42 End Block
Feedback
-1

2 Item: 18 of 44 'Mark
3 Previous Next Lab Values Notes Calculator
4- Media Exhibit X
5
6 neoaolic intussusception
7
8
9
• 10
• -1-1
• 12
13
11=
• 14
• 15
• 16
• 17

• 19
• 20
• 21
• 22
• 23
• 24
• 25
• 26
• 27
• 28
• 29
• 30
• 31
• 32
• 33
• 34
• 35
• 36
• 37
• 33
• 39
• 40


41
42
A
Feedback
a
End Block
• 1
• 2 Item: 19 of 44 V-• Mark -4(1

▪ 3 Previous Next Lab Values. Notes Calculator.


• 4-
▪ 5
▪ 6 A 3-year-old African American boy with sickle cell anemia is brought to the emergency department for fever.
▪ 7 He is exhausted and has chills and night sweats. However, he has had no bone pain, cough, or difficulty
▪ 8 breathing. The boy was hospitalized during infancy for dactylitis and last summer for splenic sequestration
▪ 9 crisis_ Other than ibuprofen and acetaminophen for fever and pain, he takes no medications_ His
• 10 vaccinations are up to date. His temperature is 39A C (1G3 F), blood pressure is 1814G mm Hg, pulse is
• 11 140/min, and respirations are 22/min. Examination shows a lethargic boy with mild pallor. Laboratory results
• 12 are as follows:
• 13
Complete blood count
• 14
Hemoglobin 8.2 WI:IL
• 15
Hematocrit 24%
• 16
Platelets 325,000/pL
• 17
Leukocytes 18,800/pL
• 18
Neutrophils 8G%
Band 6%
• 20
Gram stain Pending
• 21
Blood culture Pending
• 22
• 23
Which of the following organisms is the most likely cause of this patient's condition?
• 24
• 25
• 26 A. Escherichia coIi [3%]
• 27 Haemophilus intluenzae type 13 [5%]
• 28
C. Neisseria meningitides [7%]
• 29
• 30 D. Pseudomonas aeruginosa [3%]
• 31 E Salmonella enteritidis [16%]
• 32
E Staphylococcus aureus [13%]
• 33
• 34 G. Streptococcus pneumoniae [53%]
• 35
• 36
Explanation:
• 37
• 30
• 39
• 40
Sickle-cell anemia se
• 41
• 42 End Block
Feedback
• 1
• 2 : 19 of 44 r Flilark
• 3 Previous . Next . Lab Values, Notes Calculator.
• 4
• 5 Sickle-cell anemia
• 6
• 7
Inheritance Autosomal recessive
• 8
0
• 10 • Hemolytic anemia
Clinical
11
presentation • Acute vasoocclusive pain crises
• 12
13
• 14
• NI, Hematocrit, '1k reticulocytes, lk lactate dehydrogenase,
• 15 Laboratory \ unconjugated bilirubin
• 16 'I findings
• 17 • Peripheral smear: Sickled red cells, Howell-Jolly bodies
• 18 I

• 20 • Vaccination
• 21 • Penicillin (until age 5)
• 22
Maintenance • Folic acid supplementation
• 23
• 24 • Hydroxyurea (for patients with
• 25 Management recurrent vasooccrusive events)
• 26
• '7
Z,

• 28 • Hydration
• 29
Acute pain crises • Analgesia
• 30
• 31 • Transfusion
• 32
• 33
• 34 • Infection with encapsulated organisms

• Streptococcal pneumoniae bacterem ia & pneumonia
• 36
• Salmonella osteomyelitis

• 38 • Osteonecrosis
• 39
Complications • Acute chest syndrome
• 40
• 41
• 42 End Block,
Feedback
• 1
• 2 Item: 19 of 44 V/lark
▪ 3 Previous Next Lab Values. Notes Calculator.
• 4 • Pnapisrn
▪ 5
• Stroke
▪ 6
▪ 7 ODUSPALEWorktr U_C
▪ 8
▪ 9 The primary problem in homozygous sickle cell disease is constant clumping of sickle cells, which leads to
• 10 repeated microinfarctions in any part of the body, but particularly the spleen. The slow blood flow in the spleen
• 11 permits splenic sequestration of red blood cells, resulting in recurrent splenic infarction and splenic
• 12 dysfunction_ The spleen is the site of removal for encapsulated organisms, and patients with functional
• 13 asplenia are at extremely high risk of overwhelming infection with Streptococcus pneurnoniae
• 14 (pneumococcus), Haemophilus influenzae type E, Neisseria meningitides, and Salmonella species.
• 15
• 16 This patient's presentation (eg, high fever, hypotension, lethargy, neutrophil-predominant leukocytosis with
• 17 bandemia) is alarming for sepsis_ S pneumoniae is the most common cause of bacteremia, with H
• 18 influenzae type B (Choice B) as a distant second. All patients with sickle cell anemia should receive routine
pediatric vaccinations (eg, 13-valent conjugate pneumococcus and H influenzae type B vaccines) plus the
• 20 23-valent polysaccharide pneumococcus and meningococcal conjugate vaccines. The incidence of
• 21 bacteremia has decreased dramatically with widespread vaccination, but children with sickle cell disease
• 22 continue to be at serious risk of invasive pneumococcal disease from other pneumococcal serotypes.
• 23 Therefore, penicillin prophylaxis should also be given until at least age 5 years. Unfortunately, this patient
• 24 was not taking penicillin.
• 25
• 26 (Choices A and D) Given the patient's history of sickle cell anemia and likelihood of splenic dysfunction,
• 27 bacteremia due to pneumococcus is more likely than Escherichia coli or Pseudomonas aeruginosa_ E coli is
• 28 the most common cause of urinary tract infection in all patients, but this patient has no related symptoms (eg.
• 29 dysuria, hesitancy, abdominal pain). P aeruginosa is a common cause of bacteremia in patients with burn
• 30 wounds and neutropenia.
• 31
• 32
(Choice C) N meningitides is an encapsulated organism that is responsible for most cases of bacterial
• 33
meningitis in children and young adults in the United States. Infected patients can have fever, hypotension,
• 34
tachycardia, and drowsiness. The absence of headache, photophobia, and neck pain makes meningitis
• 35
unlikely in this patient
• 36
(Choices E and F) Salmonella species and Staphylococcus aureus are the most common causes of
• 37
osteomyelitis in sickle cell patients in the United States. However, this patient has no bone pain suggestive of
• 33
osteomyelitis.
• 39
• 40
se
• 41
• 42 End Block
Feedback
_ A
• 1
• 2 Item: 19 of 44 [MI V-Mark
▪ 3 Previous Next Lab Values. Notes Calculator.
r '
• 4-
bacteremia has decreased dramatically with widespread vaccination, but children with sickle cell disease
▪ 5
continue to be at serious risk of invasive pneumococcal disease from other pneumococcal serotypes.
▪ 6
Therefore, penicillin prophylaxis should also be given until at least age 5 years. Unfortunately, this patient
▪ 7
was not taking penicillin.
▪ 8
▪ 9 (Choices A and D) Given the patient's history of sickle cell anemia and likelihood of splenic dysfunction,
• 10 bacteremia due to pneumococcus is more likely than Escherichia coli or Pseudomocas aeruginosa_ E coli is
• 11 the most common cause of urinary tract infection in all patients, but this patient has no related symptoms (eg,
• 12 dysuria, hesitancy, abdominal pain}_ P aeruginosa is a common cause of bacteremia in patients with burn
• 13 wounds and neutropenia.
• 14
• 15 (Choice C) N meningitides is an encapsulated organism that is responsible for most cases of bacterial
• 16 meningitis in children and young adults in the United States. Infected patients can have fever, hypotension,
• 17 tachycardia, and drowsiness. The absence of headache, photophobia, and neck pain makes meningitis
• 18 unlikely in this patient

• 20 (Choices E and F) Salmonella species and Staphylococcus aureus are the most common causes of
• 21 osteomyelitis in sickle cell patients in the United States. However, this patient has no bone pain suggestive of
• 22 osteomyelitis.
• 23
• 24 Educational objective:
• 25 Sickle cell anemia is associated with functional asplenia due to recurrent splenic infarction, which puts these
• 26 patients at risk of dangerous infection with encapsulated organisms. Pneumococcus is the most common
• 27 cause of sepsis in this population. Vaccination and penicillin prophylaxis are extremely important infection
• 28 prevention measures.
• 29
• 30 References:
• 31
1. Sickle-cell disease.
• 32
• 33 2. Incidence of invasive pneumococcal disease among individuals with sickle cell disease
• 34 before and after the introduction of the pneumococcal conjugate vaccine.
• 35 3. Increase in invasive Streptococcus pneumoniae infections in children with sickle cell
• 36 disease since pneumococcal conjugate vaccine licensure.
• 37
• 30
• 39
Copyright © UWorld Last updated: [10/25/2014]
• 40
• 41
• 42 End Block
Feedback
• 1
• 2 Item: 20 of 44 V- Mark
▪ 3 Previous Next Lab Values. Notes Calculator.
• 4-
▪ 5
▪ 6 A previously healthy 5-year-old girl is brought to the emergency department with difficulty swallowing. For the
▪ 7 past week, she has had fever, chills, malaise, muscle aches, and sore throat She now refuses to drink fluids
▪ 8 and spits out any water that is given to her. The girl sustained a bite to her right forearm when she and her
▪ 9 family went camping in Delaware last month_ Family members are unsure of the bite source as it was dark.
• 10 The wound was rinsed with water and has since healed_ The girl takes no medications, has no allergies, and
• 11 is up to date on standard vaccinations_ There is no history of foreign travel_ Vital signs are normal.
• 12 Examination shows an agitated, disoriented girl with copious drool and facial grimacing. The neck has full
• 13 range of motion. A well-healed scar is seen on her right forearm_ Of the following disease reservoirs, which
• 14 is the most likely source of her symptoms?
• 15
• 16
• 17
A. Mosquito [1 °A]
• 18 Raccoon [79%]
• 19
C. Rat [3%]
• 20
• 21
ID. Spider [3%]
• 22 E. Squirrel [4%]
• 23
_ F. Tick [1 ON
• 24
• 25
• 26 Explanation:
• 27
• 28
• 29 Human rabies
• 30
• 31 Pathogenesis Transmission of rabies virus by bite from infected animal
• 32
• 33
• 34 • Wild carnivores
Reservoir
• 35 • Bats
• 36
• 37
• 39 • Hydrophobia
• 39 • Acrophobia
Encephalitic
• 40 Phorionnn.01 cri,ocrn en.o.ctir.
• 41
• 42 End Block
Feedback
• 1
• 2
▪ 3 Previous Next Lab Values. Notes Calculator.
• 4
▪ 5 Human rabies
▪ 6
▪ 7
Pathogenesis Transmission of rabies virus by bite from infected animal
▪ 8
▪ 9
• 10
• Wild carnivores
• 11 Reservoir
• Bats
• 12
• 13
• 14 • Hydrophobia
• 15

• Acrophobia
• 16 Encephalitic
Clinical • Pharyngeal spasm, spastic paralysis
• 17
• 18 features • Agitation
• 19

Paralytic Ascending flaccid paralysis


• 21
• 22
• 23 Post-exposure Rabies immune globulin & rabies vaccine immediately
• 24 prophylaxis after exposure to high-risk wild animal
• 25
• 26
• 27 Prognosis Coma, respiratory failure & death within weeks
• 28
OUSMLEWuddr LLC
• 29
• 30
Rabies is a fatal disease that is transmitted to humans by contact with infected animals. Transmission of the
• 31
neurotropic rabies virus typically involves exposure to saliva from an infected animal through a bite.
• 32
Raccoons are the most prevalent rabid animal in the United States, especially on the east coast In rare
• 33
cases, transmission can occur by contact with mucous membranes, infected aerosol particles, and corneal
• 34
and organ transplantations_
• 35
• 36
Bites from wild carnivores or bats require thorough wound cleansing with soap and water and urgent
• 37
post-exposure prophylaxis. Post-exposure prophylaxis includes both rabies immune globulin and rabies
• 38
vaccine to prevent progression to life-threatening encephalitic or paralytic rabies disease. Those who miss
• 39
the opportunity for passive and active immunization after a high-risk animal bite are in danger of deadly rabies se
• 40
• 41
• 42 End Block
Feedback
• 1
• 2 Item: 20 of 44 F' Mark
▪ 3 Previous Next Lab Values. Notes Calculator.
• 4
▪ 5 OLISMLEWarld, LLC
▪ 6
▪ 7 Rabies is a fatal disease that is transmitted to humans by contact with infected animals. Transmission of the
▪ 8 neurotropic rabies virus typically involves exposure to saliva from an infected animal through a bite.
▪ 9 Raccoons are the most prevalent rabid animal in the United States, especially on the east coast In rare
• 10 cases, transmission can occur by contact with mucous membranes, infected aerosol particles, and corneal
• 11 and organ transplantations_
• 12
• 13
Bites from wild carnivores or bats require thorough wound cleansing with soap and water and urgent
• 14
post-exposure prophylaxis. Post-exposure prophylaxis includes both rabies immune globulin and rabies
• 15
vaccine to prevent progression to life-threatening encephalitic or paralytic rabies disease. Those who miss
• 16
the opportunity for passive and active immunization after a high-risk animal bite are in danger of deadly rabies
• 17
disease. Hydrophobia and aerophobia are pathognomonic features of encephalitic rabies; the feeling of
• 18
water or air triggers involuntary pharyngeal muscle spasms_ Many patients are disoriented and agitated, with
- 19
fluctuating mental status. Almost all patients die within weeks_
• 20
(Choices A, D and F) Mosquitos are vectors for West Nile virus and Dengue hemorrhagic fever; ticks are
• 21
vectors for Lyme meningitis and Rocky Mountain spotted fever; and spider bites can cause local inflammation.
• 22
including necrosis in severe cases. However, insects do not transmit rabies.
• 23
• 24
(Choices C and E) Small rodents such as squirrels, chipmunks, and rats are less common reservoirs of
• 25
rabies.
• 26
• 27 Educational objective:
• 28 Post-exposure rabies prophylaxis can be life-saving and should be initiated after a bite wound from a wild
• 29 carnivore. Hydrophobia and aerophobia are pathognomonic for encephalitic rabies, and affected patients
• 30 typically die within weeks of illness onset.
• 31
• 32
References:
• 33
. 34 1. Rabies surveillance in the United States during 2012.
• 35 2. Diagnosis, management and post-mortem findings of a human case of rabies imported
• 36 into the United Kingdom from India: a case report.
• 37
• 33
• 39
Copyright © UWorld Last updated: [11/16/2014]
• 40 Ii
• 41
• 42 End Block
Feedback
• 1
• 2 Item: 21 of 44 V-• Mark
▪ 3 Previous Next Lab Values. Notes Calculator.
• 4
▪ 5
▪ 6 7 An 18-month-old child is brought to the emergency department by his mother with fever, vomiting, and
▪ 7 lethargy. He developed a fever during the day that initially responded to treatment with
▪ 8 acetaminophen_ Throughout the day, he became increasingly lethargic and developed a rash on his lower
▪ 9 extremities that has acutely worsened during the past few hours. The child's immunizations are up to date
• 10 and he is otherwise healthy. On examination he is drowsy and lethargic_ He flexes his hips when his neck is
• 11 flexed. He also has an erythematous, nonblanching pinpoint rash on his trunk and bilateral lower
• 12 extremities. What is the most likely organism causing this patient's symptoms?
• 13
• 14
A. Borrefia burgdorferi [1%]
• 15
• 16 E Cytomegalovirus [1%]
• 17 C. Group B Streptococcus [10%]
• 1a
ID. Haemophilus influenzae [4%]
• 19
• 20 E Herpes simplex virus [1%]
I 21 • Listeria monocytogenes [6%]
• 22
• G. Neisseria meningitides [77%]
• 23
• 24
• 25 Explanation:
• 26
• 27 This patient with signs of meningitis and a petechial rash most likely has Neisseria meningitides_ Neisseria
• 28 meningitides is the most common cause of bacterial meningitis in children and young adults in the United
• 29 States_ The mortality rate of meningococcal meningitis is extremely high, approaching nearly 15%. Children
• 30 age < 2 are at greatest risk for contracting this illness, with rates 10 times higher than those of the general
• 31 population. Seventy-five percent of patients with Meningococcus meningitis present with a petechial rash that
• 32 is prominent on the axilla, wrists, flanks and ankles. It appears within 24 hours of the infection.
• 33
• 34 Prompt identification and treatment are critical given that the disease progresses rapidly over the course of a
• 35 few hours and carries a very high morbidity and mortality rate even with appropriate treatment
• 36
• 37 (Choice A) Borrefia burgdorferi causes Lyme disease. It is usually acquired in the late summer months after
• 39 an ixodes sp. tick bite. Lyme disease typically presents with a rash (ie, erythema chronicum migrans),
• 39 headaches, fevers, chills, and malaise. The skin rash is very characteristic and is large, annular, and
• 40 erythematous, occasionally with central clearing_ Although Lyme disease can cause meningitis, it does not se
• 41
• 42 End Block
Feedback
• 1
• 2 Item: 21 of 44 V-Mark
▪ 3 Previous Next Lab Values. Notes Calculator.
• 4 (Choice A) Borrefia burgdorferi causes Lyme disease. It is usually acquired in the late summer months after
▪ 5 an ixodes sp. tick bite. Lyme disease typically presents with a rash (ie, erythema chronicum migrans),
▪ 6 7 headaches, fevers, chills, and malaise. The skin rash is very characteristic and is large, annular, and
▪ 7 erythematous, occasionally with central clearing_ Although Lyme disease can cause meningitis, it does not
▪ 8 present with a petechial rash.
▪ 9
• 10 (Choice B) Cytornegaiovirus can cause infectious mononucleosis like illness, which presents with high
• 11 fevers, fatigue, malaise, splenomegaly, and pharyngitis. It usually does not cause meningitis or present with a
• 12 rash.
• 13
• 14 (Choice C) Group 13 Streptococcus is the most common cause of meningitis in infants. It is usually acquired
• 15 from the mother during childbirth. It is not associated with a rash and is quite unlikely in an 18-month-old.
• 16
• 17
(Choice 0) Haernopfulus inthierrzae may cause meningitis but typically does not present with a rash. It may
•18 also cause rhinorrhea, fever, epiglottis, and ear infections. The rate of H irrfluerrzae infections has decreased
• 19
dramatically since the vaccine was introduced.
• 20
(Choice E) Herpes simplex virus generally causes temporal lobe encephalitis in neonates and infants and
I 21
typically presents with seizures. It would be less likely in an 18-month-old child.
• 22
• 23
(Choice F) Listeria rnorrocytogenes, which is transmitted vaginally, is 1 of the 3 most common causes of
• 24
meningitis in newborns but does not present with a rash. Another mode of transmission is ingestion of
• 25
unpasteurized milk or cheese from infected cows. It would be less likely to occur in an 18-month-old child_
• 26
• 27 Educational objective:
• 28 Meningococcal meningitis is the most common cause of bacterial meningitis in children and young adults.
• 29 It typically presents with fever, headache, neck stiffness, altered mental status. and a petechial or purpuric
• 30 rash. Prompt diagnosis and treatment are critical given that it has a high morbidity and mortality rate even 1
• 31 with appropriate treatment
• 32
• 33
References:
• 34
• 35 1. Multicenter surveillance of invasive meningocaccal infections in children.
• 36 2. Clinical recognition of meningococcal disease in children and adolescents.
• 37
• 39
• 39
Copyright © UWorld Last updated: [8/24/2014]
• 40
• 41
• 42 End Block
Feedback
• 1
• 2 Item: 22 of 44 V- Mark
• 3 Previous Next Lab Values. Notes Calculator.
• 4 11.
• 5
• 6 A 2-month-old infant is brought to the clinic for the evaluation of poor feeding. He was born at 32 weeks of
• 7 gestation with a birth weight of 12GG g. The pertinent physical findings are pallor, tachypnea. tachycardia. and
• 8 flow murmurs. The laboratory studies are as follows:
• 9
Hb 7 WI:IL
• 10
WBC 7,DDD!cmm
• 11
Platelets 230,000/cl-rim
• 12
Reticulocytes Low
• 13
• 14
• 15
The peripheral smear shows normocytic normochromic RBC. What is the most likely diagnosis?
• 16
• 17

• 19 A. Alpha thalassemia [3%]


• 20 B. Beta thalassemia [5%]
• 21
C. Hemolytic disease of newborn [4%]
• 22
• 23 ID. Sickle cell anemia [29C]
- 24 ✓ E Anemia of prematurity [74%]
- 25
• Iron deficiency [99C]
• 26
• 27 G. Vitamin 812 deficiency [156]
• 28 H. Pancytopenia [15C]
• 29
G-6-PD deficiency [2%]
• 30
• 31
• 32 Explanation:
• 33
• 34 Anemia of prematurity is seen in hospitalized, premature or low birth weight infants. The causes are usually
• 35 pathophysiologic, and include: (1) transitions in the erythropoiesis sites of the neonate, (2) shorter life span of
• 36 red blood cells in neonates, and (3) diminished fetoplacental transfusion (occurs when the baby is held above
• 37 the level of the placenta after delivery). Symptoms include poor weight gain, poor feeding, lethargy,
• 38 tachypnea, tachycardia, and pallor_ The hemoglobin level is usually around 7-10 gldL.
• 39
• 40 Anemia of orp.maturitv is defined by the followinn laboratory studips-
• 41
• 42 End Block
Feedback
• 1
• 2 Item: 22 of 44 V-Mark
• 3 Previous Next Lab Values. Notes Calculator.
• 4 Anemia of prematurity is seen in hospitalized, premature or low birth weight infants. The causes are usually
• 5 pathophysiologic, and include: (1) transitions in the erythropoiesis sites of the neonate, (2) shorter life span of
• 6 red blood cells in neonates, and (3) diminished fetoplacental transfusion (occurs when the baby is held above
• 7 the level of the placenta after delivery). Symptoms include poor weight gain, poor feeding, lethargy,
• 8 tachypnea, tachycardia, and pallor_ The hemoglobin level is usually around 7-10 gidL.
• 9
• 10 Anemia of prematurity is defined by the following laboratory studies:
• 11
1. Peripheral smear shows normocytic and normochromic anemia_ No other abnormal forms are seen_
• 12
2. The reticulocyte count is low, and red blood cell precursors in the bone marrow are decreased.
• 13
3. Normal WE C and platelet counts.
• 14
4. Normal total bilirubin level.
• 15
• 16
• 17 The treatment involves iron supplementation, periodic hemoglobin checking and blood transfusion, if needed.
Erythropoietin is not routinely used.
• 19
• 20 (Choice A) Alpha thalassemia is caused by abnormalities in the synthesis of alpha chains of hemoglobin_ It is
• 21 usually a hypochromic, microcytic anemia, and a positive family history is usually present.
• 22
(Choice B) Beta thalassemia is also a hypochromic, microcytic anemia that usually presents at 6-12 months
• 23
of age.
- 24
- 25
(Choice C) Hemolytic disease of the newborn is due to Rh or ARO incompatibility. The clinical features
• 26
depend on the severity of the disease. Symptoms include jaundice, hepatosplenomegaly, pallor, and hydrops
• 27
fetalis in severe cases. The reticulocyte count is increased due to hemolysis.
• 28
• 29 (Choice ID) Sickle cell anemia presents much later because the presence of hemoglobin F protects the
• 30 infants from sickling during the first 4-6 months of life_ Dactylitis may be the initial presentation that warrants
• 31 further work-up for sickle cell disease.
• 32
• 33 (Choice F) Even though iron supplementation is routinely administered in the management of anemia of
• 34 prematurity, iron deficiency is not the usual cause. Iron is given because it will be stored and reutilized during
• 35 the active phase of erythropoiesis.
• 36
• 37 (Choice G) Low levels of folic acid or vitamin B 12 do not cause anemia of prematurity. Furthermore, the
• 38 anemia in such cases is megaloblastic_
• 39
• 40 Educational Objective:
• 41
• 42 End Block
Feedback
• 1
• 2 Item: 22 of 44 Bli- Mark
-<1

. 3 Previous Next Lab Values. Notes Calculator.


1-4.1 ici 1110 UI pi CI 1101411 ILy 13 uciii ICU LI IC 1U111.JYY11 Iy. !OULU CILIA y 3LUUIC-3.
• 4
• 5 1. Peripheral smear shows normocytic and normochromic anemia_ No other abnormal forms are seen.
• 6 2. The reticulocyte count is low, and red blood cell precursors in the bone marrow are decreased.
. 7 3. Normal VVEC and platelet counts.
• 8 4. Normal total bilirubin level.
• 9
• 10
The treatment involves iron supplementation, periodic hemoglobin checking and blood transfusion. if needed.
• 11
Erythropoietin is not routinely used.
• 12
• 13
(Choice A) Alpha thalassemia is caused by abnormalities in the synthesis of alpha chains of hemoglobin. It is
• 14
usually a hypochromic, microcytic anemia, and a positive family history is usually present.
• 15
• 16 (Choice B) Beta thalassemia is also a hypochromic, microcytic anemia that usually presents at 6-12 months
• 17 of age.

• 19 (Choice C) Hemolytic disease of the newborn is due to Rh or ABC incompatibility. The clinical features
• 20 depend on the severity of the disease. Symptoms include jaundice, hepatosplenomegaly, pallor, and hydrops
▪ 21 fetalis in severe cases. The reticulocyte count is increased due to hemolysis.
• 22
▪ 23 (Choice 0) Sickle cell anemia presents much later because the presence of hemoglobin F protects the
- 24 infants from sickling during the first 4- months of life_ Dactylitis may be the initial presentation that warrants
- 25 further work-up for sickle cell disease.
• 26
• 27 (Choice F) Even though iron supplementation is routinely administered in the management of anemia of
• 28 prematurity, iron deficiency is not the usual cause. Iron is given because it will be stored and reutilized during
• 29 the active phase of erythropoiesis.
• 30
(Choice G) Low levels of folic acid or vitamin E 12 do not cause anemia of prematurity. Furthermore. the
• 31
anemia in such cases is megaloblastic.
• 32
• 33
Educational Objective:
. 34
Anemia of prematurity is the most common anemia in premature and low birth weight infants. The pathology
• 35
involves a combination of diminished RE C production, shortened REC life span, and blood loss. Iron
• 36
supplementation does not prevent falling hemoglobin levels, and iron deficiency is not the cause of anemia of
• 37
prematurity.
• 38
• 39
Copyright © UWorld Last updated: [8/19/2014]
• 40
• 41
• 42 End Block
Feedback
2 Item: 23 of 44 V- Mark
3 Previous Next Lab Values. Notes Calculator.
4
5
6 A 1G-week-old girl is brought to the office by her mother for a well-infant examination after missing her visit 2
7 weeks earlier. She was born at 34 weeks gestation and weighed 2_2 kg (4 lb 14 oz) at birth. She has been
breastfed exclusively since then, and her growth and development are appropriate for her chronological and
9 gestational ages. Her physical examination is notable for mild pallor but is otherwise normal. The mother
• 10 eats a balanced diet that includes meat products. At this time, which of the following is the most appropriate
• 11 nutritional intervention for this patient?
• 12
• 13
A. Introduction of eggs and meat to the diet [0%]
• 14
• 15 B. Introduction of fruit juices to the diet [0%]
• 16 C. Introduction of pureed vegetables to the diet [1%]
• 17
tO ID. Iron and vitamin ID supplementation [85%]
• 18
• 19 E No additional supplementation needed [12%]
• 20 F. Vitamin 13 12 supplementation [116]
G. Vitamin C supplementation [0%]
• 22

• 24 Explanation:
• 25
• 26 I
• 27 Timeline of infant nutrition
• 28 I
• 29
• 30
• 31 Birth 6 months 1 year
• 32
• 33
• 34
• 35 Exclusive breastfeeding
• 36
• 37
Introduction of Introduction
• 38
• 39 pureed foods of cow's milk
• 40
• 41
• 42
Feedback End Block
1
2 Item: 23 of 44 F'Mark
3 Previous Next Lab Values, Notes Calculator.
4
Timeline of infant nutrition
5
6
7

1 Birth 6 months 1 year


9
• 10
• 11
• 12
• 13
Exclusive breastfeeding
• 14
• 15 Introduction of Introduction
• 16
pureed foods of cow's milk
• 17
• 18
• 19 OUVVorId
• 20
Iron deficiency anemia is the single most common nutritional deficiency in infants and children and is often
• 22 asymptomatic. Fullterm, healthy infants are born with robust iron stores that generally prevent them from
developing iron deficiency anemia until age 4-6 months, regardless of dietary intake. However. the presence
• 24 of maternal iron deficiency, prematurity, and early introduction of cow's milk before age 12 months
• 25 increases the risk of iron deficiency anemia in infants_
• 26 I
• 27 Human breast milk contains only small amounts of vitamin ID that are inadequate for meeting the infant's daily
• 28 I requirement All exclusively breastfed infants should be started on 400 International Units of vitamin ID daily
• 29 within the first month of life_
• 30
• 31 Given that this infant was born prematurely and is exclusively breastfed, she is at significantly increased risk
• 32 for iron deficiency anemia, and both iron and vitamin ID supplementation should be initiated. Iron
• 33 supplementation should be continued until age 1 year in preterm infants.
• 34
(Choices A and C) The American Academy of Pediatrics (AAP) recommends exclusive breastfeeding
• 35
until age 6 months given that earlier introduction of solid foods is associated with an increase in
• 36
gastrointestinal infections_ Pureed fruits and vegetables should be introduced first followed by pureed proteins
• 37

such as meats. There is no evidence suggesting that early introduction of highly allergenic foods such as
38
• 39
eggs is associated with an increased risk of allergies; these foods can be introduced any time after age 6
• 40
months. V



41
42
Feedback
a
End Block
1
2 Item: 23 of 44 Mark
-4Z1

3 Previous Next Lab Values. Notes Calculator.


%-4 14r4-.1 I LI IL4L LI Mil II II L41 IL TYL4il 4-.111L4LL41 4-.1). 4-.L4ilLI ...LA, 14-. 1.7 L4L il1111114,L411uyII 14.1 GQ.7GU 1 1.71'.
4
5
for iron deficiency anemia, and both iron and vitamin ID supplementation should be initiated. Iron
6
supplementation should be continued until age 1 year in preterm infants.
7
(Choices A and C) The American Academy of Pediatrics (AAP) recommends exclusive breastfeeding
until age 6 months given that earlier introduction of solid foods is associated with an increase in
9
gastrointestinal infections_ Pureed fruits and vegetables should be introduced first followed by pureed proteins
• 10
such as meats. There is no evidence suggesting that early introduction of highly allergenic foods such as
• 11
eggs is associated with an increased risk of allergies; these foods can be introduced any time after age 6
• 12
months.
• 13
• 14 (Choice B) Although some fruit juices contain vitamins, the AAP does not recommend routine introduction of
• 15 juice into an infant's diet. For families who choose to offer juice, no more than 4-6 ounces per day should be
• 16 given (after age 6 months) and it should never be given in a bottle given the significantly increased risk of
• 17 dental caries_
• 18
• 19 (Choice F) Supplementation of vitamin B12 is recommended for exclusively breastfed infants born to strict
• 20 vegetarian mothers.

• 22 (Choice G) Vitamin C deficiency is exceedingly rare in exclusively breastfed infants given that breast milk
contains adequate amounts of vitamin C to meet the infant's daily requirement until at least age 6 months.
• 24
• 25 Educational objective:
• 26 I Full-term infants are born with adequate iron stores to prevent anemia for the first 4- months of life
• 27 regardless of dietary intake. Preterm infants are at significantly increased risk for iron deficiency anemia Iron
• 28 I supplementation should be started at birth in exclusively breastfed preterm infants and continued until age 1
• 29 year. All exclusively breastfed infants should also be started on vitamin ID supplementation_
• 30
• 31 References:
• 32
1. Iron homeostasis in the neonate.
• 33
• 34 2. Diagnosis and prevention of iron deficiency and iron-deficiency anemia in infants and
• 35 young children (0-3 years of age).
• 36 3. Breastfeeding and the use of human milk.
• 37
• 38
• 39
Copyright © LIWorld Last updated: [1D/27/2014]
• 40
• 41
• 42
Feedback End Block
2 Item: 24 of 44 .11V-Mark
3 Previous Next Lab Values. Notes Calculator.
4
5
A previously healthy 8-year-old boy is brought to the office by his mother because he has multiple staring
6
episodes. During these episodes, he is unresponsive to verbal or tactile stimuli, and produces lip-smacking
7
movements. Each episode lasts for a few minutes, after which he remains confused for some time. He has
no family history of any seizure disorder. His neurological examination is unremarkable. EEG performed
9
before and after hyperventilation is normal_ Which of the following is the most likely diagnosis?
• 10
• 11 wI A. Complex partial seizure [32%]
• 12 B. Typical absence seizure [229C]
• 13
C. Atypical absence seizure [37%]
• 14
• 15 D. Juvenile myoclonic epilepsy [4%]
• 16 E. Lennox-Gastaut syndrome [4%]
• 17
• 18
• 19 Explanation:
• 20
• 21 A complex partial seizure is characterized by brief (i.e., lasting for a few minutes) episodes of impaired
• 22 consciousness, failure to respond to various stimuli during the episode, staring spells, automatisms (e.g., lip
i smacking, swallowing, picking movements of the hand), and post-ictal confusion. The EEG pattern is usually
normal or may show brief discharges. Unlike in absence seizures, hyperventilation during the EEG cannot
• 25 simulate a complex partial seizure_
• 26 I
• 27 (Choice B) A typical absence seizure is characterized by brief (i.e., typically lasting for a few seconds)
• 28 periods of impaired consciousness. Patients may also present with automatisms. but do not have a post-ictal
• 29 state. Hyperventilation during the EEG reveals a generalized 3Hz spike-and-wave pattern on a normal
• 30 background.
• 31
• 32 (Choice C) An atypical absence seizure lasts longer. The characteristic EEG pattern is slow spike-and-wave
• 33 activity with a frequency less than 2_5Hz.
• 34
• 35 (Choice 0) Juvenile myoclonic epilepsy presents in adolescents with unilateral or bilateral myoclonic jerks.
• 36 The symptoms take place in the morning, and may be precipitated by sleep deprivation.
• 37
• 38 (Choice E) Lennox-Gastaut syndrome usually presents in childhood as seizures of multiple types, impaired
• 39 cognitive function, and slow spike-and-wave activity on EEG.
• 40
se
• 41
• 42 End Block
Feedback
1
2 Item: 24 of 44 V- Mark
3 Previous Next Lab Values. Notes Calculator.
4
5
A. Complex partial seizure [32%]
6 B. Typical absence seizure [22%]
7
C. Atypical absence seizure [37%]
8
9
ID. Juvenile myoclonic epilepsy [4%]
• 10 E. Lennox-Gastaut syndrome [4%]
• 11
• 12
• 13 Explanation:
• 14
A complex partial seizure is characterized by brief (Le., lasting for a few minutes) episodes of impaired
• 15
consciousness, failure to respond to various stimuli during the episode, staring spells, automatisms (e.g., lip
• 16
smacking, swallowing, picking movements of the hand), and post-ictal confusion. The EEG pattern is usually
• 17
normal or may show brief discharges. Unlike in absence seizures, hyperventilation during the EEG cannot
• 18
simulate a complex partial seizure_
• 19
• 20
(Choice B) A typical absence seizure is characterized by brief (Le., typically lasting for a few seconds)
• 21
periods of impaired consciousness. Patients may also present with automatisms, but do not have a post-ictal
• 22
state. Hyperventilation during the EEG reveals a generalized 3Hz spike-and-wave pattern on a normal
i
background.
• 25
(Choice C) An atypical absence seizure lasts longer. The characteristic EEG pattern is slow spike-and-wave
• 26 I
activity with a frequency less than 2_5Hz.
• 27
• 28
(Choice ID) Juvenile myoclonic epilepsy presents in adolescents with unilateral or bilateral myoclonic jerks.
• 29
The symptoms take place in the morning, and may be precipitated by sleep deprivation.
• 30
• 31
(Choice E) Lennox-Gastaut syndrome usually presents in childhood as seizures of multiple types. impaired
• 32
cognitive function, and slow spike-and-wave activity on EEG.
• 33
• 34
Educational objective:
• 35
A complex partial seizure is characterized by brief episodes of impaired consciousness, failure to respond to
• 36
various stimuli during the episode, staring spells, automatisms, and post-ictal confusion. The EEG pattern is
• 37
usually normal or may show brief discharges.
• 38
• 39
Copyright © LIWorld Last updated: [12/29/2014]
• 40 •

• 41
• 42 End Block
Feed back
• 1
• 2 Item: 25 of 44 V-Mark .4111°.
▪ 3 Previous Next Lab Values. Notes Calculator.

• 4-
▪ 5
▪ 6 A 15-year-old girl is brought to the physician by her mother because she has not begun menstruating_ She is
▪ 7 otherwise healthy and has no medical problems_ Vitals signs are within normal limits. Physical examination
▪ 8 shows absence of breast development and no pubic hair is seen. Examination shows no other
▪ 9 abnormalities. Ultrasound confirms the presence of a uterus. Which of the following is the most appropriate
• 10 next step in management?
• 11
• 12
A. Estrogen level [13%]
• 13
• 14 • MRI of pituitary [1%]
• 15 C. Serum FSH level [41%]
• 16
• 17 D. Karyotyping [20%]
• 18 E GnRH stimulation test [14%]
• 19 • Reassurance [12%]
• 20
• 21
• 22 Explanation:
• 23
• 24 Evaluation of primary amenorrhea

• 26
Pelvic examination
• 27
or ultras-410g FaITI
• 28
• 29
• 30

• 31
Uterus present Uterus absent
• 32
• 33
• 34
• 35 Serum FSI-1 Ha ryotype
• 36 serum testosterone
• 37
• 38 Increased Decreased
• 39 46, XX 46„ XV V
• 40
• 41
• 42 End Block
Feedback

• 2 Item: 25 of 44 F'Mark .4111°.
▪ 3 Previous Next Lab Values. Notes Calculator.

• 4-
▪ 5 Evaluation of primary amenorrhea
▪ 6
▪ 7
Pelvic examination
▪ 8
or ultrasonogram
▪ 9
• 10
• 11
• 12
Uterus present Uterus absent
• 13
• 14
• 15 1
• 16 Serum 151-I Ha ryotype
• 17 serum testosterone
• 18
• 19 Increased Decreased
• 20 46, XX 46, XY
• 21 Normal female Normal male
• 22 1 testosterone testosterone 1

Karyotyping Cranial MRI


• 23 levels levels
• 24

• 26
• 27 Abnormal Mullerian Androgen
development insensitivity syndrome
• 28 utactwoktut c ail
• 29
• 30 This patient has primary amenorrhea and underdevelopment of secondary sexual characteristics. Isolated
• 31 amenorrhea with well-developed secondary sexual characteristics can be considered normal up to the age of
• 32 16. However, if secondary sexual characteristics are absent, as with this patient, work-up should not be
• 33 delayed beyond age 14 (Choice F). The absence of breast development indicates a lack of estrogen, so
• 34 measuring the estrogen level provides no additional information (Choice A).
• 35
• 36 Primary amenorrhea can be due to either hypothalamic/pituitary (central) abnormalities, or to gonadal
▪ 37 (peripheral) abnormalities. This distinction can be made by measurement of the FSH level. Increased FSH
• 38 (hypergonadotropic amenorrhea) indicates a peripheral cause, and decreased FSH (hypogonadotropic
▪ 39 amenorrhea) indicates a central cause. If the amenorrhea is of central origin, a pituitary MRI is indicated to
• 40 Innk fnr a leginn in the cella tl irrir inhnirta RI If AMPrinfrhPA is of nerinherAl nrinin 1.cAritni-itninn wni ilri hp the
• 41
• 42 End Block
Feedback
• 1
• 2 Item: 25 of 44 V-. Mark
-<1

▪ 3 Previous Next Lab Values. Notes Calculator.


• 4- Normal female Normal male
Jr
▪ 5 testosterone testosterone
Karyotyping Cranial MAI
▪ 6 levels levels
▪ 7
▪ 8
▪ 9
I

Abnormal Muilerian Androgen


• 10
development insensitivity syndrome
• 11 iD 201 1

• 12
• 13 This patient has primary amenorrhea and underdevelopment of secondary sexual characteristics. Isolated
• 14 amenorrhea with well-developed secondary sexual characteristics can be considered normal up to the age of
• 15 16. However, if secondary sexual characteristics are absent, as with this patient, work-up should not be
• 16 delayed beyond age 14 (Choice F). The absence of breast development indicates a lack of estrogen, so
• 17 measuring the estrogen level provides no additional information (Choice A).
• 18
• 19 Primary amenorrhea can be due to either hypothalamic/pituitary (central) abnormalities, or to gonadal
• 20 (peripheral) abnormalities. This distinction can be made by measurement of the FSH level. Increased FSH
• 21 (hypergonadotropic amenorrhea) indicates a peripheral cause, and decreased FSH (hypogonadotropic
• 22 amenorrhea) indicates a central cause. If the amenorrhea is of central origin, a pituitary PORI is indicated to
• 23 look for a lesion in the sells turcica (Choice 6). If amenorrhea is of peripheral origin, karyotyping would be the
• 24 next step (Choice I3).

(Choice E) A GnRH stimulation test is used for the evaluation of precocious puberty.
• 26
• 27
Educational objective:
• 28
In a patient with primary amenorrhea:
• 29
• 30 • FSH measurement should be ordered if there is no breast development
• 31 • Pituitary MRI is the next step if FSH is decreased
• 32 • Karyotyping is the next step if FSH is increased
• 33
• 34
• 35 References:
• 36 1. Evaluation and management of adolescent amenorrhea.
▪ 37
• 38
▪ 39
Copyright © LIWorld Last updated: [10/30/2014]
• 40
• 41
• 42 End Block
Feedback
Item: 26 of 44 V- Mark
-461 M=E--
Previous , Next , Lab Values, Notes Calculator.

Bacterial meningitis in children age >1 month


7
• 8 * Fever
• 0
• Vomiting/poor feeding
• 10
Clinical * Seizures
• 11
• 12
features • Altered mental status (eg, lethargy, irritability)
• 13 0 Nuchal rigidity, Kernig & Brudzinski signs

• 14 • Bulging anterior fontanelle


• 15
• 16
• 17 • Complete blood count & electrolytes
• 18 Workup • Blood cultures
• 19 • LP & cerebrospinal fluid studies
• 20
• 21
• 22 Indications for • History of hydrocephalus or neurosurgical procedure
• imaging prior • History of head trauma
• 24- to LP • Coma or focal neurologic findings

▪ 27 0 intravenous vancomycin & ceftriaxone OR


• 28 I cefotaxime
Treatment
• 29 0 Dexannethasone for Haotnophilus influenzae type b

• 30 meningitis

• 32 OUSMLEWarld, LLC

• 34 This infant's clinical presentation (altered mental status, lethargy, fever, nuchal rigidity. vomiting) is
concerning for bacterial meningitis. Streptococcus prieumoniae and Neisseria meningiticlis are the most
• 36 common causes of bacterial meningitis in children age >1 month. Management requires prompt
administration of empiric antibiotics and cerebrospinal fluid (CSF) analysis to establish the diagnosis and
• 38 identify the offending pathogen. As it can take >30 minutes to prepare and perform a lumbar puncture (LP)
• 39 with sterile technique. ill-appearing patients with high likelihood of bacterial meningitis should receive empiric
• 40 antibiotic therapy as soon as blood cultures are sent. Althouah the antibiotics aiven before LP can affect the
• 41
• 42 11.
Feedback, End Block.
• 1
• 2 Item: 26 of 44 F'Mark
▪ 3 Previous Next Lab Values. Notes Calculator.
• 4 to LP • Coma or focal neurologic findings
▪ 5
▪ 6
▪ 7 • ntravenous vancomycin & cariaxone OR
I

▪ 8 cefotaxi me
Treatment
▪ 9 • Dexamethasone for Haotnophilus infiuenzae type b
• 10 meningitis
• 11
• 12 QUSHLEWarkl, LLC
• 13
• 14 This infant's clinical presentation (altered mental status, lethargy, fever, nuchal rigidity, vomiting) is
• 15 concerning for bacterial meningitis. Streptococcus pneumoniae and Neisseria meningitidis are the most
• 16 common causes of bacterial meningitis in children age >1 month. Management requires prompt
• 17 administration of empiric antibiotics and cerebrospinal fluid (CSF) analysis to establish the diagnosis and
• 18 identify the offending pathogen_ As it can take >30 minutes to prepare and perform a lumbar puncture (LP)
• 19 with sterile technique, ill-appearing patients with high likelihood of bacterial meningitis should receive empiric
• 20 antibiotic therapy as soon as blood cultures are sent Although the antibiotics given before LP can affect the
• 21 yield of cerebrospinal fluid Gram stain and culture, pathogens other than meningococcus can usually be
• 22 identified up to several hours after antibiotic administration_
• 23
• 24 Empiric antibiotic therapy should be directed at the most common pathogens. Ceftriaxone is effective
• 25 against most strains of S pneurnorriae and N meningitidis; vancomycin is given due to increasing prevalence
of resistant strains of S pneurnorriae. In neonates age X28 days, ceftriaxone is avoided as it displaces bilirubin
• 27 from albumin and increases the risk of kernicterus. This risk does not exist in older infants who are unlikely to
• 28 have hyperbilirubinemia.
• 29
(Choices A and B) Intravenous antibiotics are the priority when meningitis is suspected. Head CT would
• 30
delay LP and expose the infant to high doses of radiation. In addition, herniation is less likely in infants age <1
• 31
year as the fontanelles have not fully closed and can accommodate some brain swelling. The risk of
• 32
herniation is low in the absence of coma, focal neurologic findings, or history of a neurosurgical procedure
• 33
(eg, ventriculoperitoneal shunt) (Table). Therefore, head CT is not usually indicated in infants such as this
• 34
patient
• 35
• 36
(Choice C) Although ceftriaxone and vancomycin are indicated, it would not be appropriate to hospitalize the
• 37
infant without first performing LP.
• 38
• 39 (Choice El LP should be performed prior to administering antibiotics in patients with lower suspicion for
• 40
se
• 41
• 42 End Block
Feedback
1
2 Item: 26 of 44 V-• Mark -4Z1

3 Previous Next Lab Values. Notes Calculator.


ULJI 5....I.J111U1 I ILIIU Ul I I LUII I Ul ULI ici uiaiI Iiiciiiii utiutitiu3 tiraiI
4
5
identified up to several hours after antibiotic administration.
6
Empiric antibiotic therapy should be directed at the most common pathogens. Ceftriaxone is effective
7
against most strains of S pneurnorriae and N meningitidis; vancomycin is given due to increasing prevalence
8
of resistant strains of S pneurnorriae. In neonates ages28 days, ceftriaxone is avoided as it displaces bilirubin
9
from albumin and increases the risk of kernicterus. This risk does not exist in older infants who are unlikely to
• 10
have hyperbilirubinemia.
• 11
• 12 (Choices A and B) Intravenous antibiotics are the priority when meningitis is suspected. Head CT would
• 13 delay LP and expose the infant to high doses of radiation. In addition, herniation is less likely in infants age <1
• 14 year as the fontanelles have not fully closed and can accommodate some brain swelling_ The risk of
• 15 herniation is low in the absence of coma, focal neurologic findings, or history of a neurosurgical procedure
• 16 (eg, ventriculoperitoneal shunt) (Table). Therefore, head CT is not usually indicated in infants such as this
• 17 patient
• 18
• 19 (Choice C) Although ceftriaxone and vancomycin are indicated, it would not be appropriate to hospitalize the
• 20 infant without first performing LP.
• 21
;•
• sm
22 (Choice El LP should be performed prior to administering antibiotics in patients with lower suspicion for
• 23 meningitis and with a stable neurologic examination.
• 24
• 25 Educational objective:
Bacterial meningitis in infants and children presents with lethargy, fever, poor oral intake, and vomiting. It is
• 27 crucial to administer empiric antibiotics (ceftriaxone and vancomycin) first and then perform a lumbar
• 28 I puncture as soon as possible. Imaging should be performed before the lumbar puncture in infants and
• 29 children in coma, with focal neurologic findings, or with a history of a neurosurgical procedure_
• 30
• 31 References:
• 32
1. Clinical features suggestive of meningitis in children: a systematic review of prospective
• 33
data.
• 34
• 35 2. Bacterial meningitis in the United States, 1998-2007.
• 36 3. Acute bacterial meningitis in infants and children: epidemiology and management.
• 37
• 38
• 39
Copyright © LIWorld Last updated: [1G/16/n14]
• 40
• 41
• 42
Feedback End Block
1
• 2 Item: 27 of 44 V-. Mark
3 Previous Next Lab Values. Notes Calculator.
• 4-
5
6 A 16-year-old boy is brought to the pediatrician with pain and limited motion of the right knee. He first noticed
7 knee pain and swelling 6 months earlier, and the pain has gradually worsened since then. He has no history
8 of fever, local erythema, or trauma. The patient reports that his right knee and right ankle have swelled
9 several times before_ He has a history of easy bruising since childhood and had an episode of excessive
• 10 bleeding after a tooth extraction. His uncle had similar problems_ Which of the following is the most likely
• 11 cause of this patient's joint pain?
• 12
• 13
A. Avascular necrosis [3%]
• 14
• 15 B. Crystal deposition [1%]
• 16 C. Hemosiderin deposition and fibrosis [56%]
• 17
ID. Immunologic tissue injury [8%]
• 18
• 19 E Occult traumatic injury [12%]
• 20 • Repetitive microtrauma [2094)]
• 21
G. Vector-borne illness [TA]
• 22
• 23
• 24 Explanation:
• 25
• 26

Hemophilia A & B
• 28
• 29
• 30 X-linked recessive disorder of factor VIII deficiency
Pathophysiology
• 31 (hemophilia A) or factor IX deficiency (hemophilia B)
• 32
• 33
• 34 • Occurs almost exclusively in males
Epidemiology
• 35 • Female carriers may have mild symptoms
• 36
• 37
• 38 • Hemarthrosis, hemophilic arthropathy
• 39 Clinical features • Intramuscular hernatornas
V
• 40 • inAstrriiritp,stinRi nr irinAry tr rt hIppriinn
• 41
• 42 End Block
Feedback
• 1
• 2 Item: 27 of 44 F'Mark
3 Previous Next . Lab Values. Notes Calculator.
4 A

5
6
Hemophilia A & B
7
8
X-linked recessive disorder of factor VIII deficiency
9 Pathophysiology
(hemophilia A) or factor IX deficiency (hemophilia B)
• 10
• -1-1
• 12 • Occurs almost exclusively in males
• 13 Epidemiology
• Female carriers may have mild symptoms
I
• 14
- .
• 15
• 16 • Hemarthrosis, hemophilic arthropathy
• 17 Clinical features • Intramuscular hematomas
• 18 • Gastrointestinal or genitourinary tract bleeding
• 19
• 20
• 21 • Normal platelet count
ffmi
2 • Normal bleeding time
• 23
Laboratory findings • Normal prothrombin time
• 24
• 25
• Prolonged activated partial thromboplastin time
• 26 • Decreased or absent factor VIII or IX activity

• 28
Treatment Administration of factor VIII or factor IX
• 29
• 30 VUSAILEWurld,LLC
• 31
• 32 This patient's history of easy bruising since childhood, excessive bleeding after a dental procedure, and
• 33 recurrent joint swelling, along with his family history, are most consistent with hemophilia. Hemophilia A and
• 34 B, which are caused by deficiencies of Factor VIII and Factor IX, respectively, are bleeding disorders inherited
• 35 in an X-linked recessive pattern. Although only male patients are severely affected, female carriers may be
• 36 mildly symptomatic. Hemophilia can affect nearly every organ system and result in long-term complications
• 37 such as hemophilic arthropathy.
• 33
• 39 Hemophilic arthropathy, a late complication that occurs in both types of hemophilia and is a significant source
• 40 of morbidity is thounht to ho hausp.d by ironihamosidarin danosition 1p.adinn to svnovitis and fibrosis so
• 41
• 42 End Block
Feedback
• 1
• 2 Item: 27 of 44 V-Mark
▪ 3 Previous Next Lab Values. Notes Calculator.
• 4- B, which are caused by deficiencies of Factor VIII and Factor IX, respectively, are bleeding disorders inherited A

▪ 5 in an X-linked recessive pattern. Although only male patients are severely affected, female carriers may be
▪ 6 mildly symptomatic. Hemophilia can affect nearly every organ system and result in long-term complications
▪ 7 such as hemophilic arthropathy.
▪ 8
▪ 9 Hemophilic arthropathy, a late complication that occurs in both types of hemophilia and is a significant source
• 10 of morbidity, is thought to be caused by ironihemosiderin deposition leading to synovitis and fibrosis
• 11 within the joint However, other mechanisms may also be involved. Hemophilic arthropathy typically presents
• 12 with chronic worsening joint pain and swelling and can result in contractures of the joint and limited range of
• 13 motion. It is most common in patients with a history of recurrent hemarthroses. Although severe hemophilic
• 14 arthropathy may be visible on plain films, magnetic resonance imaging allows for much earlier detection
• 15 and characterization of the degree of joint damage. Hemophilic arthropathy cannot be entirely prevented.
• 16 However, early prophylactic treatment with factor concentrates can significantly reduce the risk of
• 17 developing arthropathy.
• 18
• 19 (Choice A) Avascular necrosis of the femoral head is seen in children with sickle cell disease and in
• 20 Legg-Calve-Perthes disease (LCP). LCP, or idiopathic avascular necrosis of the femoral head, typically
• 21 presents with limp or hip pain and is most common in children age 4-12. It is rare in adolescents. Sickle cell
• 22 disease may also cause recurrent vaso-occlusive episodes, which may affect the joints, but are not
• 23 associated with excessive bruising or bleeding.
• 24
• 25
(Choice B) Urate and calcium pyrophosphate deposition are the mechanisms of arthritis in gout and
pseudogout, respectively. Both conditions are uncommon in children.
• 26

(Choice 0) Immunologic tissue injury occurs in a variety of conditions, such as rheumatoid arthritis and
• 28
psoriatic arthritis_ But while inflammatory arthritis may cause joint swelling, these conditions would not explain
• 29
the patient's bleeding symptoms.
• 30
• 31 (Choice El Although minor unnoticed trauma can result in episodes of hemarthrosis in patients with
• 32 hemophilia, the prolonged course of symptoms in this patient does not suggest acute bleeding into the joint
• 33 Rather, it is more consistent with a chronic inflammatory process due to iron/hemosiderin deposition.
• 34
• 35 (Choice F) Repetitive microtrauma, also known as "the wear and tear" phenomenon. is thought to play a role
• 36 the development of osteoarthritis, which is most common in adults age >55.
• 37
• 38 (Choice G) Lyme disease is a vector-borne illness that can result in arthritis of the large joints as a late
• 39 finding. Although Lyme arthritis can present with swelling and limited range of motion, it would be unlikely to
• 40 account for this patient's other findinas_ so
• 41
• 42 End Block
Feedback
• 1
• 2 Item: 27 of 44 V-Mark
3 Previous Next Lab Values. Notes Calculator.
• 4- ILL.IIIJIL;C VING131-U1111 I ICU! U3I3 UI LI IC ICI I IUI CH I ICGIU 13 3CCI I II I 4,1111U1 CI I MU I 311...pdc LCII U13CCI3C AI IU II I

5 Legg-Calve-Perthes disease (LOP)_ LOP, or idiopathic avascular necrosis of the femoral head, typically
6 presents with limp or hip pain and is most common in children age 4-12. It is rare in adolescents. Sickle cell
7 disease may also cause recurrent vaso-occlusive episodes, which may affect the joints, but are not
8 associated with excessive bruising or bleeding.
9
(Choice B) Urate and calcium pyrophosphate deposition are the mechanisms of arthritis in gout and
• 10
pseudogout, respectively. Both conditions are uncommon in children.
• 11
• 12
(Choice ID) Immunologic tissue injury occurs in a variety of conditions, such as rheumatoid arthritis and
• 13
psoriatic arthritis_ But while inflammatory arthritis may cause joint swelling, these conditions would not explain
• 14
the patient's bleeding symptoms.
• 15
• 16 (Choice E) Although minor unnoticed trauma can result in episodes of hemarthrosis in patients with
• 17 hemophilia, the prolonged course of symptoms in this patient does not suggest acute bleeding into the joint
• 18 Rather, ft is more consistent with a chronic inflammatory process due to iron/hemosiderin deposition.
• 19
• 20 (Choice F) Repetitive microtrauma, also known as "the wear and tear' phenomenon, is thought to play a role
• 21 the development of osteoarthritis, which is most common in adults age >55.
• 22
• 23 (Choice G) Lyme disease is a vector-borne illness that can result in arthritis of the large joints as a late
• 24 finding_ Although Lyme arthritis can present with swelling and limited range of motion. it would be unlikely to
• 25 account for this patient's other findings.
• 26
Educational objective:
• 28 Hemophilic arthropathy is a delayed consequence of recurrent hemarthrosis. It is associated with 4

• 29 hemosiderin deposition leading to synovitis and fibrosis within the joint The risk of hemophilic arthropathy can
• 30 be significantly reduced by prophylactic treatment with factor concentrates_
• 31
• 32 References:
• 33
1. Pathobiology of hemophilic synovitis I: overexpression of mdm2 oncogene.
• 34
• 35 2. Hemophilic arthropathy: a review of imaging and staging.
• 36 3. Pathogenesis of haemophilic arthropathy.
• 37
• 38
• 39
Copyright © UWorld Last updated: [7/16/2014]
• 40
• 41
• 42 End Block
Feedback
2 Item: 27 of 44 .111M 'Mark FF.
3 Previous Next Lab Values Notes Calculator

4- Media Exhibit X
5
6
7
8
9
• 10
• -1 -1
• 12
• 13
• 14
• 15
• 16
• 17
• 18
• 19
• 20
• 21
z=
22
• 23
• 24
• 25
• 26
rF;
• 23
• 29
• 30
• 31
• 32
• 33
• 34
• 35
• 36
• 37
• 33
• 39
•••••
• 40


41
42
Feedback
a
End Block
• 1
• 2 Item: 28 of 44 V- Mark
▪ 3 Previous Next Lab Values. Notes Calculator.
• 4-
▪ 5
▪ 6 A 4-day-old boy in the newborn nursery has failed a bilateral hearing screening_ He was born at 37 weeks
▪ 7 gestation by elective cesarean section for intrauterine growth retardation. His mother traveled to Egypt during
▪ 8 the first trimester of pregnancy and developed a brief febrile illness but otherwise felt well during the
▪ 9 pregnancy. The infant's weight, length, and head circumference are <5th percentile. Physical examination
• 10 shows bilateral white pupillary reflexes. The boy does not alert to voice or sound. Auscultation shows a
• 11 continuous systolic-diastolic murmur over the left 2nd intercostal space. The most likely cause of the
• 12 newborn's symptoms is infection with which of the following organisms?
• 13
• 14
• 15
A. Cytomegalovirus [14%]
• 16 B. Herpes simplex virus [0%]
• 17
C. Rubella virus [80'36]
• 18
• 19
D. Toxoptasma gondii [4%]
• 20 E. Treporrema paltidurn [1'36]
• 21
F. Varicella zoster virus [0%]
• 22
• 23
• 24 Explanation:
• 25
- 25
- 27
Rubella (German measles)
m
28
• 29 • Sensorineural hearing loss
• 30 Congenital • Cardiac anomalies (eg, patent ductus anenosus)
• 31 • Cataracts, glaucoma
• 32
• 33
• Low-grade fever
• 34
▪ 35
Clinical • Conjunctivitis, coryza, cervical
• 36 presentation Children lymphadenopathy, Forschheimer spots
▪ 37 • Cephalocaudal spread of blanching,
• 38
erythematous maculopapular rash
▪ 39
• 40
so
• 41
• 42
• -1
• 2 Item: 28 of 44 F'Mark
▪ 3 Previous Next Lab Values. Notes Calculator.
• 4-
▪ 5
▪ 6
Rubella (German measles)
▪ 7
▪ 8
• Sensorineural hearing loss
▪ 9 Congenital • Cardiac anomalies (eg, patent ductus artenosus)
• 10 • Cataracts, glaucoma
• -1 -1
• 12
• Low-grade fever
• 13
Clinical • Conjunctivitis, coryza, cervical
• 14
• 15 presentation Children lym phadenopathy, Forschheimer spots
• 16 • Cephalocaudal spread of blanching,
• 17
erythematous maculopapular rash
• 18
• 19
• 20 Adolescents]
• Same -as children + arthralgiasiarthritis
• 21 Adults
• 22
• 23
• Polymerase chain reaction
• 24 Diagnosis
• Acute & convalescent serology for ant-rubella IgM & IgG
• 25
- 25
- 27 Prevention . Live attenuated rubella vaccine
'Do

• 29
Treatment . Supportive care
• 30 © SWILEVYarld, LLC
• 31
• 32 Rubella infection in adults can be asymptomatic in 25%-50% of patients or characterized by mild symptoms
33 (eg, low-grade fever, maculopapular rash). However, infection during pregnancy is dangerous due to high risk
• 34 for transmission of the rubella virus from the mother's bloodstream to the placenta and into fetal circulation.
• 35 First-trimester infections are the most devastating, with potential outcomes such as spontaneous abortion
• 36 and congenital rubella syndrome. This newborn has the classic triad of leukocoria (white papillary reflex) from
• 37 cataracts, murmur of patent ductus arteriosus, and hearing loss_
• 30
• 39 A probable diagnosis can be made clinically and confirmed by culture, serology, or polymerase chain reaction
• 40 (Table). Congenital rubella syndrome can be prevented by selective immunization of women of childbearing so
• 41
• 42 End Block
Feedback
• 1
• 2 Item: 28 of 44 'Mark
-<1

▪ 3 Previous Next Lab Values. Notes Calculator.


• 4- for transmission of the rubella virus from the mother's bloodstream to the placenta and into fetal circulation.
▪ 5 First-trimester infections are the most devastating, with potential outcomes such as spontaneous abortion
▪ 6 and congenital rubella syndrome. This newborn has the classic triad of leukocoria (white pupillary reflex) from
▪ 7 cataracts., murmur of patent ductus arteriosus, and hearing loss_
▪ 8
▪ 9 A probable diagnosis can be made clinically and confirmed by culture, serology, or polymerase chain reaction
• 10 (Table). Congenital rubella syndrome can be prevented by selective immunization of women of childbearing
• 11 age, but widespread immunization is preferable for complete eradication of the virus from herd immunity.
• 12 Rubella immunization is contraindicated during pregnancy due to a theoretical risk for the live vaccine to
• 13 cause fetal infection.
• 14
• 15
(Choice A) Maternal cytomegalovirus infection can be mild and minimally symptomatic, but congenital
• 16
infection shares similar manifestations with in utero rubella infection, including deafness, purpura,
• 17
hepatosplenomegaly, and jaundice. However, the deafness is typically unilateral, blindness is usually due to
• 18
chorioretinitis, and the heart is unaffected.
• 19
(Choice B) Herpes simplex virus is usually acquired perinatally from vaginal delivery. Newborns can
• 20
deteriorate rapidly from invasive meningoencephalitis, which can result in permanent hearing loss and/or
• 21
blindness. Cataracts, deafness, and heart defects would not be present at birth as the fetus was not infected
• 22
in utero.
• 23
• 24
(Choice ID) Congenital toxoplasmosis causes the triad of chorioretinitis, hydrocephalus, and intracranial
• 25
calcifications. It is also associated with sensorineural hearing loss, but not cardiac defects_
- 25
- 27 (Choice E) Congenital syphilis infection can cause cataracts and sensorineural hearing loss but is not
m 28 associated with congenital heart disease.
• 29
• 30 (Choice F) Congenital varicella syndrome is characterized by microcephaly. limb hypoplasia. intrauterine
• 31 growth retardation, and cataracts with a maternal history of a pruritic, generalized vesicular rash. Congenital
• 32 heart defects are not a typical finding in this disease.
33
• 34 Educational objective:
• 35 Maternal-fetal transmission of the rubella virus is most teratogenic during the first trimester of pregnancy. The
• 36 classic triad of congenital rubella syndrome consists of sensorineural deafness. cardiac defects (eg, patent
• 37 ductus arteriosus), and cataracts.
• 38
• 39 References:
• 40
se
• 41
• 42 End Block
Feedback
• 1
• 2 Item: 28 of 44 F Mark -4Z1

▪ 3 Previous Next Lab Values. Notes Calculator.


• 4- Kuaeiia immunization is contrainoicatea airing pregnancy aue to a tneoreticai risk- Tor tne live vaccine to
▪ 5 cause fetal infection.
▪ 6
(Choice A) Maternal cytomegalovirus infection can be mild and minimally symptomatic, but congenital
▪ 7
infection shares similar manifestations with in utero rubella infection, including deafness, purpura,
▪ 8
hepatosplenomegaly, and jaundice. However, the deafness is typically unilateral, blindness is usually due to
▪ 9
chorioretinitis, and the heart is unaffected.
• 10
• 11
(Choice B) Herpes simplex virus is usually acquired perinatally from vaginal delivery. Newborns can
• 12
deteriorate rapidly from invasive meningoencephalitis, which can result in permanent hearing loss and/or
• 13
blindness. Cataracts, deafness, and heart defects would not be present at birth as the fetus was not infected
• 14
in utero.
• 15
• 16 (Choice ID) Congenital toxoplasmosis causes the triad of chorioretinitis, hydrocephalus, and intracranial
• 17 calcifications. It is also associated with sensorineural hearing loss, but not cardiac defects_
• 18
• 19 (Choice El Congenital syphilis infection can cause cataracts and sensorineural hearing loss but is not
• 20 associated with congenital heart disease.
• 21
• 22 (Choice F) Congenital varicella syndrome is characterized by microcephaly, limb hypoplasia, intrauterine
• 23 growth retardation, and cataracts with a maternal history of a pruritic, generalized vesicular rash_ Congenital
• 24 heart defects are not a typical finding in this disease_
• 25
- 25 Educational objective:
- 27 Maternal-fetal transmission of the rubella virus is most teratogenic during the first trimester of pregnancy. The I
m 28 classic triad of congenital rubella syndrome consists of sensorineural deafness, cardiac defects (eg, patent
• 29 ductus arteriosus), and cataracts.
• 30
• 31 References:
• 32
1. Rubella and congenital rubella (German measles).
33
34 2. Rubella.
• 35 3. Three cases of congenital rubella syndrome in the postelimination era--Maryland,
▪ 36 Alabama, and Illinois, 2012.
• 37
▪ 38
• 39
Copyright © UWorld Last updated: [1011/2014]
• 40 Ii
• 41
• 42 End Block
Feedback
• 1
• 2 Item: 29 of 44 V- Mark
▪ 3 Previous Next Lab Values. Notes Calculator.
• 4-
▪ 5
▪ 6 A 14-year-old boy comes to the physician with two days of fever and nasal discharge. He has also had
▪ 7 malaise, fatigue, and myalgia_ He has no other medical problems_ Family history is negative for any kidney
▪ 8 disease. His temperature is 38.8° C (101.9° F), blood pressure is 122/74 mm Hg, pulse is 95/min, and
▪ 9 respirations are 15/min. Examination shows no abnormalities_ An incidental urine dipstick testing shows 2+
• 10 proteinuria but no hematuria, pyuria, or active urine sediment Serum creatinine is within normal limits_
• 11 Which of the following is the most appropriate next step in management?
• 12
• 13
• 14
▪ A. Repeat dipstick testing on two subsequent occasions [3296]
• 15 B. Check serum protein and albumin levels [10%]
• 16
C. Order 24-hour urinary collection for protein [26%]
• 17
• 18
D. Order renal ultrasound [3%]
• 19 E. Reassure, with no further workup [27%]
• 20
• Renal biopsy [2%]
• 21
• 22
• 23 Explanation:
• 24
• 25 A urine dipstick can be positive in up to 10% of school-aged children. Proteinuria in children can be transient
• 26 (intermittent), orthostatic, or persistent Transient proteinuria is the most common cause of proteinuria and
• 27 can be caused by fever, exercise, seizures, stress, or volume depletion. Orthostatic proteinuria is very
• 28 common in adolescent boys and is defined as increased protein when the patient is in an upright position that
returns to normal when the patient is recumbent If the urinalysis shows no hematuria and is otherwise
• 30 normal, the urine dipstick should be repeated on at least two additional specimens (Choice A). If these
• 31 subsequent tests are negative for protein, the diagnosis is transient proteinuria. Transient and orthostatic
• 32 proteinurias are usually benign conditions that require no further evaluation_
• 33
• 34 If the proteinuria persists on the repeat sample or if any of the initial studies are abnormal, the patient should
• 35 be referred to a pediatric nephrologist and evaluated for underlying renal disease. Further investigation may
• 36 include 24-hour urinary collection for protein, renal ultrasound, and, possibly, renal biopsy (Choices C, D, and
• 37 F). This patient presents with proteinuria during a febrile illness and should be tested again in the future to rule
• 38 out persistent proteinuria.
• 39
(Choice B) Checking serum protein (albumin) has little value in evaluating proteinuria since many patients „„.
• 40
• 41
• 42 End Block
Feedback
1
• 2 Item: 29 of 44 V-Mark
3 Previous Next Lab Values. Notes Calculator.
• 4-
5 D. Order renal ultrasound [3%]
6 E Reassure, with no further workup [7°•0]
7
Renal biopsy [2%]
8
9
• 10 Explanation:
• 11
• 12 A urine dipstick can be positive in up to 10% of school-aged children_ Proteinuria in children can be transient
• 13 (intermittent), orthostatic, or persistent Transient proteinuria is the most common cause of proteinuria and
• 14 can be caused by fever, exercise, seizures, stress, or volume depletion_ Orthostatic proteinuria is very
• 15 common in adolescent boys and is defined as increased protein when the patient is in an upright position that
• 16 returns to normal when the patient is recumbent If the urinalysis shows no hematuria and is otherwise
• 17 normal, the urine dipstick should be repeated on at least two additional specimens (Choice A). If these
• 18 subsequent tests are negative for protein, the diagnosis is transient proteinuria. Transient and orthostatic
• 19 proteinurias are usually benign conditions that require no further evaluation_
• 20
• 21 If the proteinuria persists on the repeat sample or if any of the initial studies are abnormal, the patient should
• 22 be referred to a pediatric nephrologist and evaluated for underlying renal disease_ Further investigation may
• 23 include 24-hour urinary collection for protein, renal ultrasound, and, possibly, renal biopsy (Choices C, 13, and
• 24 F). This patient presents with proteinuria during a febrile illness and should be tested again in the future to rule
• 25 out persistent proteinuria.
• 26
• 27 (Choice B) Checking serum protein (albumin) has little value in evaluating proteinuria since many patients
• 28 with even nephrotic-range proteinuria have normal serum protein levels.

Educational objective:
• 30
Transient proteinuria is the most common cause of isolated proteinuria in children and should be reevaluated
• 31
with a repeat urine dipstick testing on two separate occasions to rule out persistent proteinuria, which requires
• 32
further evaluation for underlying renal disease_
• 33
• 34
• 35 References:
• 36 1. Isolated proteinuria: analysis of a school-age population
• 37
• 38
• 39
Copyright © LNVorld Last updated: [11/4/2014]
• 40
• 41
• 42 End Block
Feedback
• 1
• 2 Item: 30 of 44 F' Mark
▪ 3 Previous Next Lab Values. Notes Calculator.
• 4
▪ 5
▪ 6 A newborn girl is evaluated in the neonatal intensive care unit for cyanosis. She was born at 36 weeks
▪ 7 gestation by cesarean section for fetal distress_ The girl remained hypoxic despite administration of
▪ 8 supplemental oxygen. On examination, she is cyanotic and tremulous. She has low-set ears, micrognathia,
▪ 9 and a cleft palate. Chest radiograph shows an absent thymus.
• 10
• 11
• 12
• 13
• 14
• 15
• 16
• 17
• 18
• 19
• 20
• 21
• 22
• 23
• 24
• 25
• 26
- 27
- 28
▪ 29
• 30
• 31
• 32
33
• 34
• 35
• 36
• 37
▪ 38
• 39
• 40
se
• 41
• 42 End Block
Feedback
• 1
• 2 Item: 30 of 44 F' Mark
▪ 3 Previous Next Lab Values. Notes Calculator.
• 4-
▪ 5
▪ 6
▪ 7
▪ 8
▪ 9
• 10
• 11
• 12
• 13
• 14
• 15 Echocardiography confirms truncus arteriosus_ This patient is at greatest risk for which of the following?
• 16
• 17 A. Duodenal atresia [10%]
• 18
B. Hypocalcemia [7€9C]
• 19
• 20 C. Hypoglycemia [2%]
• 21 ID. Infertility [4%]
• 22
E Myelomeningocele [3%]
• 23
• 24 • Neutropenia [5%]
• 25 G. Thrombocytopenia [1a4]
• 26
- 27
- 28 Explanation:
▪ 29
• 30
• 31 DiGeorge syndrome
• 32
33
• 34
35
• Chromosome 22q11,2 deletion
Pathogenesis
36 • Defective development of pharyngeal pouches
37
38
• 39 • Conotruncal cardiac defects
• 40
se
• 41
• 42 End Block
Feedback
• 1
• 2 Item: 30 of 44 r Mark
▪ 3 Previous Next Lab Values. Notes Calculator.
• 4-
▪ 5
▪ 6 DiGeorge syndrome
▪ 7
▪ 8
▪ 9 • Chromosome 2411.2 deletion
• 10 Pathogenesis
• 11
• Defective development of pharyngeal pouches
• 12
• 13
• 14 • Conotruncal cardiac defects
• 15
• Abnormal facies
• 16 Clinical
• 17 • Thymic aplasiaihypoplasia
• 18
features
• Cleft palate
• 19
• 20 • Hypocalcemia
• 21
• 22 L6MLEWorld, LLC
• 23
• 24 DiGeorge syndrome (DOS) is disorder usually caused by the microdeletion of chromosome 2201, resulting
• 25 in poor development of the pharyngeal pouch system and subsequent abnormalities of the face, neck, and
• 26 mediastinum. The clinical presentation typically includes the pentad of congenital heart disease, facial
- 27 dysmorphia, thymic hypoplasia, cleft palate, and hypocalcemia
- 28
▪ 29 Once DOS is suspected, serum calcium levels and echocardiography must be ordered immediately_
• 30 Hypocalcemia results from hypoplasia of the parathyroid glands_ Neonates are at high risk for life-threatening 1
• 31 tetany, seizures, and arrhythmias that can further exacerbate co-existing heart problems and may require
• 32 aggressive calcium repletion. Echocardiography is critical as the cardiac anomalies in DOS are variable and
33 place the patient at great risk for heart failure and hypoperfusion. Truncus arteriosus is strongly associated
• 34 with DGS, and other common conditions include tetralogy of Fallot, interrupted aortic arch, and septal defects.
35
36 Depending on the degree of thymic hypoplasia, patients can have T-cell lymphopenia and increased risk of
37 viral and fungal infections_ Humoral immunodeficiency can also result from defective T-cell help in B-cell
38 activation for antibody production, increasing susceptibility to bacterial infections as well_ Therefore, all
• 39 patients with DOS should receive routine killed or subcomponent vaccines_ However, the safety of live
• 40 vaccines (measles-mumps-rubella, intranasal influenza, rotavirus, and oral polio virus vaccines) for these
• 41
• 42 End Block
Feedback
• 1
• 2 Item: 30 of 44 V-Mark
▪ 3 Previous Next Lab Values. Notes Calculator.
• 4- A
DiGeorge syndrome (DGS) is disorder usually caused by the microdeletion of chromosome 2201, resulting
▪ 5 in poor development of the pharyngeal pouch system and subsequent abnormalities of the face, neck, and
▪ 6 mediastinum. The clinical presentation typically includes the pentad of congenital heart disease, facial
▪ 7 dysmorphia, thymic hypoplasia, cleft palate, and hypocalcemia.
▪ 8
▪ 9 Once DOS is suspected. serum calcium levels and echocardiography must be ordered immediately.
• 10 Flypocalcemia results from hypoplasia of the parathyroid glands. Neonates are at high risk for life-threatening
• 11 tetany, seizures, and arrhythmias that can further exacerbate co-existing heart problems and may require
• 12 aggressive calcium repletion. Echocardiography is critical as the cardiac anomalies in DGS are variable and
• 13 place the patient at great risk for heart failure and hypoperfusion. Truncus arteriosus is strongly associated
• 14 with DGS, and other common conditions include tetralogy of Fallot, interrupted aortic arch, and septal defects.
• 15
• 16 Depending on the degree of thymic hypoplasia, patients can have T-cell lymphopenia and increased risk of
• 17 viral and fungal infections_ Humoral immunodeficiency can also result from defective T-cell help in B-cell
• 18 activation for antibody production, increasing susceptibility to bacterial infections as well. Therefore, all
• 19 patients with DGS should receive routine killed or subcomponent vaccines_ However, the safety of live
• 20 vaccines (measles-mumps-rubella, intranasal influenza, rotavirus, and oral polio virus vaccines) for these
• 21 patients depends on the degree of immunodeficiency_ Decisions regarding administration should be tailored
• 22 to the patient's immune status.
• 23
• 24 (Choice A) Congenital heart disease and low-set ears are seen in Down syndrome. These patients are at
• 25 increased risk of duodenal atresia. However, DGS is not associated with intestinal malformations.
• 26
(Choice C) Patients with DGS do not have problems with glycemic control. However, hypoglycemia can be
- 27
seen in infants of diabetic mothers and in disorders of glycogen metabolism or gluconeogenesis.
- 28
- 29
(Choice 0) Congenital heart disease (especially coarctation of the aorta) and ovarian failure are common
• 30
manifestations of Turner syndrome. Infertility is not a feature of DOS.
31
32 (Choice E) Folic acid antagonists (eg, phenytoin, methotrexate) increase the risk of neural tube defects and
33 possibly cardiac anomalies and oral clefts. However, neural tube defects are not associated with DOS.
34
35 (Choice F) Lymphopenia. rather than neutropenia. is a feature of DGS.
36
37 (Choice G) Patients with congenital rubella syndrome can have congenital heart disease and
38 thrombocytopenia. However, patients with DOS generally do not have platelet abnormalities.
• 39
• 40 Educational objective:
• 41
• 42 End Block
Feedback
• 1
• 2 Item: 30 of 44 F'Mark
▪ 3 Previous Next Lab Values. Notes Calculator.
• 4 with Lg..3, and other common conditions include tetralogy of allot, interrupted aortic arch, and septa) detects_
▪ 5
▪ 6 Depending on the degree of thymic hypoplasia, patients can have T-cell lymphopenia and increased risk of
▪ 7 viral and fungal infections_ Humoral immunodeficiency can also result from defective T-cell help in B-cell
▪ 8 activation for antibody production, increasing susceptibility to bacterial infections as well. Therefore, all
▪ 9 patients with DGS should receive routine killed or subcomponent vaccines_ However, the safety of live
• 10 vaccines (measles-mumps-rubella, intranasal influenza, rotavirus, and oral polio virus vaccines) for these
• 11 patients depends on the degree of immunodeficiency_ Decisions regarding administration should be tailored
• 12 to the patient's immune status.
• 13
(Choice A) Congenital heart disease and low-set ears are seen in Down syndrome. These patients are at
• 14
increased risk of duodenal atresia. However, DGS is not associated with intestinal malformations.
• 15
• 16
(Choice C) Patients with DGS do not have problems with glycemic control. However, hypoglycemia can be
• 17
seen in infants of diabetic mothers and in disorders of glycogen metabolism or gluconeogenesis.
• 18
• 19 (Choice ID) Congenital heart disease (especially coarctation of the aorta) and ovarian failure are common
• 20 manifestations of Turner syndrome. Infertility is not a feature of DGS.
• 21
• 22 (Choice El Folic acid antagonists (eg, phenytoin, methotrexate) increase the risk of neural tube defects and
• 23 possibly cardiac anomalies and oral clefts. However, neural tube defects are not associated with DGS.
• 24
• 25 (Choice F) Lymphopenia, rather than neutropenia, is a feature of DGS.
• 26
- 27 (Choice G) Patients with congenital rubella syndrome can have congenital heart disease and
- 28 thrombocytopenia_ However, patients with DGS generally do not have platelet abnormalities.
▪ 29
Educational objective:
• 30
Newborns with DiGeorge syndrome must be assessed immediately for potentially life-threatening
• 31
hypocalcemia. The diagnosis should be suspected in the setting of craniofacial anomalies, thymic
• 32
hypoplasia, and congenital heart disease.
33
• 34
• 35 References:
• 36 1. The spectrum of parathyroid gland dysfunction associated with the microdeletion
• 37
• 38
• 39
Copyright @ UWorld Last updated: [1018/2014]
• 40
• 41
• 42 End Block
Feedback
• -1
• 2 Item: 30 of 44 V-Mark
▪ 3 Previous Next Lab Values Notes Calculator
• 4- Media Exhibit
▪ 5
▪ 6 DiGeorge syndrome
▪ 7 o's
▪ 8
▪ 9
• 10
• -1-1
• 12
• 13
• 14
• 15
• 16
• 17
Classic location
• 18
• 19
and shape of
• 20 thymus in
• 21 newborn
• 22
("sail sign")
• 23
• 24
• 25
• 26
- 27
- 28
▪ 29
• :30
• 31
• 32
33
• 34
35
36
37 se
33
• 39
• 40
• 41
• 42 End Block
Feedback
• 1 A

• 2 Item: 30 of 44 !Mark
Previous Next Lab Values, Notes , Calculator,
• 3
• 4 Media Exhibit
• 6
▪ 6 Truncus arteriosus
▪ 7 A

• 8 Truncus arteriosus

• 10
• 11
• 12
• V
• 14 Ao r ta
• 15.
• 16
Pulmonary
• 17
drtry
• 18
• 19
• 20
• 21
• 22 Trumus
• 23 arteriosus
• 24-
• 25
• 26 Ventricular
• 27 septa!
• 2a defect
29

• 31
• 32
• 33
• 34
35
36
-47 C WAIILEwerld. LL(

38
39
40
41 0
42 Feedback End Block.
2 Item: 31 of 44 F'Mark
3 Previous Next Lab Values. Notes Calculator.
4
5
6 Differential diagnosis of delayed passage of meconium
7
8 Diagnosis Level of obstruction Meconium consistency
9
• 10 Hirschsprung
• 11 Rectosigmoid Normal
disease
• 12
• 13
Meconium ileus Ileum Inspissated
• 14
• 15 OUSMLEWorld,LLC I
• 16
• 17 Meconium ileus and Hirschsprung disease (congenital aganglionic megacolon) should be considered in any
• 18 neonate who has delayed passage of meconium as 99% of full-term infants stool within 48 hours of birth.
• 19 These 2 conditions have overlapping clinical features but can usually be differentiated by the level of intestinal
• 20 obstruction and consistency of the meconium (Table).
• 21
• 22 This patient's presentation is most consistent with meconium ileus, the earliest life-threatening manifestation
• 23 of cystic fibrosis (CF). Meconium ileus is virtually diagnostic for CF. Although only 20% of patients with CF
• 24 develop meconium ileus, almost all newborns with meconium ileus have CF. The disease is most commonly
• 25 due to a mutation in the CF transmembrane conductance regulator gene, resulting in abnormal chloride and
• 26 sodium transport and resultant tenacious secretions in multiple organs. The meconium is as thick as glue
• 27 and difficult to propel, resulting in obstruction at the level of the ileum and a narrow, underused colon
• 28 I (microcolon). Although contrast enema is typically performed to assess level of bowel obstruction in stable
29 patients, the free air above the liver in this patient's x-ray was indicative of intestinal perforation and required
• 30 emergency surgery. The discovery of inspissated meconium should prompt sweat chloride testing to
confirm the diagnosis of CF
• 32
• 33 Nearly all patients with CF develop sinopulmonary disease. °pacification of all sinuses can be seen as
• 34 early as age 8 months, and patients often require surgical debridement of their sinuses during childhood and
• 35 adulthood. Chronic lung inflammation and recurrent pneumonias can progress to irreversible bronchiectasis_
• 36 Because of frequent treatment with aminoglycosides for Gram-negative infections (eg, Pseudornorras
• 37 aentginosa), 10%-50% of patients may develop sensorineural hearing loss (Choice E).
• 38
(Choices B and C) Patients with CF are no more likely to develop Alzheimer's disease or hypothyroidism
• 39
compared to the general population_ On the other hand, Hirschsprung disease is associated with Down St
• 40
• 41
• 42 End Block
Feedback
• 1
• 2 Item: 31 of 44 F' Mark
▪ 3 Previous Next Lab Values. Notes Calculator.
• 4-
▪ 5 This patient's presentation is most consistent with meconium ileus, the earliest life-threatening manifestation
▪ 6 of cystic fibrosis (CF)_ Meconium ileus is virtually diagnostic for CF Although only 20% of patients with CF
▪ 7 develop meconium ileus, almost all newborns with meconium ileus have CF The disease is most commonly
▪ 8 due to a mutation in the CF transmembrane conductance regulator gene, resulting in abnormal chloride and
▪ 9 sodium transport and resultant tenacious secretions in multiple organs. The meconium is as thick as glue
• 10 and difficult to propel, resulting in obstruction at the level of the ileum and a narrow, underused colon
• 11 (microcolon). Although contrast enema is typically performed to assess level of bowel obstruction in stable
• 12 patients, the free air above the liver in this patient's x-ray was indicative of intestinal perforation and required
• 13 emergency surgery. The discovery of inspissated meconium should prompt sweat chloride testing to
• 14 confirm the diagnosis of CF
• 15
• 16
Nearly all patients with CF develop sinopulmonary disease. Opacification of all sinuses can be seen as
• 17
early as age 8 months, and patients often require surgical debridement of their sinuses during childhood and
• 18
adulthood. Chronic lung inflammation and recurrent pneumonias can progress to irreversible bronchiectasis_
• 19
Because of frequent treatment with aminoglycosides for Gram-negative infections (eg, Pseudornonas
• 20
aeruginosa), 10%-50% of patients may develop sensorineural hearing loss (Choice E).
• 21
(Choices B and C) Patients with CF are no more likely to develop Alzheimer's disease or hypothyroidism
• 22
compared to the general population_ On the other hand, Hirschsprung disease is associated with Down
• 23
syndrome, which in turn is associated with these comorbidities_
• 24
• 25
(Choice 0) In contrast to men, who are usually infertile, only 20% women with CF have fertility problems.
• 26
Infertility is due to the combination of secondary amenorrhea from malnutrition and thick cervical mucus
• 27
obstructing sperm entry.
• 28
• 29 Educational objective:
• 30 Meconium ileus is virtually diagnostic for cystic fibrosis. Inspissated meconium is responsible for
life-threatening obstruction at the level of the ileum.
• 32
• 33
References:
• 34
• 35 1. Does presenting with meconium ileus affect the prognosis of children with cystic fibrosis?
• 36 2. Rhinosinusitis in cystic fibrosis: not a simple story.
• 37
• 38
• 39
Copyright © LIWorld Last updated: [11/23/2014]
• 40
• 41
• 42 End Block
Feedback
-1

2 Item: 31 of 44 .111M Mark FF. 11


.
3 Previous Next Lab Values Notes Calculator
4 Media Exhibit X
▪ 5
▪ 6 ezonium ileus
▪ 7
• 8
• 9
• 10
• -1-1
• 12
• 13
• 14
• 15
• 16
• 17
• 19
• 19
• 20
• 21
• 22
• 23
• 24
• 25
• 26
• 27
• 23
29
• 30

• 32
• 33
• 34
• 35
• 36
• 37
• 39
• 39
• 40
• L1-1
• 42 End Block
Feedback
• -1
• 2 Item: 31 of 44 .11M Mark 11
.

▪ 3 Previous Next Lab Values Notes Calculator


• 4- Media Exhibit
▪ 5
▪ 6 Meconium ileus
▪ 7
▪ 8
▪ 9
• 10
• 11
• 12
• 13
• 14
• 15
• 16
• 17
• 18
• 19
• 20
• 21
• 22
• 23
• 24 Contrast enema
• 25
• 26 demonstrating
• 27
microcolon
• 23
• 29
• 30

• 32
• 33
• 34
• 35
• 36
• 37 V
• 33
• 39
• 40


41
42
Feedback
a
End Block
2 Item: 31 of 44 V/lark
3 Previous Next Lab Values Notes Calculator
4- A
5
6
7

9
• 10
• -1 -1
• 12
• 13
• 14
• 15
• 16
• 17
• 18
• 19
• 20
• 21
• 22
• 23
• 24
• 25
• 26
• 27
• 28
29
• 30

• 32
• 33
• 34
• 35
• 36
• 37
• 30
• 39
• 40


41
42
4.# Feedback
a
End Block
• -1
• 2 Item: 31 of 44 .11 1, Mark 11
.

▪ 3 Previous Next Lab Values Notes Calculator


• 4- Media Exhibit
▪ 5
▪ 6 Bilateral maxillary sinusitis
▪ 7 o's
▪ 8
▪ 9
• 10
• -1-1
• 12
• 13
• 14
• 15
• 16
• 17
• 18
• 19
• 20
• 21
• 22
• 23
• 24
• 25
• 26
• 27
• 28
• 29
• 30

• 32
• 33
• 34
• 35
• 36
• 37 se
• 30
• 39
• 40
• 41
• 42 End Block
Feedback
2 Item: 32 of 44 11 V- Mark -4(1

3 Previous Next Lab Values. Notes Calculator.


4
5
6 A newborn boy is evaluated shortly after vaginal delivery for respiratory distress. He was born at 37 weeks
7 gestation to a primigravid mother who had no medical problems. Second-trimester prenatal ultrasound
8 showed bilateral hydronephrosis and oligohydramnios. His temperature is 36.7 C (98 F), pulse is 144/min,
9 and respirations are 50/min. Pulse oximetry is 84% on room air. Examination shows decreased aeration of
• 10 both lungs and intercostal retractions. The child has flattened facies, abdominal distension, and clubfeet
• 11 What is the most likely underlying diagnosis in this patient?
• 12
• 13
A. Bilateral cryptorchidism [1%]
• 14
• 15 B. Congenital diaphragmatic hernia [10%]
• 16 C. Duodenal atresia [9%]
• 17
• 18
D. Hypospadias [3%]
• 19 • E. Posterior urethral valves [68%]
• 20 H Prostatic hyperplasia [1%]
• 21
• 22
G. Wilms tumor [8%]
• 23
• 24
Explanation:
• 25
• 26
Potter sequence
• 27
• 28
• 29
Urinary tract anomaly
• 30
• 31

• 33 1
• 34
• 35 Anurialoliguria in utero
• 36
• 37
• 38
• 39
V
• 40
• 41
• 42 End Block
Feedback
• 1
• 2 Item: 32 of 44 I'Mark
3 Previous Next Lab Values. Notes Calculator.
4 A
1 Potter sequence
5
6
7
8
Urinary tract anomaly
1
9
• 10
• -1 -1
• 12
• 13 Anuriaioliguna in utero
• 14
• 15
• 16
• 17
• 18
• 19 Oligohydramnio7
• 20
• 21
• 22
• 23
• 24
• 25
• 26 Pulmonary hypoplasia Flat facies Limb deformities
• 27
• 28
• 29
LISMLEWorld, LLC
• 31h16
:1
• 31
Posterior urethral valves (PUV) are the most common cause of urinary tract obstruction in newborn
• 33 boys. Abnormal folds in the distal prostatic urethra obstruct urinary flow, resulting in progressive dilation of
• 34 the bladder, ureters and kidneys_ Prenatal ultrasonography findings of bladder distention, bilateral
• 35 hydroureters, and bilateral hydronephrosis are highly suggestive of PUV. Poor urine output in utero
• 36 results in oligohydramnios as fetal urine is a major source of amniotic fluid. Oligohydramnios in the second
• 37 trimester is ominous and associated with high perinatal mortality because normal amniotic fluid levels are
• 30 1 required for lung development Low amniotic fluid also restricts fetal movement, leading to a cascade of
• 39 physical anomalies including flat facies and limb deformities (Potter sequence).
• 40 so
• 41
• 42
Feedback End Block
• 1
• 2 Item: 32 of 44 Mark
-4(1

▪ 3 Previous Next Lab Values. Notes Calculator.


• 4 t
▪ 5
▪ 6 Pulmonary hypoplasia Flat facies Limb deformities
▪ 7
▪ 8
▪ 9
0 USMLEWorld, LLC
• 10
• 11
• 12 Posterior urethral valves (PUV) are the most common cause of urinary tract obstruction in newborn
• 13 boys. Abnormal folds in the distal prostatic urethra obstruct urinary flow, resulting in progressive dilation of
• 14 the bladder, ureters and kidneys_ Prenatal ultrasonography findings of bladder distention, bilateral
• 15 hydroureters, and bilateral hydronephrosis are highly suggestive of PUV. Poor urine output in utero
• 16 results in oligohydramnios as fetal urine is a major source of amniotic fluid. Oligohydramnios in the second
• 17 trimester is ominous and associated with high perinatal mortality because normal amniotic fluid levels are
• 18 required for lung development Low amniotic fluid also restricts fetal movement, leading to a cascade of
• 19 physical anomalies including flat facies and limb deformities (Potter sequence).
• 20
• 21 Other affected infants can present with poor urinary stream, straining with voiding, urosepsis, failure to thrive,
• 22 and renal failure. Diagnosis is confirmed by voiding cystourethrogram and cystoscopy. Treatment options
• 23 include PUV ablation and urinary diversion. Despite prenatal diagnosis and early surgical intervention, patients
• 24 are at high risk for permanent kidney damage.
• 25
(Choice A) Cr-yptorchidism is the failure of one or both testes to descend from the abdomen through the
• 26
inguinal canal(s) into the scrotum. Examination shows a hypoplastic or poorly rugated scrotum. However.
• 27
cryptorchidism would not cause obstructive uropathy and oligohydramnios_
• 28
• 29
(Choice B) Congenital diaphragmatic hernia is a life-threatening defect of the diaphragm that allows
• 30
abdominal viscera to herniate into the chest, resulting in restriction of lung development Patients can present
• 31
with respiratory distress, but on examination the abdomen appears scaphoid due to loss of the abdominal
contents into the chest. Prenatal ultrasound demonstrates a mass in the chest (abdominal contents) and
• 33
sometimes polyhydramnios from esophageal compression, making this diagnosis unlikely.
• 34
• 35 (Choice C) Fetal swallowing is responsible for amniotic fluid removal. An atretic duodenum can obstruct
• 36 amniotic fluid clearance, resulting in polyhydramnios, in contrast to the oligohydramnios seen in this patient
• 37 Another significant finding on prenatal ultrasound includes the characteristic "double bubble" sign due to
• 38 dilation of the stomach and the first part of the duodenum.
• 39
• 40 frhnir-a rll 1-11.(nnqnqrliqQ is a rnnnnnitql qhnnrrnqIihf in inshirh thin ninnilin I innthrq nninnQ nn thin lainntrql rsi ir-Fqria se
• 41
• 42 End Block
Feedback
• 1
• 2 Item: 32 of 44 F'Mark
▪ 3 Previous Next Lab Values. Notes Calculator.
• 4- are at nigh risk ror permanent Kidney damage.
▪ 5
(Choice A) Cryptorchidism is the failure of one or both testes to descend from the abdomen through the
▪ 6
inguinal canal(s) into the scrotum. Examination shows a hypoplastic or poorly rugated scrotum. However,
▪ 7
cryptorchidism would not cause obstructive uropathy and oligohydramnios_
▪ 8
▪ 9
(Choice B) Congenital diaphragmatic hernia is a life-threatening defect of the diaphragm that allows
• 10
abdominal viscera to herniate into the chest, resulting in restriction of lung development Patients can present
• 11
with respiratory distress, but on examination the abdomen appears scaphoid due to loss of the abdominal
• 12
contents into the chest Prenatal ultrasound demonstrates a mass in the chest (abdominal contents) and
• 13
sometimes polyhydramnios from esophageal compression, making this diagnosis unlikely.
• 14
• 15 (Choice C) Fetal swallowing is responsible for amniotic fluid removal. An atretic duodenum can obstruct
• 16 amniotic fluid clearance, resulting in polyhydramnios, in contrast to the oligohydramnios seen in this patient
• 17 Another significant finding on prenatal ultrasound includes the characteristic "double bubble" sign due to
• 18 dilation of the stomach and the first part of the duodenum.
• 19
• 20 (Choice DO Hypospadias is a congenital abnormality in which the penile urethra opens on the ventral surface
• 21 of the penis rather than the tip. These patients are able to urinate but may have difficulty controlling the urinary
• 22 stream.
• 23
• 24 (Choice F) Benign prostatic hyperplasia can cause obstructive uropathy at the same level as posterior
• 25 urethral valves. However, this condition is common in men age >513 and does not affect children.
• 26
• 27 (Choice G) Wilms tumor (nephroblastoma) is the most common renal malignancy in childhood. Most
• 28 children present at age 2-5 years with a large, palpable flank mass but no other symptoms. It is extremely
• 29 rare for Wilms tumor to present in utero.
• 30
• 31
Educational objective:
Posterior urethral valves are the most common cause of urinary tract obstruction in newborn boys. Classic
findings on prenatal ultrasonography include bladder distension, bilateral hydroureters, and bilateral
• 33
hydronephrosis_ Oligohydramnios from low urine production in utero can cause pulmonary hypoplasia and
• 34
postnatal respiratory distress_
• 35
• 36
• 37 References:
• 38
1. Congenital urethral anomalies in boys. Part I: posterior urethral valves.
• 39
• 40 2. Risk factors for end stage renal disease in children with posterior urethral valves. se
• 41
• 42 End Block
Feedback
1
2 Item: 32 of 44 V- Mark
3 Previous Next Lab Values. Notes Calculator.
4
5 (Choice B) Congenital diaphragmatic hernia is a life-threatening defect of the diaphragm that allows
6 abdominal viscera to herniate into the chest, resulting in restriction of lung development Patients can present
7 with respiratory distress, but on examination the abdomen appears scaphoid due to loss of the abdominal
8 contents into the chest Prenatal ultrasound demonstrates a mass in the chest (abdominal contents) and
1 sometimes polyhydramnios from esophageal compression, making this diagnosis unlikely.
9
• 10
(Choice C) Fetal swallowing is responsible for amniotic fluid removal. An atretic duodenum can obstruct
• 11
amniotic fluid clearance, resulting in polyhydramnios, in contrast to the oligohydramnios seen in this patient
• 12
Another significant finding on prenatal ultrasound includes the characteristic "double bubble" sign due to
• 13
dilation of the stomach and the first part of the duodenum.
• 14
• 15 (Choice 0) Hypospadias is a congenital abnormality in which the penile urethra opens on the ventral surface
• 16 of the penis rather than the tip. These patients are able to urinate but may have difficulty controlling the urinary
• 17 stream.
• 18
• 19 (Choice F) Benign prostatic hyperplasia can cause obstructive uropathy at the same level as posterior
• 20 urethral valves. However, this condition is common in men age >5Q and does not affect children.
• 21
• 22 (Choice G) Wilms tumor (nephroblastoma) is the most common renal malignancy in childhood. Most
• 23 children present at age 2-5 years with a large, palpable flank mass but no other symptoms. It is extremely
• 24 rare for Wilms tumor to present in utero.
• 25
• 26 Educational objective:
• 27 Posterior urethral valves are the most common cause of urinary tract obstruction in newborn boys. Classic
findings on prenatal ultrasonography include bladder distension, bilateral hydroureters, and bilateral 4
• 28
• 29 hydronephrosis_ Oligohydramnios from low urine production in utero can cause pulmonary hypoplasia and
• 31h:1 postnatal respiratory distress_
• 31
References:
• 33
1. Congenital urethral anomalies in boys. Part I: posterior urethral valves.
• 34
• 35 2. Risk factors for end stage renal disease in children with posterior urethral valves.
• 36 3. Fetal surgery for posterior urethral valves: long-term postnatal outcomes.
• 37
• 38
• 39
Copyright © UWorld Last updated: [1018/2014]
• 40
• 41
• 42
Feedback End Block
• -1
• 2 Item: 32 of 44 V/lark
▪ 3 Previous Next Lab Values Notes Calculator
• 4- Media Exhibit X
▪ 5
▪ 6 Posterior urethral valves
▪ 7
▪ 8 Posterior urethral valves
▪ 9 Normal urinary system Posterior urethral valves
• 10
• 11
• 12
• 13 Kidney Hydro-
• 14
nephrosis
• 15
• 16
• 17
• 18 Ureter Dilated
• 19 ureters
• 20
• 21
• 22 Distended
• 23 Bladder bladder
• 24
• 25
• 26
• 27
• 28
Urethra Urethra obstructed by congenital
• 29
IDUSIALEVO-dd. LL1: posterior urethral membrane
• 30
• 31

• 33
• 34
• 35
• 36
• 37
• 38
• 39
• 40
• 41
• 42 End Block
Feedback
• -1
• 2 Item: 32 of 44 V/lark
▪ 3 Previous Next Lab Values Notes Calculator
• 4- 141edia Exhibit
▪ 5
▪ 6 Cryptordiidism
▪ 7
▪ 8
▪ 9
• 10
• -1 -1
• 12
• 13
• 14
• 15
• 16
• 17
• 18
• 19
• 20
• 21
• 22
• 23
• 24
• 25
• 26
• 27
• 28
• 29
• 30
• 31

• 33
• 34
• 35
• 36
• 37
• 30
• 39
• 40
• 41
• 42 End Block
Feedback
• 1
• 2 Item: 33 of 44 .11V-Mark
▪ 3 Previous Next Lab Values. Notes Calculator.
• 4-
▪ 5
▪ 6 A 13-year-old girl is brought to the physician for evaluation of sunburn. She just returned from a week-long
▪ 7 vacation to the Bahamas with her family. For the past 2 days, the patient has had redness on her face and
▪ 8 shoulders despite applying sun protection factor 50 sunscreen just before swimming. This is her first
▪ 9 sunburn. Her parents say that "everybody tans easily" in their family. Family history is negative for skin
• 10 cancer. Physical examination shows a well-appearing adolescent with olive skin, brown eyes, and light-brown
• 11 hair. Tender, blanching erythema is present on the face and arms. There is no edema or vesicles. Which of
• 12 the following is the most appropriate recommendation?
• 13
• 14
• 15
A. Apply sun protection factor 50 sunscreen 30 minutes before sun exposure [72°4]
• 16 B. Apply topical corticosteroids to inflamed skin [4%]
• 17
C. Change to sun protection factor 75 sunscreen [11%]
• 18
• 19
D. Continue current sun protection regimen [2970]
• 20 E Form a baseline tan with a tanning bed before sun exposure [0°•0]
• 21
F. Refrain from outdoor activities on cloudless days [6%]
• 22
• 23
G. Wear light-colored clothing while outdoors [5%]
• 24
• 25
Explanation:
• 26
• 27
• 28
Sunburn
• 29
• 30
• 31 • Remain indoors between 10 AM-4 PM
• 32 • \Near protective clothing:
O Hats, pants, long-sleeved shirts
• 34 Prevention
O Tightly woven, thick, or dark-colored fabrics
• 35
• Apply sunscreen 3D minutes before sun exposure
• 36
• 37 • Avoid tanning beds
• 38
• 39 • Mild-moderate sunburn:
• 40
so
• 41
• 42 End Block
Feedback
• 2 Item: 33 of 44 r. Mark U.P
3 Previous Next Lab Values, Notes Calculator.
• 4- A

5
6 Sunburn
7 .,
8 • Remain indoors between 10 AM-4 PM
9 • Wear protective clothing:
• 10
0 Hats, pants, long-sleeved shirts
• 11 Prevention
• 12 0 Tightly woven, thick, or dark-colored fabrics
• 13 • Apply sunscreen 30 minutes before sun exposure
• 14 • Avoid tanning beds
• 15
• 16
• Mild-moderate sunburn:
• 17
• 18 0 Topical: Cool compresses, calamine lotion, aloe vera
• 19 0 Oral: NSAIDs
Treatment
• 20 • Severe sunburn: Hospitalization
• 21 0 Intravenous fluids & analgesia
• 22
0 Wound care
• 23
• 24
• 25
• Cancer:
• 26 0 Melanoma
• 27 Complications 0 Basal cell carcinoma
• 28 0 Squamous cell carcinoma
• 29
• Photoaging
• 30
• 31 LIWorld
• 32
Sunburn is an inflammatory response to excessive exposure of ultraviolet (UV) radiation. Excessive exposure
• 34 to UV light increases the risk of photoaging and skin cancer_ Regardless of skin type, everyone is at risk of
• 35 the carcinogenic effects of excessive UV exposure and could benefit from education regarding
• 36 photo-protection.
• 37
• 38 The best method of photo-protection is sun avoidance, especially during peak hours of UV intensity. If it is
• 39 impractical to stay indoors (Choice F), broad-spectrum sunscreens with sun protection factor (SPF) 15-30
• 40 or higher should be used reoularlv when outdoors Sunscreen should be =lied 15-30 minutes before sun so
• 41
• 42 End Block
Feedback
2 Item: 33 of 44 V-Mark
3 Previous Next Lab Values. Notes Calculator.
4 to UV light increases the risk of photoaging and skin cancer. Regardless of skin type, everyone is at risk of
5 the carcinogenic effects of excessive UV exposure and could benefit from education regarding
6 photo-protection.
7
The best method of photo-protection is sun avoidance, especially during peak hours of UV intensity. If it is
9 impractical to stay indoors (Choice F), broad-spectrum sunscreens with sun protection factor (SPF) 15-30
• 10 or higher should be used regularly when outdoors. Sunscreen should be applied 15-30 minutes before sun
• 11 exposure to allow the formation of a protective film on the skin and reapplied at least every 2 hours.
• 12 Because sunscreen is washed off with swimming and sweating, reapplication is needed even for products
• 13 labeled "water-resistant" or "very water-resistant." However, patients should be reminded that sunscreen
• 14 alone does not fully protect against sun damage_ In addition, patients should be informed that cloud
• 15 coverage does not block UV rays and that UV rays can be reflected off water, sand, snow, and concrete.
• 16 As a result. patients can burn in the shade or while skiing in the winter.
• 17
• 18 Sunscreen is generally avoided in infants age < months because of their thin skin and high surface
• 19 area-to-body weight ratio, which increases exposure to sunscreen chemicals. However, a small amount of
• 20 sunscreen can be used if sun exposure is unavoidable and additional protection is necessary.
• 21
(Choice B) Topical corticosteroids are commonly used to soothe sunburns, but there is little evidence that
• 22
they reduce symptoms and healing time.
• 23
• 24
(Choice C) Sunscreens are rated by the strength of their SPF. The amount of UVB radiation filtered by SPF
• 25
15, 30, and 50 sunscreens is 93%, 97%, and 98%, respectively. Sunscreens with SPF >50 provide a
• 26
negligible increase in UV protection (eg, SPF 100 sunscreen filters 99% of UVB rays).
• 27
• 28 (Choice 0) This patient has been applying sunscreen just before swimming. The family should be educated
• 29 about applying sunscreen 15-30 minutes prior to sun exposure.
• 30
1.
• 31
11= (Choice E) "Fre-tanning" with tanning beds can increase skin pigmentation but does not protect against
• 32 sunburn. In fact, tanning bed use in teens and young adults is associated with a 75% increase in melanoma.

• 34 (Choice G) Although dark fabrics attract more heat than light fabrics, they actually offer greater UV protection
• 35 because the rays cannot penetrate the fabric as well.
• 36
• 37 Educational objective:
• 38 Sun avoidance is the best form of photo-protection_ Sunscreen with sun protection factor 15-30 or higher
• 39 should be applied 15-30 minutes prior to sun exposure to allow time for development of a protective film.
V
• 40 r
• 41
• 42 End Block
Feedback
• 1
• 2 Item: 33 of 44 F Mark
▪ 3 Previous Next Lab Values. Notes Calculator.
• 4- alone does not fully protect against sun damage_ In addition, patients should be informed that cloud
▪ 5 coverage does not block UV rays and that UV rays can be reflected off water, sand, snow, and concrete.
▪ 6 As a result. patients can burn in the shade or while skiing in the winter.
▪ 7
▪ 8 Sunscreen is generally avoided in infants age < months because of their thin skin and high surface
▪ 9 area-to-body weight ratio, which increases exposure to sunscreen chemicals. However, a small amount of
• 10 sunscreen can be used if sun exposure is unavoidable and additional protection is necessary.
• 11
(Choice B) Topical corticosteroids are commonly used to soothe sunburns, but there is little evidence that
• 12
they reduce symptoms and healing time.
• 13
• 14
(Choice C) Sunscreens are rated by the strength of their SPF. The amount of LJVB radiation filtered by SPF
• 15
15, 30, and 50 sunscreens is 93%, 97%, and 98%, respectively. Sunscreens with SPF >50 provide a
• 16
negligible increase in UV protection (eg, SPF 100 sunscreen filters 99% of UVB rays).
• 17
• 18 (Choice 0) This patient has been applying sunscreen just before swimming_ The family should be educated
• 19 about applying sunscreen 15-30 minutes prior to sun exposure.
• 20
• 21 (Choice El "Pre-tanning' with tanning beds can increase skin pigmentation but does not protect against
• 22 sunburn. In fact, tanning bed use in teens and young adults is associated with a 75% increase in melanoma
• 23
• 24 (Choice G) Although dark fabrics attract more heat than light fabrics, they actually offer greater UV protection
• 25 because the rays cannot penetrate the fabric as well.
• 26
• 27 Educational objective:
• 28 Sun avoidance is the best form of photo-protection. Sunscreen with sun protection factor 15-30 or higher
• 29 should be applied 15-30 minutes prior to sun exposure to allow time for development of a protective film.
• 30
• 31 References:
• 32
1. The association of use of sunbeds with cutaneous malignant melanoma and other skin
cancers: A systematic review.
• 34
• 35 2. Ultraviolet radiation: a hazard to children and adolescents.
• 36 3. Photoprotection.
• 37
• 38
• 39
Copyright © UWorld Last updated: [9/23/2014]
• 40
• 41
• 42 End Block
Feedback
1
• 2 Item: 34 of 44 V-Mark
3 Previous Next Lab Values. Notes Calculator.
• 4-
5
6 A 10-year-old patient with sickle cell disease comes to the physician for a routine visit The patient has a
7 history of multiple pain crises and pneumonias. He was started on hydroxyurea one year ago and has not had
8 any further pain crises since then. He has had no recent illnesses or hospitalizations. His physical
9 examination is unremarkable. The patient's laboratory results are shown below.
• 10
Complete blood count
• 11
Hemoglobin 9.0 WI_
• 12
Erythrocyte count 21 mlnimm
• 13
• 14
MCHC 32%
• 15
MCV 105 fl
• 16
Reticulocytes 2_0%
• 17 Platelets 212,0001mm3
• 18 Leukocyte count 9500/mm3
• 19 Neutrophils 56%
• 20 Eosinophils 3%
• 21 Lymphocytes 36%
• 22 Monocytes 5%
• 23
• 24 Which of the following best describes the role of hydroxyurea in the treatment of patients with sickle cell
• 25 disease?
• 26
• 27
A. Removes sickled red blood cells from the circulation [3%]
• 28
• 29 B. Lyses microthrombi in the circulation [3%]
• 30 C. Protects against encapsulated bacterial infections [11C]
• 31
• ID. Increases fetal hemoglobin [91%]
• 32
• 33 E Chelates iron to prevent iron toxicity [1%]

• 35
Explanation:
• 36
• 37 Hydroxyurea is a chemotherapy agent used in sickle cell disease to decrease vaso-occlusive pain crises as
• 38 well as episodes of acute chest syndrome. Hydroxyurea works by increasing the amount of fetal hemoglobin
• 39 in the circulation. Increasing the fetal hemoglobin dilutes the number of sickled cells in the circulation and
• 40
v
• 41
• 42 End Block
Feedback
2 Item: 34 of 44 V-Mark
3 Previous Next Lab Values. Notes Calculator.
4 A
5
A. Removes sickled red blood cells from the circulation [3%]
6
7
B. Lyses microthrombi in the circulation [3%]
8 C. Protects against encapsulated bacterial infections [1561
9
D. Increases fetal hemoglobin [91%]
• 10
• 11
E Chelates iron to prevent iron toxicity [196]
• 12
• 13
Explanation:
• 14
• 15 Hydroxyurea is a chemotherapy agent used in sickle cell disease to decrease vaso-occlusive pain crises as
• 16 well as episodes of acute chest syndrome. Hydroxyurea works by increasing the amount of fetal hemoglobin
• 17 in the circulation. Increasing the fetal hemoglobin dilutes the number of sickled cells in the circulation and
• 18 reduces vaso-occlusive episodes. Hydroxyurea has been shown to decrease pain crises, the need for
• 19 transfusions, and episodes of acute chest syndrome. Side effects occur because hydroxyurea suppresses
• 20 the bone marrow. Leukopenia, anemia, and thrombocytopenia may occur. These effects are generally
• 21 temporary and reversible but may predispose the patient to infection.
• 22
• 23 (Choice A) Hydroxyurea does not remove sickled cells from the circulation. The spleen is responsible for
• 24 filtering out the sickled red blood cells.
• 25
• 26 (Choice B) Microthrombi caused by the sickled cells are the cause for vaso-occlusive pain crises.
• 27 Hydroxyurea does not prevent or lyre these microthrombi, but decreases the proportion of sickled cells in the
• 28 circulation_
• 29
• 30 (Choice C) Prophylactic penicillin is given to sickle cell patients to prevent infection with encapsulated
31
irm organisms.
• 32
(Choice E) Patients with sickle cell disease often require transfusions. Frequent transfusions may lead to
• 33
iron toxicity. Patients can be treated with iron chelators such as deferoxamine to help reduce the effects of
e the transfusion related hemosiderosis_
• 35
•• 36 I
Educational objective:
• 37
Hydroxyurea benefits patients with sickle cell disease by increasing fetal hemoglobin.
• 38
• 39
Copyright © LIWorld Last updated: [10/24/2014]
• 40
• 41
• 42 End Block
Feedback
1
• 2 Item: 35 of 44 V-Mark U.P
3 Previous Next Lab Values, Notes Calculator.
• 4-
5
6 A 17-year-old girl is admitted to the hospital for purging behavior, weight loss, and syncope. She is started on
7 nasogastric feeds and her activities are restricted. On the second day of admission, the patient complains of
8 severe shortness of breath and has recurrent episodes of nonsustained ventricular tachycardia. Her blood
9 pressure is 82/55 mm Hg, heart rate is 11 /min, and respirations are 22/min. Her body mass index is 14
• 10 kg/m2. Physical examination shows an emaciated girl with bibasilar crackles and jugular venous distension_
• 11 Serum chemistry results are as follows:
• 12
Sodium 138 mEq/L
• 13
Potassium 2_1
• 14
Chloride 92 mEgil_
• 15
Bicarbonate 28 mail
• 16
Blood urea nitrogen 14 mg/dL
• 17
Creatinine 0_8 mg/dL
• 18
Calcium 8 mg/dL
• 19
Glucose 90 mgldL
• 20
Magnesium 1 mg/dL
• 21
Phosphorus 0_9 mg/dL
• 22
• 23
A surge in which of the following hormones best explains this patient's deterioration?
• 24
• 25
• 26 A. Aldosterone [27%]
• 27 Cortisol [18%]
• 28
C. Glucagon [5%]
• 29
• 30 ID. Insulin [42%)]
• 31 E Triiodothyronine [8%]
• 32
• 33
Explanation:

• 36
• 37 Pathogenesis of refeeding syndrome
• 38
• 39
• 40 I ORM .1.11 NA All ARAI 11.101 ; Oh
se
• 41
• 42 End Block
Feedback
• 1
• 2 Item: 35 of 44 F'Mark
3 Previous Next Lab Values. Notes Calculator.
4 Pathogenesis of refeeding syndrome
5
6
7 Starvation: Clinical manifestations
8
catabolic state • Arrhythmia
9
• 10
• Congestive heart failure
• 11 (pulmonary edema,
• 12 peripheral edema)
1 Insulin
• 13 • Seizures
Glucagon
• 14 • Wernicke encephalopathy
Cortisol
• 15
• 16
• 17
• 18
1 Ketone bodies 1. Serum phosphorus,
• 19
use in muscle potassium, magnesium
• 20
• 21 ¶ Ketone bodies j, Serum thiamine
• 22 use in brain Sodium & water retention
• 23
• 24
• 25
• 26 Glycogenolysis I Glycogen synthesis
• 27 t Lipolysis I Protein synthesis
• 28 Intracellular uptake of
t Protein catabolism
• 29
phosphorus, potassium,
• 30
magnesium & thiamine
• 32
Depletion of fat, protein,
• 33
vitamins, minerals &
• 34
intracellular electrolytes 1 Insulin
L
• 36
• 37
• 30 I Start refeeding:
• 39 anabolic state
• 40
v
• 41
• 42 End Block
Feedback
• 1
• 2 Item: 35 of 44 IM V-• Mark U.P
3 Previous Next Lab Values, Notes Calculator.
11.111..
• 4 bian refeeding.
5 anabolic state
6
7 USIPALEWorkl, LLC
8
9 Chronic starvation and acute refeeding are potentially life-threatening in patients with anorexia nervosa_
• 10 Refeeding syndrome is the constellation of pathologic derangements resulting from a surge in insulin activity
• 11 as the body resumes anabolism (Flow chart). Carbohydrate ingestion, whether enteral or intravenous,
• 12 causes pancreatic insulin secretion and cellular uptake of phosphorus, potassium, and magnesium.
• 13 Phosphorus is the primary deficient electrolyte as it is required for energy (adenosine triphosphate).
• 14 Deficiencies in potassium and magnesium potentiate cardiac arrhythmias in a heart that is already atrophic
• 15 from prolonged malnutrition. Therefore, aggressive initiation of nutrition without adequate electrolyte repletion
• 16 can quickly precipitate cardiopulmonary failure. This patient has signs of volume overload from heart failure
• 17 based on increased pulse and respiratory rate, jugular venous distension, and bibasilar lung crackles_
• 18
• 19
(Choices A and 6) Aldosterone and cortisol stimulate water resorption, sodium conservation, and potassium
• 20
secretion in an attempt to maintain blood pressure. These steroid hormones are high during starvation but
• 21
less so during refeeding_ In addition, they do not directly impact phosphorus or magnesium uptake.
• 22
(Choice C) Glucagon activity is elevated during starvation but decreased during refeeding.
• 23
• 24
(Choice E) Patients with anorexia nervosa have euthyroid hypothyroxinemia, which is characterized by
• 25
normal thyroid-stimulating hormone and normal to decreased serum thyroxine and triiodothyronine_
• 26
• 27 Educational objective:
• 28 Refeeding syndrome is a potentially fatal complication of nutritional rehabilitation in anorexia nervosa.
• 29 Carbohydrate intake stimulates insulin activity, which in turn promotes cellular uptake of phosphorus,
• 30 potassium. and magnesium. Clinical manifestations include arrhythmias and cardiopulmonary failure.
• 31
• 32
References:
• 33
1. Nutritional rehabilitation: practical guidelines for refeeding the anorectic patient.
2. Hypophosphatemia during nutritional rehabilitation in anorexia nervosa: implications for
• 36 refeeding and monitoring.
• 37
• 38
• 39
Copyright © LIWorld Last updated: [11/18/2014]
• 40
• 41
• 42 End Block
Feedback
1
• 2 Item: 36 of 44 F Mark
3 Previous Next Lab Values. Notes Calculator.
• 4-
5
6 A 4-year-old boy is brought to physician for evaluation of a rash. A few days ago, the boy had a few small red
7 bumps on his chin and cheek. The bumps became painful as they filled with fluid. The patient has no other
8 medical problems and his vaccinations are up to date. He has been playing at the park daily and has had
9 occasional insect bites_ Vital signs are normal. Physical examination shows multiple pustules on his right
• 10 cheek, nose, and chin. Some lesions are covered with a thick golden-yellow crust The rest of his
• 11 examination is within normal limits. What is the most appropriate next step in the management of this patient?
• 12
• 13
A. Anti-streptolysin 0 antibody titers [10%]
• 14
• 15 B. Express pus for culture [8%]
• 16 C. Oral cephalexin [29%]
• 17
ID. Swab intact skin for culture [3%]
• 18
• 19 • E Topical mupirocin [50%]
• 20
• 21
Explanation:
• 22
• 23
• 24
• 25
Impetigo
• 26
• 27
Type Non-bullous Bullous
• 28
• 29
• 30 Staphylococcus aureus
• 31 Microbiology &/or Streptococcus Staphylococcus aureus
• 32 pyogenes
• 33
• 34
• 35 Rapidly enlarging flaccid bullae with
Clinical Painful non-itchy pustules
36 yellow fluid; ucollarette" of scale at
features & honey-crusted lesions
• 37 the periphery of ruptured lesions
• 38
. .
• 39
• 40 Tre..n.le-rieLe-.1.
Topical antibiotics (eg, Oral antibiotics (eg, cephalexin, N
• 41
• 42 End Block
Feedback
• 1
• 2 Item: 36 of 44 V- Mark
▪ 3 Previous Next Lab Values. Notes Calculator.
• 4-
▪ 5
▪ 6
Impetigo
▪ 7
▪ 8
Type Non-bullous Bullous
▪ 9
• 10
• 11 Staphylococcus aureus
• 12 Microbio logy 8Lior Streptococcus Staphylococcus aureus
• 13 pyogenes
• 14 I
• 15
• 16 Rapidly enlarging flaccid bullae with
Clinical Painful non-itchy pustules
• 17 yellow fluid; uccillarette" of scale at
features & honey-crusted lesions
• 18 the periphery of ruptured lesions
• 19
• 20
• 21
Topical antibiotics (eg, Oral antibiotics (eg, cephalexin,
Treatment
• 22 mupirocin) didoxacillin, or ciindamycin)
• 23
QUSMLEWOrkl, LLC
• 24
• 25
The patient's rash is consistent with localized non-bullous impetigo_ Impetigo is a common pediatric rash that
• 26
is typically caused by Staphylococcus aureus or group A beta-hemolytic Streptococcus (Streptococcus
• 27
pyogerres). Predisposing factors include warm and humid climate, poverty, crowding, poor personal hygiene,
• 28
pre-existing skin trauma (eg, insect bite), and atopic dermatitis (eczema). Colonization with staphylococci
• 29
or streptococci is also a risk factor.
• 30
• 31 The superficial skin infection manifests with multiple painful pustules on the exposed areas of the face and
• 32 extremities_ aver the course of a week, the pustules eventually rupture and harden into a characteristic
• 33 golden-yellow ("honey") crust Local lymphadenopathy can be present but fever and chills are unusual.
• 34
• 35 Antibiotics are indicated to reduce infection transmission and recovery time. Topical antibiotics (eg,
36 mupirocin) are preferred due to less side effects and antibiotic resistance risk compared to oral therapy_
• 37 Thorough handwashing is important to prevent the spread of this contagious infection.
• 38
• 39 (Choices A and B) Impetigo diagnosis is based on clinical presentation. Culture of pustular fluid can be
• 40 nonsidprp.d for natip.nts whn fail pninirin thpranv Antibody titprs r.an takp wp.p.ks to riso and am np.nprallv not
• 41
• 42 End Block
Feedback
• 1
• 2 Item: 36 of 44 F' Mark
▪ 3 Previous Next Lab Values. Notes Calculator.
• 4- I ne patients rasn IS consistent vim localizeo non-ounous impetigo_ impetigo is a common peal-al-sic rasn mat
▪ 5 is typically caused by Staphylococcus aureus or group A beta-hemolytic Streptococcus (Streptococcus
▪ 6 pyogenes). Predisposing factors include warm and humid climate, poverty, crowding, poor personal hygiene,
▪ 7 pre-existing skin trauma (eg, insect bite), and atopic dermatitis (eczema). Colonization with staphylococci
▪ 8 or streptococci is also a risk factor.
▪ 9
The superficial skin infection manifests with multiple painful pustules on the exposed areas of the face and
• 10
• 11
extremities. Over the course of a week, the pustules eventually rupture and harden into a characteristic
golden-yellow ("honey") crust Local lymphadenopathy can be present but fever and chills are unusual.
• 12
• 13
Antibiotics are indicated to reduce infection transmission and recovery time. Topical antibiotics (eg,
• 14
mupirocin) are preferred due to less side effects and antibiotic resistance risk compared to oral therapy_
• 15
Thorough handwashing is important to prevent the spread of this contagious infection.
• 16
• 17 (Choices A and El) Impetigo diagnosis is based on clinical presentation. Culture of pustular fluid can be
• 18 considered for patients who fail empiric therapy. Antibody titers can take weeks to rise and are generally not
• 19 useful in acute illness_ Anti-streptolysin 0 titers can be considered for patients who develop complications
• 20 such as post-streptococcal glomerulonephritis and rheumatic fever.
• 21
• 22 (Choice C) Bullous impetigo is characterized by flaccid bullae containing yellow fluid. It is usually caused by
• 23 S aureus and should be treated with oral cephalexin, dicloxacillin, or clindamycin. Oral antibiotics are also
• 24 indicated when topical therapy is impractical for widespread non-bullous impetigo.
• 25
• 26 (Choice 13) Swabs of intact skin are not useful as skin flora would be detected.
• 27
• 28 Educational objective:
• 29 Non-bullous impetigo is characterized by painful pustules and honey-crusted lesions; diagnosis is clinical.
• 30 Topical mupirocin is the treatment of choice.
• 31
• 32 References:
• 33
1. Bacterial resistance and impetigo treatment trends: a review.
• 34
• 35 2. Impetigo - review.
36
3. Treatment of impetigo: oral antibiotics most commonly prescribed.
• 37
• 38
• 39
Copyright © UWorld Last updated: [8/2G/2014]
• 40
• 41
• 42 End Block
Feedback
• 1
• 2 Item: 37 of 44 .11M Mark U.P
▪ 3 Previous Next Lab Values, Notes Calculator.
• 4
▪ 5
▪ 6 An 11-month-old girl is brought to the physician for fever. For the past few days, she has had rhinorrhea and
▪ 7 nasal congestion. The fever started 2 days ago and the girl has since been pulling at both ears. The ear pain
▪ 8 seems worse when she lies down. The girl has no medical problems and takes no medications. She attends
▪ 9 day care and her diet consists of infant formula and finger foods. Both parents smoke cigarettes. Her
• 10 temperature is 38.9 C (102 F). Otoscopy shows bulging, erythematous bilateral tympanic membranes with
• 11 decreased mobility on air compression. Crusted rhinorrhea is present at the nares. Her hearing is intact and
• 12 the rest of her examination is normal_ Which of the following is the most appropriate next step in
• 13 management of this patient?
• 14
• 15
• 16
A. Acetaminophen, follow-up in 2 days [9%]
• 17 Myringotomy with tympanostomy tube placement [3%]
• 18
▪ C. Oral antibiotics [80%]
• 19
• 20
ID. Ototopical antibiotics [4%]
• 21 E Tympanocentesis and culture [2°/0]
• 22
E Viral nasopharyngeal polymerise chain reaction testing [11.6]
• 23
• 24
• 25 Explanation:
• 26
• 27 Acute otitis media
• 28
• 29
• 30
• Streptococcus pneurnoniae
• 31 Microbiology • Nontypeable Haemophilus influenzae
• 32 • Moraxella catarrhafis
• 33
• 34
• 35 • Middle ear effusion
• 36
Clinical plus
a 37 features
• Bulging tympanic membrane
• 38
• 39
• 40 _ 1..:4;,..1- A ..,,‘......;.,.:11;..-.

• 41
• 42 End Block
Feedback
• 1
• 2 Item: 37 of 44 Al I Mark
▪ 3 Previous Next . Lab Values. Notes Calculator.
• 4
▪ 5 Acute otitis media
▪ 6
▪ 7
▪ 8
• Streptococcus pneumomae
▪ 9 Microbiology • Nontypeable Haeinophilus int7uenzae
• 10 • Moraxella catarrhaiis
• 11
• 12
• 13 • Middle ear effusion
Clinical
• 14 plus
• 15
features
• Bulging tympanic membrane
• 16
• 17
• 18
• Initial: Amoxicillin
• 19 Treatment
• 2nd line Amoxicillin-clavulanic acid
• 20
• 21
• 22 • Conductive hearing loss
• 23
Complications • Mastoiditis
• 24
• Meningitis
• 25
• 26 cif USMLEWorldr LLC
• 27
• 28
This patient's clinical presentation is consistent with acute otitis media (ADM). AOM is an extremely common
• 29
condition in children age 6-36 months as their Eustachian tubes are short and easily clogged. Risk factors
• 30
include formula intake (rather than breast milk), exposure to cigarette smoke, allergic rhinitis or viral upper
• 31
respiratory infection, craniofacial anomalies, and chronic middle ear effusion.
• 32
• 33
The most common offending pathogens include Streptococcus pneurnoniae, nontypeable Haemopbllus
• 34
influenzae, and Moraxelta catarrhafis_ The first-line treatment is a 1D-day course of high-dose amoxicillin. If
• 35
AOM returns within a month of initial treatment, amoxicillin-clavulanic acid should be given in anticipation of
• 36
infection with beta-lactamase-resistant strains. Potential complications of recurrent AOM are numerous and
include chronic suppurative otitis media, mastoiditis, labyrinthitis, cholesteatoma, tympanosclerosis,
111•FMI eardrum perforation, and conductive hearing loss.
• 39
• 40 rs Ali; ". I{5 J I .k. fr..r r..1-,41..-4 Ir., ri-
se
• 41
• 42 End Block
Feedback
1
2 Item: 37 of 44 V- Mark
3 Previous Next Lab Values. Notes Calculator.
4-
5 The most common offending pathogens include Streptococcus prreurnornae, nontypeable Haernoplidus
6 influenzae, and Moraxetta catarrhatis. The first-line treatment is a 10-day course of high-dose amoxicillin. If
7 ACM returns within a month of initial treatment, amoxicillin-clavulanic acid should be given in anticipation of
8 infection with beta-lactamase-resistant strains. Potential complications of recurrent AOM are numerous and
9 include chronic suppurative otitis media, mastoiditis, labyrinthitis, cholesteatoma, tympanosclerosis,
• 10 eardrum perforation, and conductive hearing loss.
• 11
• 12 (Choice A) Analgesia (eg, ibuprofen, acetaminophen) is appropriate for relief from otalgia, but delaying
• 13 antibiotics in infants and young children could increase the risk of developing complications. Observation is a
• 14 reasonable option if the child is age years, has a normal immune system, and symptoms are mild and
• 15 unilateral.
• 16
• 17
(Choices B and E) Empiric antibiotics are usually adequate for complete treatment of
• 18
AOM. Tympanocentesis and culture during myringotomy with tympanostomy tube placement should be
• 19
considered in children with multiple episodes of ACM (eg, L3 episodes within 6 months or L.4 episodes within
• 20
12 months) despite appropriate antibiotic treatment
• 21
(Choice 0) Therapy with otic drops is appropriate for otorrhea from tympanostomy tubes, chronic
• 22
suppurative otitis media, or external otitis, but not for AOM.
• 23
• 24
(Choice F) Although viral upper respiratory infections often precede or occur with A011. viral identification
• 25
would not change management, which consists of empiric antibiotic treatment.
• 26
• 27 Educational objective:
• 28 Acute otitis media is a common infection in infants and young children, especially with cigarette smoke
• 29 exposure, recent or concurrent upper respiratory infection, day care attendance, and formula intake. The
• 30 most common causative organisms are Streptococcus prreurnorriae, non-typeable Haernophitus influeraae,
• 31 and Moraxella catarrhafis_ Oral amoxicillin should be administered to prevent complications (eg, mastoiditis).
• 32
• 33
References:
• 34
• 35 1. Otitis media: diagnosis and treatment.
•36 2. The diagnosis and management of acute otitis media.

• 38
• 39
Copyright © LIWorld Last updated: [10/20/2014]
• 40
• 41
• 42 End Block
Feedback
• 1
• 2 Item: 37 of 44 IM V-Mark
▪ 3 Previous Next Lab Values Notes Calculator
• 4- Media Exhibit
▪ 5
▪ 6 CT - Acute Mastoiditis
▪ 7
▪ 8
▪ 9
• 10
• 11
• 12
• 13
• 14
• 15
• 16
• 17
• 18
Normal pneumatized
• 19
mastoid air cells
°pacified middle ear cavity
• 20 °pacified mastoid air cells
• 21
• 22
• 23
• 24
• 25
• 26
• 27
• 28
• 29
• 30
• 31
• 32
• 33
• 34
• 35
• 36
a 37
• 38
• 39
• 40
• 41
• 42
A
• 1
• 2 Item: 38 of 44 V-• Mark
▪ 3 Previous Next Lab Values. Notes Calculator.
• 4
5
▪ 6 A 12-year-old boy is brought to the emergency department because of severe pain near his left knee. He has
▪ 7 sickle cell disease and has been hospitalized previously for sickle cell crisis_ Vital signs are notable for
▪ 8 persistent fever. Examination of the left lower extremity reveals a normal knee joint with marked tenderness
▪ 9 and swelling over the proximal tibia. Laboratory studies show leukocytosis and elevated ESR_ Imaging
• 10 studies confirm the diagnosis of osteomyelitis. Which of the following organisms is the most likely cause of
• 11 his condition?
• 12
• 13
• 14
A. Escherichia cob. [1%]
• 15 Pseudornonas species [29C]
• 16
• C. Salmonella species [929C]
• 17
• 18
D. Streptococcus pneurnoniae [4%]
• 19 E. Group B streptococcus [1%]
• 20
• 21
• 22 Explanation:
• 23
This patient presents with clinical findings suggestive of osteomyelitis in the setting of sickle cell disease.
• 24
Because of the early decline in their splenic function, patients with sickle cell disease are more susceptible to
• 25
infections of the bone and joints_ Salmonella and Staph aureus are the most common cause of osteomyelitis
• 26
in patients with sickle cell disease. Although controversial, more recent studies have shown that S. aureus is
• 27
more common than Salmonella in children with sickle cell.
• 28
• 29
(Choices A and El E. colt and group B streptococcus are common causes of septic arthritis and
• 30
osteomyelitis in neonates_ Group B streptococcus is the most common organism responsible for
• 31
osteomyelitis in children less than one year of age.
• 32
• 33 (Choice B) Pseudomonal osteomyelitis is seen in diabetic patients and after puncture wounds of the foot and
• 34 is a less common cause of osteomyelitis in children with sickle cell_
▪ 35
• 36 (Choice 0) Sickle cell patients are more susceptible to encapsulated organisms due to abnormal function of
- 37 the spleen. and Streptococcus pneurnoniae is the most common cause of bacteremia in these children. S.
• 38 pneurnoniae typically causes meningitis and pneumonia and is a less likely cause of osteomyelitis.
- 39
- 40 Educational objective: se
▪ 41
▪ 42 End Block
Feedback
A'S
1
• 2 Item: 38 of 44 V/lark
▪ 3 Previous Next Lab Values. Notes Calculator.
• 4 14.F..le Lel! dl IU ridgy =el I I iuNmi.dilLeu reviuuNy iur 14...Kle Lel! LI VILdI iyri dl e I IULdUle IUI

▪ 5 persistent fever_ Examination of the left lower extremity reveals a normal knee joint with marked tenderness
▪ 6 and swelling over the proximal tibia Laboratory studies show leukocytosis and elevated ER_ Imaging
▪ 7 studies confirm the diagnosis of osteomyelitis. Which of the following organisms is the most likely cause of
▪ 8 his condition?
▪ 9
• 10 A. Escherichia colt [1%]
• 11
Pseudomonas species [2%]
• 12
• 13 C. Salmonella species [92%]
• 14 D. Streptococcus pneumonilae [4%]
• 15
E Group B streptococcus [1%]
• 16
• 17
• 18 Explanation:
• 19
• 20 This patient presents with clinical findings suggestive of osteomyelitis in the setting of sickle cell disease.
• 21 Because of the early decline in their splenic function, patients with sickle cell disease are more susceptible to
• 22 infections of the bone and joints_ Salmonella and Stapir aureus are the most common cause of osteomyelitis
• 23 in patients with sickle cell disease. Although controversial, more recent studies have shown that S. aureus is
• 24 more common than Salmonella in children with sickle cell.
• 25
• 26 (Choices A and E) E. colt and group B streptococcus are common causes of septic arthritis and
• 27 osteomyelitis in neonates. Group B streptococcus is the most common organism responsible for
• 28 osteomyelitis in children less than one year of age.
• 29
• 30 (Choice B) Pseudomonal osteomyelitis is seen in diabetic patients and after puncture wounds of the foot and
• 31 is a less common cause of osteomyelitis in children with sickle cell_
• 32
(Choice 0) Sickle cell patients are more susceptible to encapsulated organisms due to abnormal function of
• 33
the spleen, and Streptococcus pneurnorriae is the most common cause of bacteremia in these children. S.
• 34
pneurnorriae typically causes meningitis and pneumonia and is a less likely cause of osteomyelitis.
• 35
• 36
Educational objective:
37
Salmonella and Staph aureus are the most common cause of osteomyelitis in patients with sickle cell disease.
• 38
- 39
Copyright @ UWorld Last updated: [7/23/2014]
- 40
▪ 41
▪ 42 End Block
Feedback
• 1
• 2 Item: 39 of 44 F' Mark
▪ 3 Previous Next Lab Values. Notes Calculator.
• 4-
▪ 5
▪ 6 A 4-year-old girl is brought to the physician for a rash. She lives in Poland and arrived in the United States last
▪ 7 week to visit her family. The child was in good health until 3 days ago, when she developed nasal congestion
▪ 8 and a low-grade fever_ Today she developed a "pink rash" on her face that spread rapidly to her trunk and
▪ 9 extremities. Review of systems is otherwise negative. She completed an antibiotic course for a 'throat
• 10 infection" 2 months ago but has otherwise been healthy. She has received no vaccinations and takes no
• 11 medications. Her temperature is 38.3 C (101 F), blood pressure is 100/50 mm Hg, pulse is 88/min, and
• 12 respirations are 18/min. Physical examination shows a cooperative girl with bilateral nonexudative
• 13 conjunctivitis and patchy erythema on the soft palate. Small, tender lymph nodes are palpable in the
• 14 suboccipital, posterior auricular, and posterior cervical areas_ The nontender rash is shown below_
• 15
• 16
• 17
• 18
• 19
• 20
• 21
• 22
• 23
• 24
• 25
• 26
• 27
• 28
• 29
• 30
• 31
• 32
• 33
• 34

se

Feedback End Block


• -1
• 2 Item: 39 of 44 .111 II Mark U.P
▪ 3 Previous Next Lab Values, Notes Calculator.
• 4-
▪ 5
▪ 6
▪ 7
▪ 8
▪ 9
• 10
• -1 -1
• 12
• 13
• 14
• 15
• 16
• 17
• 18
• 19
• 20
• 21
• 22
• 23
• 24
• 25
• 26
• 27
• 28
• 29
• 30
• 31
• 32
• 33
▪ 34
• 35
• 36
• 37
• 30

• 40
41
42 End Block
Feedback
• -1
• 2 Item: 39 of 44 Mark
▪ 3 Previous Next Lab Values. Notes Calculator.
• 4-
▪ 5
▪ 6
▪ 7
▪ 8
▪ 9
• 10
• -1 -1
• 12
• 13 The remainder of the examination is normal. Which of the following is the most likely cause of this patient's
• 14 condition?
• 15
• 16 A. Erythema. multiforme [1%]
• 17
B. Exanthematous drug eruption [1%]
• 18
• 19 C. Kawasaki disease [7%]
• 20 D. Measles [35%]
• 21
E Mumps [1%]
• 22
• 23 Rubella [45%]
• 24 G. Scarlet fever [9%]
• 25 1

H. Varicella [19.6]
• 26
• 27
• 28 Explanation:
• 29
• 30
• 31 Rubella (German measles)
• 32
• 33 • Sensorineural hearing loss
• 34 Congenital • Cardiac anomalies (eg, patent ductus artenosus)
• 35
• Cataracts, glaucoma
• 36
• 37
• 38 • Low-grade fever
Clinical • Conjunctivitis, coryza, cervical
• 40 nrpnAntntinn l errurthsui nnrwtorth%i Fanrer+11-tebi r-nn r cnntc
se
• 41
42
• 1
• 2 Item: 39 of 44 Mark
▪ 3 Previous Next Lab Values. Notes Calculator.
• 4
▪ 5 Rubella (German measles)
▪ 6
▪ 7 • Sensorineural hearing loss
▪ 8
Congenital • Cardiac anomalies (eg, patent ductus arteriosus)
▪ 9
• 10
• Cataracts, glaucoma
• -1 -1
• 12 • Low-grade fever
• 13 Clinical • Conjunctivitis, coryza, cervical
• 14
presentation Children lymphadenopathy, Forschheimer spots
• 15
• 16 • Cephalocaudal spread of blanching,
• 17 erythematous maculopapular rash
• 18
• 19
Adolescents}
• 20 • Same as children + arthralgiasiarthritis
Adults
• 21
• 22
• 23 • Polymerase chain reaction
Diagnosis
• 24 • Acute & convalescent serology for anti-rubella IgM & IgG
• 25
• 26
Prevention • Live attenuated rubella vaccine
• 27
• 28 Treatment . Supportive care
• 29
USMLEWarkl, LLC
• 30
• 31
This unvaccinated child most likely has rubella (German measles), an RNA togavirus infection. Children
• 32
remain asymptomatic or develop mild disease 2-3 weeks after inhalation of infected respiratory
• 33
droplets_ An erythematous, "pink" maculopapular exanthem begins on the face, spreads quickly in a
• 34
cephalocaudal and centrifugal pattern to the rest of the body, and lasts <3 days_ A low-grade fever and
• 35
tender lymphadenopathy may precede or occur with the rash. Some patients also have a nonexudative
• 36
conjunctivitis and patchy erythema (Forschheimer spots) on the soft palate. Treatment is aimed at
• 37
symptomatic relief (eg, acetaminophen) as no specific antiviral therapy is available.
• 30

In contrast to postnatal infection, congenital disease is devastating. Vaccination is important to prevent se


• 40 •

• 41
• 42 End Block
Feedback
1
• 2 Item: 39 of 44 V-• Mark .4111°.
3 Previous Next Lab Values. Notes Calculator.
1.11.1J1JICLM_ /-1.1 I CI yLl ICH ICILUL13, irs. I I 1C14,UILIIJCIIJUICII CACI! ILI ICI I I L1 III 4,11 I LI IC I CIL,C, CCIU3 LAUIL,rtly II I CI
• 4-
5
cephalocaudal and centrifugal pattern to the rest of the body, and lasts <3 days. A low-grade fever and
tender lymphadenopathy may precede or occur with the rash. Some patients also have a nonexudative
6
conjunctivitis and patchy erythema (Forschheimer spots) on the soft palate. Treatment is aimed at
▪ 7
symptomatic relief (eg, acetaminophen) as no specific antiviral therapy is available.
▪ 8
▪ 9
In contrast to postnatal infection, congenital disease is devastating. Vaccination is important to prevent
• 10
spread of infection to nonimmune pregnant women. Pregnant women who are infected during the 1st
• 11
trimester are at extremely high risk for miscarriage or severe birth defects.
• 12
• 13 (Choice A) Erythema multiforme is characterized by the sudden onset of erythematous 'target' lesions and
• 14 usually follows herpes simplex infection.
• 15
• 16 (Choice B) Exanthematous (morbilliform) drug eruptions are erythematous and maculopapular and
• 17 sometimes cause a low-grade fever. However, the allergic rash presents within 1-2 weeks of treatment and
• 18 rarely has mucosal involvement
• 19
• 20 (Choice C) Although patients with Kawasaki disease may have high fever and mucocutaneous
• 21 abnormalities, the fever lasts days and the patient is irritable_ This patient's vaccine status, fever duration.
• 22 behavior, rash pattern, and Forschheimer spots are more suggestive of rubella
• 23
• 24 (Choice ID) Measles tends to be more severe than rubella, with high fevers up to 40 C (104 F), coryza, and
• 25 malaise_ In addition, the rash spreads more gradually and appears darker (eg, reddish-brown) in comparison
• 26 to rubella.
• 27
• 28 (Choice E) Mumps can cause a low-grade fever, but the infection causes parotitis and not a rash.
• 29
• 30 (Choice G) Scarlet fever can cause fever and a rash that spreads cephalocaudally, but the rash has a
• 31 "sandpaper" texture due to numerous tiny papules and is most pronounced in the skin folds. Conjunctivitis is
• 32 not a feature of scarlet fever_
• 33
(Choice H) Primary varicella zoster infection (chickenpox) is characterized by fever, malaise, and a diffuse.
• 34
pruritic, and painful vesicular eruption. This patient has no pustules or vesicles, making this diagnosis unlikely.
• 35
• 36
Educational objective:
• 37
Rubella infection in children is mild and characterized by low-grade fever, tender lymphadenopathy, and a
• 38
maculopapular rash that spreads cephalocaudally. Vaccination is important to prevent infection of vulnerable
pregnant women due to the risk of debilitating congenital rubella syndrome.

Feedback End Block


I
• 1
• 2 Item: 39 of 44 'Mark
▪ 3 Previous Next Lab Values. Notes Calculator.
10.0.11.411, 11 LI IC1110 111.1“1•%•V6P1 ILII L4L.LL.I ILGU Ur LI IL. il UUUGI I LOI GI LI 14-.1 I IGLU...Ail LE-11 W.-.L
• 4- A
▪ 5
usually follows herpes simplex infection.
▪ 6
(Choice B) Exanthematous (morbilliform) drug eruptions are erythematous and maculopapular and
▪ 7
sometimes cause a low-grade fever. However, the allergic rash presents within 1-2 weeks of treatment and
▪ 8
rarely has mucosal involvement
▪ 9
• 10
(Choice C) Although patients with Kawasaki disease may have high fever and mucocutaneous
• 11
abnormalities, the fever lasts days and the patient is irritable_ This patient's vaccine status, fever duration.
• 12
behavior, rash pattern, and Forschheimer spots are more suggestive of rubella.
• 13
• 14 (Choice ID) Measles tends to be more severe than rubella, with high fevers up to 40 C (104 F), coryza. and
• 15 malaise. In addition, the rash spreads more gradually and appears darker (eg, reddish-brown) in comparison
• 16 to rubella_
• 17
• 18 (Choice El Mumps can cause a low-grade fever, but the infection causes parotitis and not a rash.
• 19
• 20 (Choice G) Scarlet fever can cause fever and a rash that spreads cephalocaudally, but the rash has a
• 21 "sandpaper" texture due to numerous tiny papules and is most pronounced in the skin folds. Conjunctivitis is
• 22 not a feature of scarlet fever_
• 23
• 24 (Choice H) Primary varicella zoster infection (chickenpox) is characterized by fever, malaise, and a diffuse,
• 25 pruritic. and painful vesicular eruption. This patient has no pustules or vesicles, making this diagnosis unlikely_
• 26
• 27 Educational objective:
• 28 Rubella infection in children is mild and characterized by low-grade fever, tender lymphadenopathy, and a
• 29 maculopapular rash that spreads cephalocaudally. Vaccination is important to prevent infection of vulnerable
• 30 pregnant women due to the risk of debilitating congenital rubella syndrome.
• 31
• 32 References:
• 33
1. Rubella and congenital rubella (German measles).
• 34
2. Rubella
3. Rubella immunity and vaccination coverage of the population of northern Greece in 2006.

Copyright © LIWorld Last updated: [9/21/2014]

Feedback End Block


-1

2 Item: 39 of 44 V-Mark
3 Previous Next Lab Values Notes Calculator
4 Media Exhibit X
5
6 Erythema multforrne
7
8
9
• 10
• 11
• 12
• 13
• 14
• 15
• 16
• 17
• 18
• 19
• 20
• 21
• 22
• 23
• 24
• 25
• 26
• 27
• 28
• 29
• 30
• 31
• 32
• 33
• 34
35
36
37
30
I 39

Feedback
C
End Block.
• -1
• 2 Item: 39 of 44 V-Mark
EF. 11
.

▪ 3 Previous Next Lab Values Notes Calculator


• 4- Media Exhibit
▪ 5
▪ 6 Measles-3
▪ 7
▪ 8
▪ 9
• 10
• -1 -1
• 12
• 13
• 14
• 15
• 16
• 17
• 18
• 19
• 20
• 21
• 22
• 23
• 24
• 25
• 26
• 27
• 28
• 29
• 30
• 31
• 32
• 33
• 34
• 35
• 36
• 37
• 33
3
• 40
• 41
• 42 End Block
Feedback
• -1
• 2 Item: 39 of 44 V- Mark
▪ 3 Previous Next Lab Values Notes Calculator
• 4- Media Exhibit
▪ 5
▪ 6 Sprlet fever
▪ 7
▪ 8
▪ 9
• 10
• -1 -1
• 12
• 13
• 14
• 15
• 16
• 17
• 18
• 19
• 20
• 21
• 22
• 23
• 24
• 25
• 26
• 27
• 28
• 29
• 30
• 31
• 32
• 33
• 34
• 35
• 36
• 37
• 30

• 40


41
42
Feedback
a
End Block
• 1
• 2 Item: 40 of 44 V-• Mark
▪ 3 Previous Next Lab Values. Notes Calculator.
• 4-
5
▪ 6 A 1-year-old girl is brought to the emergency department because she is not moving her right arm. Her father
▪ 7 says that the girl acted normally when she awoke this morning. He was swinging her by the forearms when
▪ 8 she suddenly started crying and refused to use her right arm_ He has swung her by the arms previously and it
▪ 9 usually makes her laugh. The girl lives with her father and her mother, who is currently out of town_ Physical
• 10 examination shows a well-nourished child who holds her right arm in pronation against the chest She avoids
• 11 any movement of her right arm. No bruises or other abnormalities are seen. Which of the following is the
• 12 most appropriate next step in management of this patient?
• 13
• 14
A. Contact the child protection agency [1°4]
• 15
• 16 E Closed reduction and casting of the right arm [7%]
• 17 ▪ C. Gently hyperpronate the right forearm [66%]
• 18
D. Order a complete skeletal survey [2%]
• 19
• 20 E Order an x-ray of the right arm [22%]
• 21 • Refer the child to an orthopedic surgeon [11C]
• 22
• 23
• 24 Explanation:
• 25
• 26 Radial head subluxation (nursemaid's elbow)
• 27
• 28
Axial traction on forearm with elbow extended
• 29 Mechanism
• 30
(child pulled, lifted, or swung by arm)
• 31
• 32 Physical • Arm held extended & pronated
• 33 examination • No swelling, deformity, or focal tenderness
• 34
• 35
• 36
• Forearm hyperpronation
• 37 Treatment OR
• 38 • Forearm supination & elbow flexion
• 39
CP USMLEVI1011d. LLC

41
42 End Block
Feedback
A'S
• 1
• 2
▪ 3 Previous . Next Lab Values. Notes Calculator.
• 4 reatrnent OR
▪ 5 * Forearm supination & elbow flexion
▪ 6
lArvILEWOr Id. LLC
▪ 7
▪ 8
▪ 9 Radial head subluxation (nursemaid's elbow) is a common injury in preschool children. The injury
• 10 characteristically occurs from innocent swinging of a young child by the arms or pulling a child's arm while
• 11 in a hurry. The child is usually not in distress at presentation but may cry at any attempt to move the elbow or
• 12 forearm_ The preferred treatment of this condition is closed reduction by applying pressure on the radial head
• 13 and hyperpronating the forearm. Forearm supination with elbow flexion is also commonly performed
• 14 but may be less successful and more painful. No post-reduction films are needed when the patient resumes
• 15 full use of the extremity. A pop may also be heard on successful reduction.
• 16
• 17
(Choices A and D) The likelihood of child abuse is very low as the mechanism of injury is consistent with the
• 18
patient's examination. If the historian is inconsistent with the details or the examination is incongruent with the
• 19
reported mechanism, a complete skeletal survey should be performed to look for healing fractures in other
• 20
parts of the body_
• 21
(Choices B and F) Immobilization with slings or casts is not needed, and the patient should experience
• 22
immediate return of normal function after manual reduction. Reduction is typically performed by primary care
• 23
or emergency physicians_
• 24
• 25
(Choice E) The diagnosis is made clinically as radiographs are often normal.
• 26
• 27 Educational objective:
• 28 Subluxation of the radial head is common in preschool children. The classic mechanism is swinging or
• 29 pulling a child by the arm. Full recovery after closed reduction by forearm hyperpronation confirms the
• 30 diagnosis.
• 31
• 32 References:
• 33
• 34 1. Randomized comparison of pain perception during radial head subluxation reduction
• 35 using supination-flexion or forced pronation.
▪ 36 2. Manipulative interventions for reducing pulled elbow in young children.
• 37
• 38
• 39
Copyright © UWorld Last updated: [7/30/2014]
• 40
• 41
• 42 End Block
Feedback
• 1
• 2 Item: 41 of 44 F' Mark
▪ 3 Previous Next Lab Values. Notes Calculator.
• 4-
5
▪ 6 A 9-month-old infant is brought to the emergency department with lethargy and tachypnea_ He was healthy
▪ 7 before developing fever and diarrhea four days ago. He has been taking some formula, but has had two to
▪ 8 three episodes of diarrhea with each bottle. He has lost three pounds (1A kg) since his routine check-up two
▪ 9 weeks ago. He has had one wet diaper in the past twenty four hours. On examination, his temperature is
• 10 102.5° F (39.1' C), pulse is 200/min, respiratory rate is 42/min, and blood pressure is 70/45 mm Hg_ He is
• 11 lethargic with decreased tone and decreased deep tendon reflexes. His mucous membranes are dry.
• 12 Cardiopulmonary exam reveals tachycardia and tachypnea. His abdominal exam is unremarkable. Capillary
• 13 refill is four seconds. Laboratory results are shown below.
• 14
Chemistry panel
• 15
Serum sodium 165 mailL
• 16
Serum potassium 4.5 mEq/L
• 17
Chloride 108 mEcilL
• 18
Bicarbonate 14 mEq/L
• 19
Blood urea nitrogen (BUN) 2G mgidL
• 20
Serum creatinine 0.8 mg/dL
• 21
Calcium 10.0 mg/dL
• 22
Blood glucose 98 mg/dL
• 23
• 24
Which of the following fluids should be used as a bolus in the resuscitation of this infant?
• 25
• 26
• 27 A. 0.9% saline [6296]
• 28 B. 0.45% saline [27%]
• 29
C. 5% dextrose [89C]
• 30
• 31 ID. 5% albumin [29C]
• 32 E Packed red blood cells [1%]
• 33
• 34
• 35 Explanation:
• 36
• 37 The signs and symptoms of hypernatremia are mainly neurologic and include lethargy, altered mental status,
• 38 irritability, and seizures_ Hypernatremia can also cause muscle cramps. muscle weakness, and decreased
• 39 deep tendon reflexes.
• 40

42 End Block
Feedback
• 1
• 2 Item: 41 of 44 V/lark
▪ 3 Previous Next Lab Values. Notes Calculator.
• 4- b. U.41buk saline viIpj
5 C. 5% dextrose [8%]
▪ 6
ID. 5% albumin [2%]
▪ 7
▪ 8 E. Packed red blood cells [1%]
▪ 9
• 10
Explanation:
• 11
• 12
The signs and symptoms of hypernatremia are mainly neurologic and include lethargy, altered mental status,
• 13
irritability, and seizures. Hypernatremia can also cause muscle cramps, muscle weakness, and decreased
• 14
deep tendon reflexes.
• 15
• 16 Hypovolemic hypernatremia develops secondary to renal losses (eg, diuretic use, glycosuria) or extrarenal
• 17 losses (eg, gastrointestinal upset, excessive sweating). Hypervolemic hypernatremia occurs due to
• 18 exogenous sodium intake or mineralocorticoid excess (eg, hyperaldosteronism).
• 19
• 20 When treating a patient with hypernatremia, the sodium must be slowly returned to normal. In this infant with
• 21 hypernatremia and dehydration, the initial goal is to stabilize him with fluid resuscitation as needed. When
• 22 giving intravenous fluid boluses, only isotonic solutions such as normal saline or lactated Ringer's should be
• 23 used.
• 24
• 25 (Choices B and C) Half normal saline and 5% dextrose are hypotonic solutions. As such, they should never
• 26 be used for initial resuscitation because they quickly exit the intravascular system and lower the sodium too
• 27 rapidly. Precipitous drops in sodium levels can cause cerebral edema.
• 28
• 29 (Choice 0) Multiple studies have demonstrated that the expensive colloid solutions are no better than
• 30 crystalloids at fluid resuscitation.
• 31
(Choice E) Packed red blood cells may be appropriate in initial fluid resuscitation when bleeding is a major
• 32
issue_ Because of the limited availability of packed red blood cells on short notice, however, isotonic solutions
• 33
remain the mainstay of initial fluid resuscitation (even in individuals with massive bleeding).
• 34
• 35
Educational objective:
• 36
Isotonic solutions such as normal saline are the fluid of choice for initial resuscitation in severe hypovolemic
• 37
hypernatremia.
• 38
• 39
Copyright © LIWorld Last updated: [12/29/2014]
• 40

42
Feedback End Block
1
• 2 Item: 42 of 44 A V-Mark
3 Previous Next Lab Values. Notes Calculator.
• 4-
5
6 A newborn infant is brought to the nursery for evaluation after delivery. The mother reports that the pregnancy
7 was uncomplicated, but she had only two prenatal visits. The infant was born via normal spontaneous vaginal
8 delivery and required no resuscitation. The infant has hepatosplenomegaly on examination. While in the
9 hospital, the infant requires treatment for anemia and hyperbilirubinemia. On subsequent examinations, the
• 10 infant has clear rhinorrhea and ulcerative lesions on his feet Which of the following congenital infections is
• 11 most likely in this patient?
• 12
• 13
• 14
A. Toxoplasmosis [4%]
• 15 B. Syphilis [55%]
• 16
C. Rubella [8%]
• 17
• 18
D. Cytomegalovirus infection [25%]
• 19 E. Human immunodeficiency virus infection [8%]
• 20
• 21
• 22 Explanation:
• 23
Congenital syphilis is contracted through the transplacental transmission of Treporierna parlidurn. Most
• 24
infants born with congenital syphilis are initially asymptomatic. Those with early symptoms present with
• 25
cutaneous lesions on the palms and soles, hepatosplenomegaly, jaundice, anemia, and rhinorrhea.
• 26
Radiographs demonstrate metaphyseal dystrophy and periostitis. Late congenital manifestations (presenting
• 27
after two years of age) include frontal bossing, high arched palate, Hutchinson teeth, interstitial keratitis,
• 28
saddle nose, and perioral fissures. These late manifestations can be prevented with early treatment of the
• 29
infant
• 30
• 31
Diagnosis of syphilis is made with serologic testing. The initial screening is with nontreponemal tests such as
• 32
the venereal disease research laboratory (VDRL) test, rapid plasma reagin (RPR) test, or enzyme
• 33
immunoassay (EIA) test; confirmatory testing is with treponemal tests such as the fluorescent treponemal
• 34
antibody absorption (FTA-ABS) test or the treponema pallidum particle agglutination assay (TPPAY
• 35
Parenteral penicillin G is the treatment of choice for syphilis_
• 36
• 37 (Choice A) Major clinical findings of congenital toxoplasmosis include hepatosplenomegaly, hydrocephalus,
• 38 chorioretinitis. and intracranial calcifications_
• 39
• 40 frq% rtrac.ani-c- inrith Ina-mrinn ke.rF rlafari-c. v
• 41

Feedback End Block


1
• 2 Item: 42 of 44 .11M Mark
3 Previous Next Lab Values. Notes Calculator.
• 4- t_ human immunodeficiency virus infection pluki A
5
6
Explanation:
7
8
Congenital syphilis is contracted through the transplacental transmission of Treponerna parlidurn. Most
9
infants born with congenital syphilis are initially asymptomatic. Those with early symptoms present with
• 10
cutaneous lesions on the palms and soles, hepatosplenomegaly, jaundice, anemia, and rhinorrhea.
• 11
Radiographs demonstrate metaphyseal dystrophy and periostitis. Late congenital manifestations (presenting
• 12
after two years of age) include frontal bossing, high arched palate, Hutchinson teeth, interstitial keratitis,
• 13
saddle nose, and perioral fissures. These late manifestations can be prevented with early treatment of the
• 14
infant
• 15 F

• 16 Diagnosis of syphilis is made with serologic testing. The initial screening is with nontreponemal tests such as
• 17 the venereal disease research laboratory (VDRL) test, rapid plasma reagin (RPR) test, or enzyme
• 18 immunoassay (EIA) test; confirmatory testing is with treponemal tests such as the fluorescent treponemal
• 19 antibody absorption (FTA-ABS) test or the treponema pallidum particle agglutination assay (TPPA).
• 20 Parenteral penicillin G is the treatment of choice for syphilis_
• 21
• 22 (Choice A) Major clinical findings of congenital toxoplasmosis include hepatosplenomegaly, hydrocephalus,
• 23 chorioretinitis, and intracranial calcifications.
• 24
• 25 (Choice C) Congenital rubella presents with sensorineural hearing loss, cataracts, heart defects,
• 26 hepatosplenomegaly, microcephaly, and thrombocytopenic purpura (commonly described as a "blueberry
• 27 muffin' rash).
• 28
• 29 (Choice 0) Congenital cytomegalovirus is characterized by intrauterine growth retardation,
• 30 hepatosplenomegaly, petechiae or purpura, microcephaly, chorioretinitis, sensorineural hearing loss, and
• 31 periventricular calcifications.
• 32
(Choice E) Infants with congenital human immunodeficiency virus (HIV) infection are usually clinically
• 33
asymptomatic at birth.
• 34
35
Educational objective:
36
Congenital syphilis presents early on with hepatosplenomegaly, cutaneous lesions, jaundice, anemia, and
37
rhinorrhea_ Metaphyseal dystrophy and periostitis may be seen on radiography.
38
39
Copyright © LIWorld Last updated: [12/29/2014]
• 40
• 41
a 42 End Block
Feedback
• 1
• 2 Item: 43 of 44 F' Mark
▪ 3 Previous Next Lab Values. Notes Calculator.
• 4-
5

▪ 6 A 15-year-old Caucasian male is brought to the office by his mother for the evaluation of a six-month history of
▪ 7 unstable gait and speech difficulty which are getting worse over time_ His past medical history is insignificant
▪ 8 He is not taking any medications, and denies smoking or alcohol consumption. His blood pressure is 120/10
▪ 9 mmHg and pulse is 80/min. Musculoskeletal examination showed scoliosis and feet deformity with 'hammer
• 10 toes.' The neurologic examination showed dysarthria, dysmetria, nystagmus, and absence of deep plantar
• 11 reflexes on lower extremities. What is the most common cause of death in this patient population?
• 12
• 13
• 14
A. Cardiomyopathy [699/0]
• 15 B. Renal failure [13?..6]
• 16
C. Diabetes-related complications [3%]
• 17
• 18
ID Malignancy [10%]
• 19 E. Septic shock [4%]
• 20
• 21
• 22 Explanation:
• 23
This clinical scenario is typical for Friedreich ataxia, which is the most common type of spinocerebellar
• 24
ataxias. It is an autosomal recessive disorder, and its symptoms usually begin before 22 years of age.
• 25
Neurological manifestations (e_g_, gait ataxia, frequent falling, dysarthria) result from degeneration of the spinal
• 26
tracts (spinocerebellar tracts, posterior columns, pyramidal tract). Non-neurological manifestations may also
• 27
occur, and these include concentric hypertrophic cardiomyopathy, diabetes, and skeletal deformities (e.g.,
• 28
scoliosis and 'hammer toes'). The median survival after disease onset rarely exceeds 20 years_ The most
• 29
common causes of death are cardiomyopathy and respiratory complications. Cardiomyopathy develops in up
• 30
to 90% of the patients_
• 31
• 32
(Choice B) Renal failure is not characteristic of Friedreich ataxia.
• 33
• 34 (Choice C) Diabetes develops in 10-20% of the patients with Friedreich ataxia, but it is a rare cause of death.
• 35
▪ 36 (Choice 0) Friedreich ataxia does not predispose to cancer.
• 37
▪ 38 (Choice E) Septic shock may develop due to respiratory complications, but it is not as common as cardiac
• 39 complications_
• 40
se
• 41
• 42 End Block
Feedback
• 1
• 2 Item: 43 of 44 Al II Mark
▪ 3 Previous Next Lab Values. Notes Calculator.
• 4
▪ A. Cardiomyopathy [69°/0]
5
▪ 6 B. Renal failure [13?..0]
▪ 7 C. Diabetes-related complications [3%]
▪ 8
D. Malignancy [10?..6]
▪ 9
• 10 E. Septic shock [4%]
• 11
• 12
Explanation:
• 13
• 14
This clinical scenario is typical for Friedreich ataxia, which is the most common type of spinocerebellar
• 15
ataxias. It is an autosomal recessive disorder, and its symptoms usually begin before 22 years of age_
• 16
Neurological manifestations (e_g_, gait ataxia, frequent falling, dysarthria) result from degeneration of the spinal
• 17
tracts (spinocerebellar tracts, posterior columns, pyramidal tract). Non-neurological manifestations may also
• 18
occur, and these include concentric hypertrophic cardiomyopathy, diabetes, and skeletal deformities (e_g_,
• 19
scoliosis and 'hammer toes'). The median survival after disease onset rarely exceeds 2G years. The most
• 20
common causes of death are cardiomyopathy and respiratory complications. Cardiomyopathy develops in up
• 21
to 90% of the patients_
• 22
• 23 (Choice 13) Renal failure is not characteristic of Friedreich ataxia.
• 24
• 25 (Choice C) Diabetes develops in 10-20% of the patients with Friedreich ataxia, but it is a rare cause of death.
• 26
• 27 (Choice 0) Friedreich ataxia does not predispose to cancer.
• 28
• 29 (Choice E) Septic shock may develop due to respiratory complications, but it is not as common as cardiac
• 30 complications_
• 31
Educational Objective:
• 32
Friedreich ataxia is the most common type of spinocerebellar ataxias. Remember the combination of
• 33
neurologic (ataxia, dysarthria), skeletal (scoliosis, feet deformities) and cardiac (concentric hypertrophic
• 34
cardiomyopathy) manifestations of the disease_ The most common causes of death are cardiomyopathy and
• 35
respiratory complications.
▪ 36
• 37
*very high-yield topic for LISMLE1
▪ 38
• 39
Copyright © UWorld Last updated: [8/22/2014]
• 40
• 41
• 42 End Block
Feedback
• 1
• 2 Item: 44 of 44 F Mark
▪ 3 Previous Next Lab Values. Notes Calculator.
• 4-
▪ 5
▪ 6 A previously healthy 6-year-old boy is brought to the physician after the sudden appearance of bruises
▪ 7 throughout his body. He has had no bleeding or recent trauma Three weeks ago, he had an upper
▪ 8 respiratory tract infection that resolved uneventfully. He takes no medications, has no allergies, and his
▪ 9 immunizations are up to date_ Family history is negative for bleeding or clotting disorders. Vital signs are
• 10 normal. Physical examination shows a cooperative, well-appearing child with scattered petechiae and
• 11 ecchymoses over the trunk and extremities. The rest of the examination is unremarkable. Laboratory results
• 12 are as follows:
• 13
Hemoglobin 13.5gidL
• 14
Platelets 10, CEDDip L
• 15
Leukocytes 1,000.1pL
• 16
• 17
• 18
Peripheral smear shows a few large platelets. What is the most appropriate next step in management of this
• 19
patient?
• 20
• 21 A. Bone marrow biopsy [716]
• 22
B. Intravenous antibiotics [19q
• 23
• 24 C. Intravenous immunoglobulin [38%]
• 25 D. Observation [28%]
• 26
E Platelet transfusion [25%]
• 27
• 28 • Splenectomy [2%]
• 29
• 30
Explanation:
• 31
• 32
• 33
• 34
• 35 Immune thrombocytopenia (ITP) is characterized by an abnormally low platelet count (<1113,0001pL), resulting
• 36 in increased propensity for bruising and bleeding. The disorder can affect people of any age but is most
• 37 common at age 2-5 years_ The pathogenesis involves antibodies that bind to platelets and subsequent
• 38 destruction of these antibody-platelet complexes in the spleen_ The condition is usually preceded by a viral
• 39 infection and presents with purpura and petechiae. In severe cases, patients may experience mucosal
• 40
• 41
• 42 End Block
Feedback
• 1
• 2 Item: 44 of 44 V-Mark
▪ 3 Previous Next Lab Values. Notes Calculator.
• 4
destruction of these antibody-platelet complexes in the spleen_ The condition is usually preceded by a viral
▪ 5
infection and presents with purpura and petechiae. In severe cases, patients may experience mucosal
▪ 6
bleeding. Laboratory studies shows isolated thrombocytopenia and megakaryocytes on peripheral smear.
▪ 7
▪ 8
The approach to treatment is different in children compared to adults. In children, the course is usually
▪ 9
self-limited with spontaneous recovery within 6 months. Observation without treatment is recommended
• 10
for children who experience only cutaneous symptoms, regardless of platelet count Intravenous
• 11
immunoglobulin (IVIg) (Choice C) or glucocorticoids are the first-line drugs in patients who experience
• 12
bleeding. Adults with ITP and platelet count <30,000!pL should receive glucocorticoids or intravenous
• 13
immunoglobulin as they are less likely to experience spontaneous recovery.
• 14
• 15 (Choice A) Bone marrow biopsy is not required in well-appearing children with isolated thrombocytopenia
• 16 and no other symptoms (eg, fever of unknown origin, leukopenia, anemia, unexplained weight loss, fatigue).
• 17
• 18 (Choice B) Antibiotics are not indicated in immune thrombocykopenia as the condition is not related to a
• 19 bacterial infection.
• 20
• 21 (Choice E) Platelet transfusion is very rarely indicated as it generally results in further platelet
• 22 destruction. It can be considered in severe cases with active or intracranial bleeding_ 111
• 23
• 24 (Choice F) The spleen is the site of platelet destruction and splenectomy can increase the life span and
• 25 quantity of platelets_ Splenectomy is a last resort for catastrophic bleeding or chronic ITP that is refractory to
• 26 IVIG and glucocorticoids_
• 27
Educational objective:
• 28
Immune thrombocytopenia should be suspected in children who develop isolated thrombocytopenia and
• 29
petechiae after a viral infection_ Children usually recover spontaneously within 6 months and require only
• 30
observation, regardless of platelet count Children with bleeding should receive intravenous immunoglobulin
• 31
or glucocorticoids.
• 32
• 33
• 34 References:
• 35 1. The American Society of Hematology 2011 evidence-based practice guideline for immune
• 36 thrombocytopenia.
• 37
• 38
• 39
Copyright © Morld Last updated: [11/2G/2014]
• 40 X
• 41
• 42 End Block
Feedback

Vous aimerez peut-être aussi